Cardiac Review

Pataasin ang iyong marka sa homework at exams ngayon gamit ang Quizwiz!

8. Disqualify conditions for heart transplant?

-Recent malignancy -Peptic ulcer disease -Severe obesity ^diabetes type 2 isn't a contraindications

Toxicity from which of the following medications may cause a client to see a green halo around lights? Digoxin Furosemide Metoprolol Enalapril

Digoxin

23) HF-

Digoxin, nitrates, Lasix, Ace inhibitors

Nitrates

Dilate blood vessels by reducing coronary artery spasm, increase coronary artery blood supply, and decrease oxygen demands

Calcium channel blockers

Dilate blood vessels, increase coronary artery blood supply, and decrease myocardial oxygen demands

35) The number one cause of AV blocks is ________.

Drugs

Whatnis the primary side if left side heart failure?

Dyspnea

11) Dysrhythmias are a result of what?

Either an alteration of the formation of the impulse through the SA node to the rest of the myocardium or irritability of the myocardium cells

Which of the following diagnostic tools is most commonly used to determine the location of myocardial damage? Cardiac catheterization Cardiac enzymes Echocardiogram Electrocardiogram

Electrocardiogram

26) Crystalloids-

Electrolytes, saline, LR

Take amlodipine on a ____ stomach

Empty

Which of the following actions is the first priority care for a patient exhibiting signs and symptoms of coronary artery disease? Decrease anxiety Enhance myocardial oxygenation Administer sublingual nitroglycerin Educate the client about his symptoms

Enhance myocardial oxygenation

DNase (deoxyribonuclease) (Elase)

Enzymed used over a Venous statis ulcer

17) Stimulates A and B adrengenric agonist used for HR, BP SVR and cerebral and coronary blood flow-

Epinephrine

Which of the following symptoms might a client with right sided heart failure exhibit? Adequate urine output Polyuria Nocturia Polydipsia

Nocturia

4. What is rebound hypertension?

Occurs when blood pressure rises after you stop taking or lower the dose of a drug

Morphine

Opioid Controls pain; reduces myocardial oxygen demand

43) List the abnormalities in each dysrhythmia:Sinus Brady Cardia

SB: rate (too low)

42) List the abnormalities in each dysrhythmia: Sinus Tachycardia

ST rate (too high)

44) List the abnormalities in each dysrhythmia:SupraVenticular TacyCardia

SVT: rate (too high) May also have shortened PR interval

48) List the abnormalities in each dysrhythmia:2nd Degree Heart Block type 1

Second degree AV block (Type I): irregular: Consecutively prolonged PR interval until a QRS complex is dropped More Ps than QRSs

49) List the abnormalities in each dysrhythmia: 2nd Degree heart block type 2

Second degree AV block (Type II): irregular: P waves are not conducted More Ps than QRSs

5) 0.06-0.12 is the normal duration of the QRS complex: True or false?

True

63) In cardio version you set the Defibrillator to "synchronous mode": True or false

True

70) Cardioversion is used for tachydysrhythmias: true or false?

True

Ace inhibitor cause orthostatic hypotension: true or false

True

Beta blockers compete with adrengic recptor site: true or false

True

Carvedilol (Corg) is a beta blocker. True or false?

True

Diltiazem (Cardizem) is a Calcium Channel Blocker: True or false

True

Treat of hypertension is typically lifelong: true or false?

True

True or false: nesiritide (Natrecor) causes ventricular dysrhytmias?

True

8.A gallop consist of several heart sounds: true or false?

True (gallop rhythm refers to a (usually abnormal) rhythm of the heart on auscultation. It includes three or four sounds, thus resembling the sounds of a gallop.)

4.Methylprednisone is used to treat or prevent Cytokine Release Syndrome?

Used to prevent CRS is used before, Hydrocortisone is used 30mins afterwards to treat

Pleural effusion(1538)

fluid is collecting in the inner lining of lungs

Nesiritide (Natrecor)

is the first of the drug class called human BNPs. It reduces pulmonary capillary pressure, helps improve breathing, and causes vasodilation with increase in stroke volume and cardiac output

5) Digoxin

is used for A fib

Atrial Fibrillation

occurs when the normal rhythmic contractions of the atria are replaced by rapid irregular twitching of the muscular heart wall

Defibrillation (1549)

shocking the heart to stop ventricular fibrillation, which prevents the heart from pumping blood

Atherosclerosis (1588)

the arteries are filling up with plaque and beginning to close

Aliskiren Hemifumarate (Tekturna)

the first antihypertensive drug that is a direct renin inhibitor

Heart failure(1564)

the heart cannot pump blood correctly

Ventricular Fibrillation

the rapid, irregular, and useless contractions of the ventricles

Activase Alteplase (recombinant)

thrombolytic

Urokinase (Abbokinase)

thrombolytic

streptokinase (Streptase)

thrombolytic

QRS duration

time required for depolarization of both ventricles (0.06-0.12 secs).

cortisol causes vasoconstriction and is a stress response hormone: True and false

true

Indomethacin (Indocin)

used for heart inflammtion

QRS complex

ventricular depolarization (contraction)

T wave

ventricular repolarization (Relaxtion)

16) Do not take nitro

with ED meds

Polycythemia(1541)

you have too many red blood cells in your blood

Bradycardia(1546)

slow, steady heart rate

32. For Buerger's disease, the most important patient behavior is ________________________. (1595)

smoking cessation

nuchal rigidity

stiffness in cervical neck area

Inotropic

strength of contraction

Pulse Scale +2

suggesting a slightly more diminished pulse than normal, may be obliterated with pressure

Ace inhbitors can cause ____________.

Hyperkalemia

Rosuvastatin (Crestor)

Hyperlipidemia

Large Box Method

count the number of large boxes between 2 consecutive RR and divide into 300 for ventricular rate

1 The nurse is monitoring a patient who is receiving antithrombolytic therapy in the emergency department because of a possible MI. Which adverse effect would be of the greatest concern at this time? a Dizziness b Blood pressure of 130/98 mm Hg c Slight bloody oozing from the IV insertion site d Irregular heart rhythm

d

9) Clopidogrel (Pavix)

is an antiplatelet

What is the most appropriate nursing response to a myocardial infarction client who is fearful of dying? "Tell me about your feelings right now" "When the doctor arrives, every will be fine" "This is a bad situation, but you'll feel better soon." " Please be assured we're doing everything we can to make you feel better"

"Tell me about your feelings right now"

8) Enoxaparin

(Lovenox)

2.Splinter hemrrohages

(or haemorrhages) are tiny blood clots that tend to run vertically under the nails. ... In certain conditions (in particular, infective endocarditis), clots can migrate from the affected heart valve and find their way into various parts of the body.

Angiotensin-Converting Enzyme (ACE) Inhibitors

****Observe patient closely for a precipitous drop in blood pressure within 3 hr of initial dose;****

Dobutamine (Dobutrex)

*class*: beta-adrenergic agonist, inotropic *Indication*: short term management of heart failure *Action*: Dobutamine has a positive inotropic effect (increases cardiac output) with very little effect on heart rate. Stimulates Beta1 receptors in the heart. *Nursing Considerations*: - Monitor hemodynamics: hypertension, ↑HR, PVCs - Skin reactions may occur with hypersensitivity - Beta blockers may negate therapeutic effects of dobutamine - Monitor cardiac output - Monitor peripheral pulses before, during, and after therapy - DO NOT confuse dobutamine with dopamine

5. Chills, petechial, influenza like symptoms?

- Infective endocarditis

U wave

- repolarization of the purkinje fibers - seen in patients with hypokalemia

6. Dental appointment for an extraction next month, what do you ask?

-"Have you contacted your physician about your tooth extraction?"

Rhythm strip times

-1 box is 1mm -1 box is 0.04seconds -5 boxes is 5mm -5 boxes is 5mm

2. Which Pt teaching would help prevent venous status?

-Avoid crossing leg -Elastic stockings -Elevate legs

26) How does atropine treat bradycardia?

-Blocks the parasympathetic impulses to the heart

3. A Pt has infective endocarditis, Nursing Dx r/t generalized weakness

-Decreased activity during acute phase

Oliguria, jugular vein distention

-Heart failure right side

Second Degree Type II (Mobitz II or Classical)

-In second-degree type II AV nodal block (a.k.a. Mobitz Type II AV block), the AV node becomes completely refractory to conduction on an intermittent basis. For example, three consecutive P waves may be followed by a QRS complex, giving the ECG a normal appearance, then the fourth P wave may suddenly not be followed by a QRS complex since it does not conduct through the AV node to the ventricles. -The PR interval may be normal or prolonged, however it is constant in length unlike second-degree AV block Mobitz Type I (Wenckebach) in which the PR interval progressively lengthens until a P wave is not conducted. A second-degree type II AV block indicates significant conduction disease in this His-Purkinje system and is irreversible (not subject to autonomic tone or AV blocking medications). This is a very important distinguishing factor compared to second-degree type I AV block. Because of this, a permament pacemaker is indicated in every patient with second-degree type II AV block.

List the abnormalities in each dysrhythmia.

-ST: rate (too high) -SB: rate (too low) -SVT: rate (too high) May also have shortened PR interval. -A flutter: atrial rate (too high) May be irregular Ventricular rate may also be high -A fib: no P waves Irregularly irregular Ventricular rate may also be high -First degree AV block: prolonged PR interval -Second degree AV block (Type I): irregular: Consecutively prolonged PR interval until a QRS complex is dropped More Ps than QRSs -Second degree AV block (Type II): irregular: P waves are not conducted More Ps than QRSs -Third degree AV block: no relationship between P and QRS More Ps than QRSs Prolonged QRS -PVC: prolonged QRS -V Tach: rate (too high) No visible P waves QRS prolonged -V Fib: no discernible waveforms

4. Patient teaching to endocarditis Pt?

-Taker antibiotic as prescribed

9. Immunosuppressive are given in conjunction with corticosteroids: true or false?

-True

27) Adenosine is used to treat Supraventricular Tachycardia: Ture or false?

-True (decreases heart rate and reduces conduction velocity, especially at the AV node)

1)Syncopal episodes-

-Vasovagal syncope; A sudden drop in heart rate and blood pressure leading to fainting, often in reaction to a stressful trigger (Bearing down to poop may stimulate this, due to parasympathetic nervous response)

Anticoagulant (Heparin) is different from anti-platelet (Aspirin): True or false?

-any drug that, when added to blood, prevents it from clotting. Anticoagulants achieve their effect by suppressing the synthesis or function of various clotting factors that are normally present in the blood. -antiplatelets keep clots from forming by inhibiting the production of thromboxane, anticoagulants target clotting factors, which are other agents that are crucial to the blood-clotting process.

7. What is a major cause of valve disease?

-rheumatic fever

Second Degree Type I (Mobitz or Wenckebach)

-varying failure of conduction through the AV node occurs, such that some P waves may not be followed by a QRS complex. Unlike first-degree AV nodal block, a 1:1 P-wave-to-QRS-complex ratio is not maintained. Second-degree type I AV block is specifically characterized by an increasing delay of AV nodal conduction until a P wave fails to conduct through the AV node. This is seen as progressive PR interval prolongation with each beat until a P wave is not conducted. There is an irregular R-R interval. Sometimes when the block is consistent, the QRS complexes are said to demonstrate "group beating."

4) A rhythm strip 1mm square is ____ seconds.

0.04

PR interval

0.12-0.20

1. A 62-year-old patient is admitted to the hospital with a diagnosis of congestive heart failure. She has had angina for many years, and recently her symptoms have been getting worse as a result of arteriosclerosis. In establishing a patient care plan, what is the primary goal of treatment? 1. Reduce the workload of the heart. 2. Promote rest for the heart. 3. Reduce fluid retention. 4. Reduce circulating blood volume.

1

13. A 62-year-old patient has a history of angina pectoris. To decrease the pain from angina pectoris, what should the patient do? 1. Take a cardiac glycoside at the first symptom of cardiac pain. 2. Avoid taking more than three or four nitroglycerin pills daily. 3. Take nitroglycerin sublingually qid (three times daily). 4. Take nitroglycerin sublingually prophylactically before strenuous exercise.

1

14. A 75-year-old patient is diagnosed with heart failure. The nursing diagnosis of activity intolerance, related to dyspnea and fatigue, would be appropriate. What nursing intervention would be most appropriate for this diagnosis? 1. Plan frequent rest periods. 2. Allow the patient to shower. 3. Encourage the patient to perform all ADLs. 4. Encourage fluid intake of 3000 mL/day

1

16. Dependent edema of the extremities, enlargement of the liver, oliguria, jugular vein distention, and abdominal distention are all signs and symptoms of what problem? 1. Right-sided heart failure 2. Left-sided heart failure 3. Cardiac dysrhythmias 4. Valvular heart disease

1

18. A 55-year-old female was brought to the hospital by ambulance after telling her husband that she had intense chest pain, anxiety, and nausea. Her admitting diagnosis is suspected myocardial infarction. When providing care for the patient in the emergency department, what must the nurse understand about a myocardial infarction? 1. It involves a critical reduction in blood supply to the myocardium. 2. There is a marked increase in cardiac output. 3. A sudden irregularity of cardiac contraction occurs. 4. There is a marked decrease in cardiac output.

1

4. When a 57-year-old male patient comes to the clinic for a periodic check-up, he receives a diagnosis of angina pectoris, with no subsequent cardiac involvement. His health care provider prescribes nitroglycerin. What explanation would the nurse give to this patient about why this medication is given sublingually? 1. Superficial blood vessels promote rapid absorption of the medication. 2. Stomach acids destroy the medication. 3. Saliva helps break down the medication for absorption. 4. The medication is too rapidly absorbed in the stomach

1

8. A 72-year-old patient is admitted to the medical floor with a diagnosis of HF. Which assessment findings are consistent with the medical diagnosis? (Select all that apply). 1. Increase in abdominal girth 2. Weight loss of 6 pounds in the past 2 weeks 3. Pitting edema 4. Nervous tremors 5. Night sweats

1,3

27. The nurse is providing patient teaching for a 58-year-old patient with Raynaud's disease. What information should be included? (Select all that apply.) 1. Avoid cold. 2. Warm hands and feet with heating pad. 3. Practice stress reduction techniques. 4. Comply with smoking cessation. 5. Limit caffeine intake.

1,3,4,5

A patient is experenicing renal hypoperfusion due to shock. The physician should be notified if the urine output is below?

30mL/hr

69) Pacemakers last up to ____ years.

10

41) What is the worst type of dysrhythmia? A) Premature Ventricular Contractions B) Ventricular Tachycardia C) Ventricular Fibrillation D) Atrial Flutter

C) Ventricular Fibrillation

small box method

1500/# of small boxes between R waves

11. An 86-year-old patient is receiving an intravenous infusion at 83 mL/hour via an electronic infusion pump. Why is it so vitally important that the IV lines of older adult patients be monitored carefully? 1. These patients do not become dehydrated very easily. 2. They are at an increased risk for developing fluid overload of the circulatory system. 3. They are at an increased risk for developing a venous infection. 4. Aging patients present an increased risk for developing thrombophlebitis in the peripheral system

2

12. A 34-year-old patient with a history of IV drug use is diagnosed with acute infective endocarditis. Which nursing intervention for this patient is most appropriate? 1. Early ambulation 2. Restricted activity for several weeks 3. Low-calorie diet 4. Dilution of blood by increased fluid intake

2

17. What is the primary function of patient teaching after a myocardial infarction? 1. Explaining the disease process 2. Assisting the patient in developing a healthy lifestyle 3. Describing the precipitating causes and onset of pain 4. Educating the patient on causative factors that initiate cardiac vasoconstriction

2

20. An elderly patient develops arrhythmias while she is being admitted to the hospital. What medication should the nurse expect to administer first? 1. Morphine sulfate, USP 2. Lidocaine hydrochloride, USP 3. Nitroglycerin (Nitrostat) 4. Meperidine hydrochloride (Demerol)

2

22. What is the most useful noninvasive diagnostic tool for evaluating the patient with heart failure? 1. Coronary angiography 2. Echocardiogram 3. Electrocardiogram 4. Thallium scanning

2

24. Thrombolytic agents such as streptokinase and tissue plasminogen activators are agents used to dissolve blood clots. When is it most effective to give these drugs to a patient with acute MI signs and symptoms? 1. In the first 24 hours 2. In the first 30 minutes to 1 hour 3. In the first 72 hours 4. In the second 6 hours after an MI

2

23. Which of the following actions would the nurse expect to be used to treat heart failure? (Select all that apply.) 1. Cardiotonic drugs (digitalis) 2. Diuretic agents 3. Generous fluid intake 4. ACE inhibitors, beta-adrenergic blockers (carvedilol), nitrates 5. Oxygen therapy

2,4,5

2. The nurse teaches the client with angina about the common expected side effects of nitroglycerin including: A. Headache B. High blood pressure C. Shortness of breath D. Stomach cramps

2. Headache

Exceeding which of the following serum cholesterol levels significantly increases the risk of coronary artery disease? 100 mg/dL 150 mg/dL 175 mg/dL 200 mg/dL

200 mg/dL

15. A patient recovering from an MI is being prepared for discharge. What instruction should the patient be given? 1. Remain inactive until healing is complete. 2. Remain at home and avoid exposure to cold temperatures. 3. Begin a cardiac rehabilitation program. 4. Perform isometric exercises in a relaxed environment.

3

2. What is the best nursing action that will lessen the severity of a patient's orthostatic hypotension? 1. Turn him from side to side every 2 hours. 2. Limit times he will have to get in and out of the bed. 3. Change his position routinely, especially from horizontal to vertical. 4. Encourage him to move very slowly

3

21. When a patient returns to the unit following cardiac catheterization, what nursing activity should immediately follow the taking of vital signs? 1. Placing the patient in a warm bed and encouraging sleep 2. Providing the patient with fluids 3. Assessing the patient's peripheral pulses 4. Reapplying the patient's dressing where the dye was injected

3

26. A 63-year-old patient has Buerger's disease. What is the most important aspect of patient compliance to decrease signs and symptoms of Buerger's disease? 1. Low-fat diet 2. Weight loss 3. Cessation of tobacco use 4. Keeping extremities warm

3

5. Prior to administering a dose of digoxin to an assigned patient, the nurse observes that the patient's temperature is 37.7 degrees Celsius (99.8 degrees Fahrenheit) and her pulse rate is 100. What is the most appropriate nursing action? 1. Notify the charge nurse. 2. Recognize that these are signs of digoxin toxicity and withhold the dose. 3. Administer the medication. 4. Administer the medication, but tell the charge nurse that the patient's pulse rate is higher than normal.

3

38. Which psychosocial behaviors are more likely to be associated with increased cardiovascular symptoms? (1546) 1. Frequently in a hurry and easily irritated 2. Easygoing and usually enjoys life 3. Neat, organized, and pays attention to detail 4. Pessimistic and generally expresses negativity

38. Answer 4: Recent studies indicate that type D personality has the highest risk for cardiovascular problems because of increased anxiety and depression. The type A personality who is in a hurry and often angry or irritated was formerly believed to have the highest risk.

39. The patient's cardiac monitor shows a regular rhythm with a rate of 65 beats/min, P waves precede each QRS complex, QRS complexes are symmetrical and regularly spaced, and a normal T wave shows repolarization. What is the nurse's interpretation of monitor display? (1546) 1. Vital signs should be immediately assessed. 2. The monitor indicates a normal sinus rhythm. 3. The monitor is showing a benign dysrhythmia. 4. The patient should be assessed for chest pain.

39. Answer 2: The monitor is showing a normal sinus rhythm. (Note to student: If there is ever any doubt about the monitor function or display or if you doubt your interpretation of the ECG tracing, just check on the patient.)

10. A 67-year-old patient has a diagnosis of hypertension. She is being discharged from the hospital. What would her teaching include? 1. Instruction in consuming a bland diet 2. Instruction to limit sodium intake to 4 g/day 3. Encouragement to begin a vigorous exercise program 4. Education on continuing to take antihypertensive medications as prescribed

4

19. What are the modifiable risk factors for coronary artery disease (CAD)? 1. Diabetes, family history 2. Family history, smoking 3. Smoking, heredity 4. High cholesterol, obesity

4

25. A patient is admitted with a diagnosis of possible aortic abdominal aneurysm. What is the most important factor to monitor as a possible complication? 1. Body temperature 2. Skin turgor 3. Respiratory rate 4. Blood pressure

4

3. When caring for a patient whose health care provider has ordered furosemide (Lasix), what will the nurse recognize when the medication is having the desired effect? 1. The patient becomes very thirsty. 2. The patient's resting heart rate slows. 3. The patient's blood pressure is reduced. 4. Production of urine is increased

4

6. What symptom would indicate possible thrombophlebitis? 1. Pain along a vein 2. Severe cramping 3. Edema 4. Area around a vein that is warm to the touch

4

7. When a patient is receiving heparin therapy, what would be the nurse's most appropriate action? 1. Observe him for cyanosis. 2. Assess degree of edema in all extremities. 3. Give the injection intramuscularly. 4. Observe emesis, urine, and stools for blood.

4

9. A 10-year-old patient is diagnosed with rheumatic fever. Of all the manifestations seen in rheumatic fever, which is most likely to lead to permanent complications? 1. Sydenham's chorea 2. Erythema marginatum 3. Subcutaneous nodules 4. Carditis

4

30) Cardiomyopathy is anything the ejection fraction is below ___.

40%

29. Older Americans should have their cholesterol tested once every ________ years. (1543)

5

Replenish nitroglycerin supply every __ months or before the expiration date.

6

68. A 56-year-old man arrives in the emergency department seeking care. He is complaining of crushing chest pain. The pain is radiating down his left shoulder and arm. The patient, who has a history of an- gina, reports the pain is more severe and has lasted longer than a typical angina episode. (1559-1563) a. What does the nurse anticipate this patient's medical diagnosis will be? b. Discuss the pathology of this type of occurrence. c. During the medical diagnostic workup of this patient, what tests are likely to be ordered? d. What are the goals of the medical management of this patient? e. Identify four nursing interventions for this patient's care.

68. a. Myocardial infarction b. A myocardial infarction results from the occlusion of a major coronary artery or one of its branches. This leads to ischemia. c. 12-lead ECG, chest radiograph, cardiac fluoroscopy, myocardial imaging, echocardiogram, PET scan, or multigated acquisition scanning (MUGA). Blood workup may include electrolytes, CBC, ESR, serum cardiac markers: CK-MB, myoglobin, troponin-I d. Prevention of further tissue damage, interventions to promote tissue perfusion e. Monitor vital signs, administer oxygen, monitor pain, administer medications as ordered

70. A home health nurse is caring for a 73-year-old man who has heart failure. He has been hospitalized twice for exacerbations, but is currently stable and able to live independently in his own home. a. waht changes related to aging would the nurse expect to find for the patient's cardiac system? (1543, 1581) b. What are common signs and symptoms of heart failure? (1564, 1656) c. Identify medication classes that are used in the medical management of heart failure. (1567-1569) d. Discuss patient teaching points for heart failure. (1570)

70. a. Changes in the cardiac musculature lead to reduced efficiency and strength, resulting in decreased cardiac output. Disorientation, syncope, and decreased tissue perfusion to organs and other body tissues can occur as a result of decreased cardiac output. Arterial disease resulting from the aging process causes hypertension because of the increased cardiac effort needed to pump blood through the circulatory system. Edema, secondary to heart failure, may cause tissue impairment in the immobile older adult. Immobility leads to venous stasis, venous ulcers, and poor wound healing. It also increases the risk of venous thrombosis and embolus formation. Older adults with cardiac disease often receive several medications. Even with lower doses of medications, the older adult may suffer toxicity, since the rate of drug metabolism and excretion decreases with age. Independent older adults with cardiac conditions should receive adequate teaching regarding medication, diet, and warning signs of complications. Encourage them to maintain regular contact with the health care provider and to seek care at the first sign. b. Signs and symptoms of heart failure include: Decreased cardiac output • Fatigue • Anginal pain • Anxiety • Oliguria • Decreased gastrointestinal motility • Pale, cool skin • Weight gain • Restlessness Left ventricular failure • Dyspnea • Paroxysmal nocturnal dyspnea • Cough • Frothy, blood-tinged sputum • Orthopnea • Pulmonary crackles (moist popping and crackling sounds heard most often at the end of inspiration) • Radiographic evidence of pulmonary vascular congestion with pleural effusion Right ventricular failure • Distended jugular veins • Anorexia, nausea, and abdominal distention • Liver enlargement with right upper quadrant pain • Ascites • Edema in feet, ankles, sacrum; may progress up the legs into thighs, external genitalia, and lower trunk c. Heart failure is managed with digoxin, vasodilators, ACE inhibitors, beta blockers, and angiotensin II receptor blockers. Nesiritide is the first of the drug class called human BNPs. It reduces pulmonary capillary pressure, improves breathing, and causes vasodilation with increase in stroke volume and cardiac output. d. Teach the patient to monitor for signs and symptoms of recurring problems such as shortness of breath; swelling of ankles, feet, or abdomen; and frequent nighttime urination. Plan activity to provide for rest periods; take medications as prescribed; report signs of nausea, pain, lightheadedness, and syncope to the doctor. Eat foods high in potassium and low in sodium if taking diuretics. Avoid alcohol when taking vasodilators.

71. The nurse is working in an ambulatory walk-in clinic in an urban area. Many of the patients are home- less and the clinic staff sees many patients who have venous stasis ulcers. a. What is the pathophysiology of stasis ulcers and why are the homeless at particular risk for this disorder? (1600) b. Describe how the nurse would use PATCHES to assess venous disorders. (1582) c. Identify the signs and symptoms of venous stasis ulcers. (1600) d. Review the treatment options available for venous stasis ulcers and suggest how the nurse can assist homeless patients with these options. (1601, 1602)

71. a. Venous stasis ulcers result from vein insufficiency causing stasis of blood. People who are homeless spend a lot of time with their legs in a dependent position. This puts greater strain on vessels. The corrective measure is to lie down and elevate legs, but this is not always possible for homeless persons. Poor nutrition, exposure to the elements, and lack of access to hygienic facilities impairs healing of ulcers. b. P for pulses: Assess the patient's affected extremity first. Compare the findings with previous ones or correlate them with the patient's signs and symptoms. Pulses should be present in venous disorders, but edema may interfere with palpation. Use a Doppler as needed. A for appearance: Note whether the extremity is pale; mottled; cyanotic; or discolored red, black, or brown. T for temperature: If the problem is venous, the extremity will feel normal or abnormally warm. C for capillary refill: Capillary refill is normally less than 2 seconds, but it may be extended when the patient has PVD. H for hardness: Palpate the extremity to determine whether the tissues are supple or hard and inelastic. Hardness may indicate long-standing PVD, chronic venous insufficiency, lymphedema, or chronic edema. Hardened subcutaneous skin also increases the risk of stasis ulcers. E for edema: Pitting edema frequently indicates an acute process, and nonpitting edema may be seen with chronic conditions, such as venous insufficiency. Assess both extremities for edema and compare and document the findings. S for sensation: In addition to asking the patient about pain, ask if he or she has other abnormal sensations, such as numbness or tingling, or heat or cold. c. Visibly ulcerated skin having dark pigmentation, dryness, scaling, and edema may occur. Dull aching pain relived by elevation of the extremity. Peripheral pulses are usually present with venous insufficiency. Pain, aching, and cramping associated with venous disorders are usually relieved by activity and/or elevating the extremity. d. The focus is on promotion of wound healing and preventing infection. Dietary management including adequate protein intake with supplements of vitamin A and C, and mineral zinc. Débridement of necrotic tissue, antibiotic therapy, and protection of ulcerated areas. Homeless patients may need assistance in obtaining medication or nutritious foods. The nurse should suggest ways to adapt wound care and instruct about elevating legs whenever possible.

Sinus Bradycardia

<60 bpm

4) Chronotropic

= Heart rate

Sinus Tachycardia

>100 (100-150) normal sinus rhythm

8)What diagnostic test allows observation of real time movement via radiography? A)Fluoroscopy B)Angiography C)Echocardiography D)Cardiac monitoring

A Fluoroscopy is the diagnostic test that allows observation of real time movement via radiography. This is invaluable for placement of pacemakers and intracardial catheter placement. Angiography is a series of radiographs taken following administration of contrast dye. This test aids in diagnosis of vessel occlusion, pooling in various heart chambers, and congenital abnormalities. Echocardiography uses high-frequency ultrasound directed at the heart. The reflected sound is recorded, outlining size, shape, and position of cardiac structures. This is useful in detecting pericardial effusion, evaluating ventricular function, cardiac chamber size and contents, ventricular muscle and septal motion and thickness, cardiac output, cardiac tumors, valvular function, and congenital heart disorders. Cardiac monitoring records the cardiac electrical activity of patients. A cardiac monitor displays information transferred via the conductive electrodes, which transfer electrical activity of the heart and relay it to a video display screen. This is useful for patients with known or suspected arrhythmias, or patients who may be likely to develop arrhythmias.REF: Page 1537

1)The nurse is caring for a patient with a new pacemaker. Nursing care for this patient would include which of the following? A) Monitoring the heart rate and rhythm by apical pulse and ECG patterns B) Bed rest for 24 hours C) Scheduling of an MRI to verify pacemaker placement D) Performing range-of-motion exercises every 4 hours to the arm on the pacemaker side for the first 2 days

A Nursing care for a patient with a new pacemaker would include closely monitoring heart rate and rhythm by apical pulse and ECG patterns. Also, vital signs and level of consciousness are checked frequently. The insertion site is observed for erythema, edema, and tenderness, which could be signs of infection. Bed rest for 24 hours is not necessary for a patient with a new pacemaker. Nursing care for a patient with a new pacemaker would include bed rest for the first few hours only, unless unexpected complications occurred. Scheduling of an MRI to verify pacemaker placement would be inappropriate. The patient must avoid proximity to high-output generators and to large magnets, such as MRI scanners. Performing range of motion every 4 hours to the arm on the pacemaker side would be inappropriate. The arm on the pacemaker side should be immobilized for the first few hours, and the patient should not raise the arm above his or her head for several days. After this time, normal activities can be resumed.REF: Page 1551

52. The life support system that uses special techniques, ventilation equipment, and therapies for emergency situations is .

ANS: advanced cardiac life support (ACLS) advanced cardiac life support ACLS ACLS is a life support system that uses special techniques, ventilation equipment, and therapies for emergency situations. PTS: 1 DIF: Cognitive Level: Knowledge REF: Page 1550 OBJ: 9 TOP: ACLS KEY: Nursing Process Step: N/A MSC: NCLEX: Physiological Integrity

29) Irregular, regular is used to describe ___________.

A Fib

Embolus (1559)

A blood clot or foreign matter travels in the bloodstream

Ischemia(1553)

A body part or organ is not getting enough blood, thus causing pain

Coronary artery disease (1552)

A condition that causes the blood to stop going to the arteries around the heart

46) List the abnormalities in each dysrhythmia: Atrial Fibrillation

A fib: no P waves Irregularly irregular Ventricular rate may also be high

45) List the abnormalities in each dysrhythmia:Artial Flutter

A flutter: atrial rate (too high) May be irregular Ventricular rate may also be high SAW TOOTH PATTERN

53. The nurse explains that the heart has the ability to contract in a rhythmic pattern that is called .

ANS: automaticity Automaticity is the special ability of the myocardium to contract in a rhythmic pattern. PTS: 1 DIF: Cognitive Level: Knowledge REF: Page 1534 OBJ: 2 TOP: Automaticity KEY: Nursing Process Step: Assessment MSC: NCLEX: Physiological Integrity

An Open mitral commissurotomy is the fusing of the mitral valve leaflets to treat stenosis: True or false?

False (You are splitting them, not fusing)

6)Wide complex QRS indicates: A) A Block B)A Lazy heart C) Tachycardia D)Bradycardia

A) A block (there is damage or trouble conducting)

15)Medical management of Sinus bradycardia would include which drugs? A) Atropine, dopamine, epinephrine B) Albuterol (INH), Dopamine, Epinephrine C) Lithium, midodrine, phenylephrine D) A pacemaker

A) Atropine, dopamine, epinephrine

39) Ventricular Tachycardia is usually caused by toxicity to which drugs: A) Isoproterenol and Digitalis B) Aspirin and Aleve C) Procainamide and Amiodarone D) Lisinopril and Diazepam

A) Isoproterenol and Digitalis

2)Av node is in the _______. A) Right atrium B) Left Atrium C) Left Ventricle D) Right Ventricle

A) Right Atrium (Comes out of the interatrial septum near the coronary sinus)

56) "Slows conduction through the AV node. Decreases conduction velocity and prolongs the effective refractory period in the AV node." Describes which drugs? A) digoxin, lidocaine B) procainamide, lidocaine C) procainamide,quinidine D) quinidine, digoxin

A) digoxin, lidocaine

54) "Depresses Na conductance, increases action Potential duration (APD) and effective refractory period (ERP), decreases membrane responsiveness." Describes which drugs? A) disopyramide, procainamide,quinidine B) procainamide,quinidine, digoxin C) disopyramide, procainamide, lidocaine D) quinidine, digoxin, lidocaine

A) disopyramide, procainamide,quinidine

38) Medical management of Ventricular tachycardia usually focuses on the use of which IV drugs: A) Procainamide or Amiodarone B) Lisinopril or Diazepam C) Aspirin or Aleve D) Cardioversion

A) procainamide or amiodarone

12) Dobutamine, dopamine, epinephrine, midodrine, phenylephrine, fenoldopam are what kind of drug? A) Vasopressors B) Antihypertensive C) Allergy medication D) Beta Blockers

A)Vasopressors (Antihypotensive, Vasoactive adrengenics)

The nurse is preparing to adminster adenosine to patient with supraventricular tachycardia. Important considerations before administration inculde which of the following? A. Constant ecg monitoing B. Inital recommended dosage of 12mg IV bolus C. Rapid IV bolus administration D. Saline flush following the bolus E. Long half of adenosine, which may prolong adverse medications effects

A, C, D

4)Which of the following is/are true statements regarding angina pectoris? Select all that apply. A)Angina indicates a lack of oxygen and blood supply to the heart. B)Angina only occurs at rest. C)Angina may resemble heartburn or indigestion. D)Angina is usually relieved by nitroglycerin. E)Angina may appear as jaw pain.

A,C,D,E Angina indicates a lack of oxygen and blood supply to the coronary arteries, usually as a result of a narrowing of the lumen. When the myocardial oxygen demand exceeds the available supply, ischemia occurs, resulting in chest pain or angina. Angina may resemble heartburn or indigestion in some patients. Patients may not have the "classic" symptom of chest pain, especially older adults and women. Angina is usually relieved by nitroglycerin. This is one way angina is differentiated from a myocardial infarction—pain from the MI is generally not relieved by nitroglycerin. The patient should be instructed to take a nitroglycerin tablet and lie down when experiencing an anginal attack. If the pain is not relieved, the patient may take two more nitroglycerin tablets, 5 minutes apart. If the pain is still unrelieved after the third nitroglycerin, he or she needs to seek emergency medical attention. Although the pain from angina is typically substernal chest pain, it may be present in different ways: it may radiate to the chest, arm, or jaw. Angina may appear as posterior thoracic or jaw pain only. Angina typically occurs with an increased cardiac workload (e.g., exercise, heavy meal, cold, stress, etc.) and may disappear at rest and/or with administration of nitroglycerin.REF: Page 1552-1557

When a pt is taking an adrenergic drug the nurse should expect to observe which of the following effects? A. Increased HR B. Bronchial constriction C. Perioheral vasodilation D. Increased GI peristalsis

A. Increased hr

Benazepril (Lotensin)

ACE inhibitor

Captopril (Capoten)

ACE inhibitor

Fosinopril (Monopril)

ACE inhibitor

Lisinopril (Prinivil, Zestril)

ACE inhibitor

Quinapril (Accupril)

ACE inhibitor

Ramipril (Altace)

ACE inhibitor

enalapril (Vasotec)

ACE inhibitor

These drugs end in pril

ACE inhibitors

54. The patient with congestive heart failure who is on a diuretic drug shows a weight loss of 6.6 lb. The nurse is aware that the patient has lost L of fluid.

ANS: 3 A liter of fluid equals 2.2 lb. A loss of 6.6 lb would mean the loss of 3 L of fluid. PTS: 1 DIF: Cognitive Level: Comprehension REF: Page 1564 OBJ: 9 TOP: Fluid loss KEY: Nursing Process Step: Assessment MSC: NCLEX: Physiological Integrity

57. Trace the impulse pattern of conduction in sequence through the heart. (Separate letters by a comma and space as follows: A, B, C, D) a. Atrial wall b. Atrial-ventricular (AV) node c. Purkinje fibers d. Sinoatrial (SA) node e. Bundle branches f. Bundle of His

ANS: B, A, D, F, E, C The conduction begins with the impulse from the SA node that travels down the atrial wall to the AV node, to the Bundle of His, to the bundle branches, and finally to the Purkinje fibers. PTS: 1 DIF: Cognitive Level: Analysis REF: Page 1535-1534 OBJ: 3 TOP: Conduction KEY: Nursing Process Step: N/A MSC: NCLEX: Physiological Integrity

58. Arrange in sequence the path of the blood through the coronary circulation. (Separate letters by a comma and space as follows: A, B, C, D) a. Right atrium b. Pulmonary artery c. Tricuspid valve d. Right ventricle e. Superior and inferior vena cava f. Pulmonary vein g. Left atrium h. Mitral valve i. Left ventricle j. Lungs

ANS: E, A, C, D, B, J, F, G, H, I The blood travels through the vena cava to the right atrium, through the tricuspid valve to the right ventricle, through the pulmonary artery to the lungs. The pulmonary veins deliver the blood to the left atrium, then through the mitral valve to the left ventricle and out the aorta to the body. PTS: 1 DIF: Cognitive Level: Analysis REF: Page 1535, Figure 47-4 OBJ: 5 TOP: Path of blood through heart KEY: Nursing Process Step: N/A MSC: NCLEX: Physiological Integrity

56. The process by which a heart is shocked from a persistent arrhythmia back into sinus rhythm is called a _.

ANS: cardioversion Cardioversion is the restoration of the heart's normal sinus rhythm with the delivery of synchronized electric shock. PTS: 1 DIF: Cognitive Level: Knowledge REF: Page 1540 OBJ: 10 TOP: Cardioversion KEY: Nursing Process Step: N/A MSC: NCLEX: Physiological Integrity

55. The pain that a person with arterial insufficiency feels on exertion, which is relieved by rest, is .

ANS: intermittent claudication Intermittent claudication is a pain caused by ischemia when a person with arterial insufficiency exerts to the point that the tissues have inadequate oxygen-rich blood. The pain is relieved by rest. PTS: 1 DIF: Cognitive Level: Knowledge REF: Page 1581 OBJ: 9 TOP: Intermittent claudication KEY: Nursing Process Step: Assessment MSC: NCLEX: Physiological Integrity

51. The cardiac marker rises 3 hours after a myocardial infarct and measures myocardial contractile protein.

ANS: troponin I Troponin I is a serum cardiac marker that rises 3 hours after an MI and can measure myocardial contractile tissue. Troponin I is not affected by skeletal muscle injury as is troponin T. PTS: 1 DIF: Cognitive Level: Comprehension REF: Page 1541 OBJ: 6 TOP: Troponin I KEY: Nursing Process Step: Assessment MSC: NCLEX: Physiological Integrity

34. The postsurgical patient has a painful and swollen right calf that appears to be larger than the calf of the left leg. What is the nurse assessing for when she flexes the patient's right leg and dorsiflexes the foot? a. Pain, which would be a positive Homans sign b. Muscular spasm, which would be a sign of hypocalcemia c. Rigidity, which would be a sign of ankylosis d. Crepitus, which would be a sign of a joint disorder

ANS: A A positive Homans sign for deep vein thrombosis (DVT) is a report of pain when the affected leg is flexed and the foot is dorsiflexed. PTS: 1 DIF: Cognitive Level: Application REF: Page 1595 OBJ: 21 TOP: DVT KEY: Nursing Process Step: Assessment MSC: NCLEX: Physiological Integrity

11. The nurse takes into consideration that age-related changes can affect the peripheral circulation because of: a. sclerosed blood vessels. b. hypotension. c. inactivity. d. poor nutrition.

ANS: A Aging causes sclerotic changes in the blood vessels that lead to decreased elasticity and narrowing of the vessel lumen. PTS: 1 DIF: Cognitive Level: Comprehension REF: Page 1542, Lifespan OBJ: 16 TOP: Endocarditis KEY: Nursing Process Step: Planning MSC: NCLEX: Physiological Integrity

7. When applying transdermal nitroglycerin patches, which instruction by the nurse is correct? a. "Rotate application sites with each dose." b. "Use only the chest area for application sites." c. "Temporarily remove the patch if you go swimming." d. "Apply the patch to the same site each time."

ANS: A Application sites for transdermal nitroglycerin patches need to be rotated. Apply the transdermal patch to any nonhairy area of the body; the old patch should first be removed. The patch may be worn while swimming, but if it does come off, it should be replaced after the old site is cleansed. DIF: COGNITIVE LEVEL: Applying (Application) REF: p. 380 TOP: NURSING PROCESS: Implementation MSC: NCLEX: Physiological Integrity: Pharmacological and Parenteral Therapies

31. The nurse is treating a patient who has had a pacemaker inserted for the correction of atrial fibrillation. Which diagnostic test is no longer available to the patient because of the implanted device? a. MRI b. CT scan c. Thallium scan d. PET

ANS: A Because of the large magnets in the MRI cabinet, the pacemaker may be reset to a fixed mode and interfere with the functioning of the pacemaker. PTS: 1 DIF: Cognitive Level: Application REF: Page 1551 OBJ: 10 TOP: Pacemaker KEY: Nursing Process Step: Planning MSC: NCLEX: Physiological Integrity

4. The nurse is preparing for a community education program on hypertension. Which of these parameters determine the regulation of arterial blood pressure? a. Cardiac output and systemic vascular resistance b. Heart rate and peripheral resistance c. Blood volume and renal blood flow d. Myocardial contractility and arteriolar constriction

ANS: A Blood pressure is determined by the product of cardiac output and systemic vascular resistance. The other options are incorrect. DIF: COGNITIVE LEVEL: Understanding (Comprehension) REF: p. 348 TOP: NURSING PROCESS: General MSC: NCLEX: Physiological Integrity: Pharmacological and Parenteral Therapies

23. The patient, age 26, is hospitalized with cardiomyopathy. While obtaining a nursing history from her, the nurse recognizes that the increased incidence of cardiomyopathy in young adults who have minimal risk factors for cardiovascular disease is related to which factor(s)? a. Cocaine use b. Viral infections c. Vitamin B1 deficiencies d. Pregnancy

ANS: A Cardiomyopathy caused by cocaine abuse is seen more frequently than ever before. Cocaine also causes high circulating levels of catecholamines, which may further damage myocardial cells, leading to ischemic or dilated cardiomyopathy. The cardiomyopathy produced is difficult to treat. Interventions deal mainly with the HF that ensues. PTS: 1 DIF: Cognitive Level: Analysis REF: Page 1578 OBJ: 14 TOP: Cardiomyopathy KEY: Nursing Process Step: Assessment MSC: NCLEX: Physiological Integrity

4. A 74-year-old professional golfer has chest pain that occurs toward the end of his golfing games. He says the pain usually goes away after 1 or 2 sublingual nitroglycerin tablets and rest. What type of angina is he experiencing? a. Classic b. Variant c. Unstable d. Prinzmetal's

ANS: A Classic, or chronic stable, angina is triggered by either exertion or stress and usually subsides within 15 minutes with either rest or drug therapy. DIF: COGNITIVE LEVEL: Understanding (Comprehension) REF: p. 369 TOP: NURSING PROCESS: Assessment MSC: NCLEX: Physiological Integrity: Physiological Adaptation

8. In assessing a patient before administration of a cardiac glycoside, the nurse knows that which lab result can increase the toxicity of the drug? a. Potassium level 2.8 mEq/L b. Potassium level 4.9 mEq/L c. Sodium level 140 mEq/L d. Calcium level 10 mg/dL

ANS: A Hypokalemia increases the chance of digitalis toxicity. The other levels listed are incorrect. DIF: COGNITIVE LEVEL: Analyzing (Analysis) REF: p. 391 TOP: NURSING PROCESS: Assessment MSC: NCLEX: Physiological Integrity: Pharmacological and Parenteral Therapies

22. A patient admitted to the emergency room with a possible myocardial infarction (MI) has reports back from the laboratory. Which laboratory report is specific for myocardial damage? a. CK-MB b. Elevated white count c. Elevated sedimentation rate d. Low level of sodium

ANS: A The CK-MB is elevated when there is infarcted myocardial muscle. The elevated white count, low sodium, and ESR are nonspecific. PTS: 1 DIF: Cognitive Level: Application REF: Page 1559 OBJ: 6 TOP: CK-MB KEY: Nursing Process Step: Assessment MSC: NCLEX: Physiological Integrity

5. A patient arrives in the emergency department with severe chest pain. The patient reports that the pain has been occurring off and on for a week now. Which assessment finding would indicate the need for cautious use of nitrates and nitrites? a. Blood pressure of 88/62 mm Hg b. Apical pulse rate of 110 beats/min c. History of renal disease d. History of a myocardial infarction 2 years ago

ANS: A Hypotension is a possible contraindication to the use of nitrates because the medications may cause the blood pressure to decrease. The other options are incorrect. DIF: COGNITIVE LEVEL: Analyzing (Analysis) REF: p. 370 TOP: NURSING PROCESS: Assessment MSC: NCLEX: Physiological Integrity: Reduction of Risk Potential

27. During a health interview by the home health nurse, which patient complaint suggests left-sided heart failure? a. "I have to sleep in my recliner and I have this hacking cough." b. "I have no appetite and I have lost 3 lb in the last week." c. "I have to urinate every 2 hours, even during the night." d. "I go barefoot most of the time because my feet are so hot."

ANS: A Left ventricular failure; the first is signs and symptoms of decreased cardiac output. The second is pulmonary congestion. Signs and symptoms of this condition include dyspnea, orthopnea, pulmonary crackles, hemoptysis, and cough. PTS: 1 DIF: Cognitive Level: Analysis REF: Page 1565, Box 47-3 OBJ: 9 TOP: Heart failure KEY: Nursing Process Step: Planning MSC: NCLEX: Physiological Integrity

25. The nurse caring for a patient recovering from a myocardial infarction (MI) teaches which method to avoid the Valsalva maneuver during a bowel movement? a. Mouth breathing b. Pursing the lips and whistling c. Taking a deep breath and holding it d. Breathing rapidly through the nose

ANS: A Mouth breathing will lessen the severity of straining and will decrease the effect of the Valsalva maneuver on intrathoracic pressure. PTS: 1 DIF: Cognitive Level: Application REF: Page 1562 OBJ: 9 TOP: MI KEY: Nursing Process Step: Implementation MSC: NCLEX: Physiological Integrity

11. When administering digoxin immune Fab (Digibind) to a patient with severe digoxin toxicity, the nurse knows that each vial can bind with how much digoxin? a. 0.5 mg b. 5 mg c. 5.5 mg d. 15 mg

ANS: A One vial of digoxin immune Fab binds 0.5 mg of digoxin. The other options are incorrect. DIF: COGNITIVE LEVEL: Remembering (Knowledge) REF: p. 390 TOP: NURSING PROCESS: Assessment MSC: NCLEX: Physiological Integrity: Pharmacological and Parenteral Therapies

19. What is the major cause of cardiac valve disease? a. Rheumatic fever b. Long history of malnutrition c. Drug abuse d. Obesity

ANS: A Rheumatic fever, a streptococcal infection, is the major cause of cardiac valve disease. PTS: 1 DIF: Cognitive Level: Comprehension REF: Page 1572 OBJ: 10 TOP: Valvular disease KEY: Nursing Process Step: Implementation MSC: NCLEX: Physiological Integrity

30. What is the difference between primary and secondary hypertension? a. Secondary hypertension is caused by another disorder like renal disease. b. Secondary hypertension is related to hereditary factors. c. Secondary hypertension cannot be treated effectively. d. Secondary hypertension is no real threat to health.

ANS: A Secondary hypertension is a consistently elevated blood pressure that is caused by another disorder, such as renal disease, diabetes, or Cushing syndrome. PTS: 1 DIF: Cognitive Level: Analysis REF: Page 1584 OBJ: 18 TOP: Secondary hypertension KEY: Nursing Process Step: Planning MSC: NCLEX: Physiological Integrity

2. A patient is taking digoxin (Lanoxin) and a loop diuretic daily. When the nurse enters the room with the morning medications, the patient states, "I am seeing a funny yellow color around the lights." What is the nurse's next action? a. Assess the patient for symptoms of digoxin toxicity. b. Withhold the next dose of the diuretic. c. Administer the digoxin and diuretic together as ordered. d. Document this finding, and reassess in 1 hour.

ANS: A Seeing colors around lights is one potential indication of developing digoxin toxicity. If a patient complains of this, the nurse needs to assess for other signs and symptoms of digoxin toxicity including bradycardia, headache, dizziness, confusion, nausea, and blurred vision, and then notify the prescriber. Administering the drug or withholding the diuretic are incorrect options. DIF: COGNITIVE LEVEL: Analyzing (Analysis) REF: p. 392 TOP: NURSING PROCESS: Assessment MSC: NCLEX: Physiological Integrity: Reduction of Risk Potential

5. When counseling a male patient about the possible adverse effects of antihypertensive drugs, the nurse will discuss which potential problem? a. Impotence b. Bradycardia c. Increased libido d. Weight gain

ANS: A Sexual dysfunction is a common complication of antihypertensive medications and may be manifested in men as decreased libido or impotence. The other options are incorrect. DIF: COGNITIVE LEVEL: Understanding (Comprehension) REF: p. 366 TOP: NURSING PROCESS: Assessment MSC: NCLEX: Physiological Integrity: Pharmacological and Parenteral Therapies

8. A patient's blood pressure elevates to 270/150 mm Hg, and a hypertensive emergency is obvious. He is transferred to the intensive care unit and started on a sodium nitroprusside (Nipride) drip to be titrated per his response. With this medication, the nurse knows that the maximum dose of this drug should be infused for how long? a. 10 minutes b. 30 minutes c. 1 hour d. 24 hours

ANS: A Sodium nitroprusside is a potent vasodilator and may lead to extreme decreases in the patient's blood pressure. For this reason, it is never infused at the maximum dose for more than 10 minutes. If this drug does not control a patient's blood pressure after 10 minutes, it will most likely be ordered to be discontinued. The other times listed are incorrect. DIF: COGNITIVE LEVEL: Understanding (Comprehension) REF: p. 363 TOP: NURSING PROCESS: Planning MSC: NCLEX: Physiological Integrity: Pharmacological and Parenteral Therapies

6. The nurse assesses that the home health patient has no signs or symptoms of heart failure, but does have a history of rheumatic fever and has been recently diagnosed with diabetes mellitus. The nurse is aware that using the American College of Cardiology and the American Heart Association (ACC/AHA) staging, this patient would be a: a. stage A. b. stage B. c. stage C. d. stage D.

ANS: A The ACC/AHA staging describes stage A as a person without symptoms of heart failure, but with primary conditions associated with the development of the disease. PTS: 1 DIF: Cognitive Level: Analysis REF: Page 1565, Box 47-3 OBJ: 9 TOP: Heart failure KEY: Nursing Process Step: Assessment MSC: NCLEX: Physiological Integrity

27) Colloid-

Albumin, Dextrose, Starch (Large things that pull fluid in)

4. The nurse identifies the "LUBB" sound of the "LUBB/DUBB" of the cardiac cycle as the sound of the: a. AV valves closing. b. closure of the semilunar valves. c. contraction of the papillary muscles. d. contraction of the ventricles.

ANS: A The LUBB is the first sound of a low pitch heard when the AV valves close. PTS: 1 DIF: Cognitive Level: Application REF: Page 1535 OBJ: 4 TOP: Lubb sound KEY: Nursing Process Step: Assessment MSC: NCLEX: Physiological Integrity

33. What is the patient goal of the walking exercise program designed for the rehabilitation of a post-MI patient? a. Walk 2 miles in less than 60 minutes after 12 weeks. b. Jog mile in less than 30 minutes after 12 weeks. c. "Fast walk" 1 mile in less than 20 minutes after 12 weeks. d. Walk 1 mile in 15 minutes without dyspnea after 12 weeks.

ANS: A The goal of the 12-week walking program is that the patient can walk 2 miles in less than 60 minutes. PTS: 1 DIF: Cognitive Level: Knowledge REF: Page 1563, Home Care OBJ: 11 TOP: Cardiac rehab KEY: Nursing Process Step: Planning MSC: NCLEX: Health Promotion and Maintenance

21. A patient, age 72, was admitted to the medical unit with a diagnosis of angina pectoris. Characteristic signs and symptoms of angina pectoris include: a. substernal pain that radiates down the left arm. b. epigastric pain that radiates to the jaw. c. indigestion, nausea, and eructation. d. fatigue, shortness of breath, and dyspnea.

ANS: A The pain often radiates down the left inner arm to the little finger and also upward to the shoulder and jaw. PTS: 1 DIF: Cognitive Level: Comprehension REF: Page 1553, figure 47-1 OBJ: 9 TOP: Angina pectoris KEY: Nursing Process Step: Assessment MSC: NCLEX: Physiological Integrity

20. The patient has a total cholesterol of 190 with a high-density lipid (HDL) of 110 and a low-density lipid (LDL) of 80. The nurse's reaction is one of: a. satisfaction. This is good cholesterol control. b. determination. This is evidence that more instruction is necessary. c. inquiry. This needs to clarified as to the cause of noncompliance with the drug protocol. d. regret. This shows very poor cholesterol control.

ANS: A Total cholesterol of less than 200 is desirable. The higher the number of HDLs the better. A high number of LDLs puts the patient at risk for heart disease. PTS: 1 DIF: Cognitive Level: Analysis REF: Page 1541, Box 47-1 OBJ: 6 TOP: Lipid studies KEY: Nursing Process Step: Planning MSC: NCLEX: Physiological Integrity

2. Which drug classes are considered first-line treatment for heart failure? (Select all that apply.) a. Angiotensin-converting enzyme (ACE) inhibitors b. Angiotensin II receptor blockers (ARBs) c. Digoxin (cardiac glycoside) d. Beta blockers e. Nesiritide (Natrecor), the B-type natriuretic peptide

ANS: A, B, D ACE inhibitors, ARBs, and beta blockers are now considered the first-line treatments for heart failure. Digoxin is used when the first-line treatments are not successful; nesiritide is considered a last-resort treatment. DIF: COGNITIVE LEVEL: Applying (Application) REF: p. 384 TOP: NURSING PROCESS: Planning MSC: NCLEX: Physiological Integrity: Pharmacological and Parenteral Therapies

1. A patient will be taking dabigatran (Pradaxa) as part of treatment for chronic atrial fibrillation. Which statements about dabigatran are true? (Select all that apply.) a. The dose of dabigatran is reduced in patients with decreased renal function. b. Bleeding is the most common adverse effect. c. Potassium chloride is given as an antidote in cases of overdose. d. Dabigatran levels are monitored by measuring prothrombin time/international normalized ratio (PT/INR) results. e. This drug is a prodrug and becomes activated in the liver.

ANS: A, B, E Dabigatran is excreted extensively in the kidneys, and the dose is dependent upon renal function. The normal dose is 150 mg twice daily, but it must be reduced to 75 mg twice daily if creatinine clearance is less than 30 mL/min. The most common and serious side effect is bleeding. Dabigatran is a prodrug that becomes activated in the liver. There is no antidote to dabigatran. The other options are incorrect. DIF: COGNITIVE LEVEL: Applying (Application) REF: p. 427 TOP: NURSING PROCESS: Implementation MSC: NCLEX: Physiological Integrity: Pharmacological and Parenteral Therapies

49. The nurse encourages the patient who is recovering from a myocardial infarct (MI) to ask the health care provider to prescribe a cardiac rehabilitation series in order to learn to (select all that apply): a. improve stamina. b. strengthen muscles. c. plan an appropriate diet. d. select herbal remedies. e. reduce risk of further problems. f. understand heart condition.

ANS: A, B, E, F Cardiac rehabilitation offers exercise programs to increase strength and increase stamina. Educational opportunities are offered on reduction of risk and understanding the disease process. PTS: 1 DIF: Cognitive Level: Application REF: Page 1563 OBJ: 11 TOP: Cardiac rehab KEY: Nursing Process Step: Implementation MSC: NCLEX: Health Promotion and Maintenance

50. Following an angiogram with the insertion site of the left groin, the nurse will include in the plan of care provisions for (select all that apply): a. checking pedal pulses. b. ambulating with assistance 2 hours after recovery. c. checking color and warmth of left leg frequently. d. sandbagging over insertion site. e. placing patient in semi-Fowler position.

ANS: A, C, D The pulses below the insertion site are checked to ensure patency of the vessels; the color and warmth of the left extremity is checked to ensure adequate circulation. A sandbag or other pressure device is placed over the insertion site. The patient is maintained in a supine position for several hours postprocedure. PTS: 1 DIF: Cognitive Level: Application REF: Page 1537 OBJ: 6 TOP: Angiogram KEY: Nursing Process Step: Implementation MSC: NCLEX: Physiological Integrity

45. The nurse points out which of the following as modifiable risks for coronary artery disease (CAD)? (Select all that apply.) a. Diabetes mellitus b. Heredity c. Smoking d. Hypertension e. Hyperlipidemia f. Age

ANS: A, C, D, E Modifiable risks for the development of CAD include smoking, hyperlipidemia, hypertension, diabetes mellitus, obesity, sedentary lifestyle, and stress. PTS: 1 DIF: Cognitive Level: Analysis REF: Page 1544-1545 OBJ: 7 TOP: Modifiable risks for CAD KEY: Nursing Process Step: Implementation MSC: NCLEX: Physiological Integrity

46. The nurse outlines which of the following as conditions that would disqualify a candidate for a heart transplant? (Select all that apply.) a. Recent malignancy b. Dilated cardiomyopathy c. Peptic ulcer disease d. Diabetes type 2 e. Severe obesity f. Inoperable coronary artery disease

ANS: A, C, E Contraindications for candidacy for cardiac transplant include recent malignancy, active peptic ulcer disease, severe obesity, diabetes type 1 with end-organ damage. Dilated cardiomyopathy and inoperable coronary artery disease are indications for transplant. PTS: 1 DIF: Cognitive Level: Application REF: Page 1579, Box 47-7 OBJ: 15 TOP: Contraindications for cardiac transplant KEY: Nursing Process Step: Implementation MSC: NCLEX: Physiological Integrity

43. What is the transesophageal echocardiogram (TEE) used for? (Select all that apply.) a. Detect thrombi before a cardioversion b. Check for cardiac arrhythmias c. Visualize vegetation on the heart valves d. Measure effectiveness of diuretic therapy e. Visualize abscesses on the heart valves

ANS: A, C, E The TEE is used to check for thrombi before cardioversion, and to visualize vegetation and abscesses on the valves of the heart. PTS: 1 DIF: Cognitive Level: Knowledge REF: Page 1592 OBJ: 16 TOP: TEE KEY: Nursing Process Step: N/A MSC: NCLEX: Physiological Integrity

1. The nurse is providing education about the use of sublingual nitroglycerin tablets. She asks the patient, "What would you do if you experienced chest pain while mowing your yard? You have your bottle of sublingual nitroglycerin with you." Which actions by the patient are appropriate in this situation? (Select all that apply.) a. Stop the activity, and lie down or sit down. b. Call 911 immediately. c. Call 911 if the pain is not relieved after taking 1 sublingual tablet. d. Call 911 if the pain is not relieved after taking 3 sublingual tablets in 15 minutes. e. Place a tablet under the tongue. f. Place a tablet in the space between the gum and cheek. g. Take another sublingual tablet if chest pain is not relieved after 5 minutes, up to three total.

ANS: A, C, E, G With sublingual forms, the medication is taken at the first sign of chest pain, not delayed until the pain is severe. The patient needs to sit down or lie down and take one sublingual tablet. According to current guidelines, if the chest pain or discomfort is not relieved in 5 minutes, after one dose, the patient (or family member) must call 911 immediately. The patient can take one more tablet while awaiting emergency care and may take a third tablet 5 minutes later, but no more than a total of three tablets. These guidelines reflect the fact that angina pain that does not respond to nitroglycerin may indicate a myocardial infarction. The sublingual dose is placed under the tongue, and the patient needs to avoid swallowing until the tablet has dissolved. Placing a tablet between the gum and cheek is the buccal route. DIF: COGNITIVE LEVEL: Analyzing (Analysis) REF: pp. 379-380 TOP: NURSING PROCESS: Implementation MSC: NCLEX: Physiological Integrity: Pharmacological and Parenteral Therapies

1. When teaching a patient about antihypertensive drug therapy, which statements by the nurse are correct? (Select all that apply.) a. "You need to have your blood pressure checked once a week and keep track of the readings." b. "If you notice that the symptoms have gone away, you should be able to stop taking the drug." c. "An exercise program may be helpful in treating hypertension, but let's check with your doctor first." d. "If you experience severe side effects, stop the medicine and let us know at your next office visit." e. "Most over-the-counter decongestants are compatible with antihypertensive drugs." f. "Please continue taking the medication, even if you are feeling better."

ANS: A, C, F Keeping a record of weekly blood pressure checks helps to monitor the effectiveness of the therapy. Remind the patient not to stop taking the medication just because he or she is feeling better. Abruptly stopping the medication may lead to rebound hypertension. Therapy is often life-long, even though symptoms may improve. Many over-the-counter drugs, especially decongestants, have serious interactions with antihypertensive drugs. The patient needs to consult his or her prescriber before taking any other medication. DIF: COGNITIVE LEVEL: Analyzing (Analysis) REF: pp. 365-366 TOP: NURSING PROCESS: Assessment MSC: NCLEX: Physiological Integrity: Pharmacological and Parenteral Therapies

5. A patient is admitted from the emergency department. The emergency department physician notes the patient has a diagnosis of heart failure with a New York Heart Association (NYHA) classification of IV. This indicates the patient's condition as: a. moderate heart failure. b. severe heart failure. c. congestive heart failure. d. negligible heart failure.

ANS: B Class IV: Severe; patient unable to perform any physical activity without discomfort. Angina or symptoms of cardiac inefficiency may develop at rest. PTS: 1 DIF: Cognitive Level: Knowledge REF: Page 1565, Box 47-3 OBJ: 9 TOP: Classification of heart failure KEY: Nursing Process Step: Assessment MSC: NCLEX: Physiological Integrity

10. The nurse notes in a patient's medical record that nesiritide (Natrecor) has been ordered. Based on this order, the nurse interprets that the patient has which disorder? a. Atrial fibrillation b. Acutely decompensated heart failure with dyspnea at rest c. Systolic heart failure d. Long-term treatment of heart failure

ANS: B Nesiritide is indicated for the treatment of acutely decompensated heart failure with dyspnea at rest. Digoxin is used for the treatment of atrial fibrillation and systolic heart failure. Long-term treatment of heart failure is not an indication for nesiritide. DIF: COGNITIVE LEVEL: Understanding (Comprehension) REF: p. 386 TOP: NURSING PROCESS: Assessment MSC: NCLEX: Physiological Integrity: Pharmacological and Parenteral Therapies

8. The nurse assesses pitting edema that can be depressed approximately inch and refills in 15 seconds. The nurse would document this assessment as: a. +1 edema. b. +2 edema. c. +3 edema. d. +4 edema.

ANS: B A +2 edema can be documented if the skin can be depressed inch and respond within 15 seconds. PTS: 1 DIF: Cognitive Level: Analysis REF: Page 1565, Table 47-5 OBJ: 9 TOP: Pitting edema KEY: Nursing Process Step: Assessment MSC: NCLEX: Physiological Integrity

3. While assessing a patient who is receiving intravenous digitalis, the nurse recognizes that the drug has a negative chronotropic effect. How would this drug effect be evident in the patient? a. Decreased blood pressure b. Decreased heart rate c. Decreased conduction d. Decreased ectopic beats

ANS: B A negative chronotropic effect results in a reduced heart rate; this is one effect of cardiac glycosides. The other options are incorrect. DIF: COGNITIVE LEVEL: Applying (Application) REF: p. 384 TOP: NURSING PROCESS: Evaluation MSC: NCLEX: Physiological Integrity: Pharmacological and Parenteral Therapies

12. A patient has had recent prosthetic heart valve surgery and is receiving anticoagulant therapy. While monitoring the patient's laboratory work, the nurse interprets that the patient's international normalized ratio (INR) level of 2.5 indicates that a. the patient is not receiving enough warfarin to have a therapeutic effect. b. the patient's warfarin dose is at therapeutic levels. c. the patient's intravenous heparin dose is dangerously high. d. the patient's intravenous heparin dose is at therapeutic levels.

ANS: B A normal INR (without warfarin) is 1.0, whereas a therapeutic INR (with warfarin) ranges from 2 to 3.5, depending on the indication for use of the drug (e.g., atrial fibrillation, thromboprevention, prosthetic heart valve). DIF: COGNITIVE LEVEL: Applying (Application) REF: p. 424 TOP: NURSING PROCESS: Implementation MSC: NCLEX: Physiological Integrity: Pharmacological and Parenteral Therapies

Aneurysm (1593)

Bulging of an artery, like a tire with a bulge

10. A patient who has been taking antihypertensive drugs for a few months complains of having a persistent dry cough. The nurse knows that this cough is an adverse effect of which class of antihypertensive drugs? a. Beta blockers b. Angiotensin-converting enzyme (ACE) inhibitors c. Angiotensin II receptor blockers (ARBs) d. Calcium channel blockers

ANS: B ACE inhibitors cause a characteristic dry, nonproductive cough that reverses when therapy is stopped. The other drug classes do not cause this cough. DIF: COGNITIVE LEVEL: Understanding (Comprehension) REF: p. 355 TOP: NURSING PROCESS: Assessment MSC: NCLEX: Physiological Integrity: Pharmacological and Parenteral Therapies

13. A patient with type 2 diabetes mellitus has been found to have trace proteinuria. The prescriber writes an order for an angiotensin-converting enzyme (ACE) inhibitor. What is the main reason for prescribing this class of drug for this patient? a. Cardioprotective effects b. Renal protective effects c. Reduces blood pressure d. Promotes fluid output

ANS: B ACE inhibitors have been shown to have a protective effect on the kidneys because they reduce glomerular filtration pressure. This is one reason that they are among the cardiovascular drugs of choice for diabetic patients. The other drugs do not have this effect. DIF: COGNITIVE LEVEL: Applying (Application) REF: pp. 354-355 TOP: NURSING PROCESS: Assessment MSC: NCLEX: Physiological Integrity: Pharmacological and Parenteral Therapies

17. The elderly patient with angina pectoris says she is unsure how she should take nitroglycerin when she has an attack. The nurse's most helpful response would be: a. "Continue to take nitroglycerin sublingually at 5-minute intervals until the pain is relieved." b. "If the pain is not relieved after three doses of nitroglycerin at 5-minute intervals, call your physician and come to the hospital." c. "When nitroglycerin is not relieving the pain, lie down and rest." d. "Use oxygen at home to relieve pain when nitroglycerin is not successful."

ANS: B Administer prescribed nitroglycerin. Repeat every 5 minutes, three times. If pain is unrelieved, notify the physician. Nitroglycerin administered sublingually usually relieves angina symptoms but does not relieve the pain from an MI. Administering nitroglycerin more than three times will probably not relieve the pain. PTS: 1 DIF: Cognitive Level: Application REF: Page 1545 OBJ: 9 TOP: Angina pectoris KEY: Nursing Process Step: Implementation MSC: NCLEX: Physiological Integrity

37. Which statement would lead the nurse to offer more instruction about taking warfarin (Coumadin)? a. "I eat a banana every morning with breakfast." b. "I try to eat more green leafy vegetables, especially broccoli, spinach, and kale." c. "I try to eat a well-balanced, low-fat diet." d. "I don't drink alcohol or caffeine."

ANS: B Avoid marked changes in eating habits, such as dramatically increasing foods high in vitamin K (e.g., broccoli, spinach, kale, greens). Limit alcohol intake to small amounts. PTS: 1 DIF: Cognitive Level: Comprehension REF: Page 1597 OBJ: 10 TOP: Warfarin KEY: Nursing Process Step: Implementation MSC: NCLEX: Physiological Integrity

10. A patient is being discharged on anticoagulant therapy. The nurse will include in the patient-education conversation that it is important to avoid herbal products that contain which substance? a. Valerian b. Ginkgo c. Soy d. Saw palmetto

ANS: B Capsicum pepper, feverfew, garlic, ginger, ginkgo, St. John's wort, and ginseng are some herbals that have potential interactions with anticoagulants, especially with warfarin. DIF: COGNITIVE LEVEL: Understanding (Comprehension) REF: p. 425 TOP: NURSING PROCESS: Assessment MSC: NCLEX: Physiological Integrity: Reduction of Risk Potential

3. The nurse is aware that the symptoms of an impending myocardial infarction (MI) differ in women because acute chest pain is not present. Women are frequently misdiagnosed as having: a. hepatitis A. b. indigestion. c. urinary infection. d. menopausal complications.

ANS: B Indigestion, gallbladder attack, anxiety attack, and depression are frequent misdiagnoses for women having an MI. PTS: 1 DIF: Cognitive Level: Application REF: Page 1559 OBJ: 16 TOP: MIs in women KEY: Nursing Process Step: Planning MSC: NCLEX: Physiological Integrity

8. A patient is receiving heparin therapy as part of the treatment for a pulmonary embolism. The nurse monitors the results of which laboratory test to check the drug's effectiveness? a. Bleeding times b. Activated partial thromboplastin time (aPTT) c. Prothrombin time/international normalized ratio (PT/INR) d. Vitamin K levels

ANS: B Ongoing aPTT values are used to monitor heparin therapy. PT/INR is used to monitor warfarin therapy. The other two options are not used to monitor anticoagulant therapy. DIF: COGNITIVE LEVEL: Understanding (Comprehension) REF: p. 437 TOP: NURSING PROCESS: Evaluation MSC: NCLEX: Physiological Integrity: Pharmacological and Parenteral Therapies

28. The home health nurse caring for a patient with infective endocarditis overhears the patient making a dental appointment for an extraction next month. Which question is most important for the nurse to ask? a. "Do you have a toothache?" b. "Have you contacted your physician about your dental appointment?" c. "Is your dentist board certified?" d. "Do you think you should wait that long for your tooth extraction?'

ANS: B Patients with endocarditis are put on a protocol of prophylactic antibiotics for any invasive procedure. The dentist and physician should be contacted before the extraction. PTS: 1 DIF: Cognitive Level: Application REF: Page 1574 OBJ: 13 TOP: Endocarditis KEY: Nursing Process Step: Implementation MSC: NCLEX: Health Promotion and Maintenance

12. The nurse is reviewing the JNC 7 guidelines for treatment of hypertension. Which blood pressure would be classified as "prehypertension" according to the JNC 7 guidelines? a. 118/76 mm Hg b. 130/88 mm Hg c. 150/90 mm Hg d. 160/104 mm Hg

ANS: B Prehypertension is defined as having a systolic blood pressure between 120 and 139 mm Hg and a diastolic pressure between 80 and 89 mm Hg. The other parameters listed are incorrect. DIF: COGNITIVE LEVEL: Applying (Application) REF: p. 360 TOP: NURSING PROCESS: Assessment MSC: NCLEX: Physiological Integrity: Physiological Adaptation

7. The nurse caring for a patient recovering from a myocardial infarct who is on remote telemetry recognizes the need for added instruction when the patient says: a. "I can ambulate in the hallway with this gadget on." b. "I always take off the telemetry device when I shower." c. "My EKG is being watched by one of the nurses in CCU on the home unit." d. "I am able to sleep just fine with this device on."

ANS: B Remote telemetry allows the patient to be on a separate unit, but be monitored in a central location. The patients can be ambulatory and can sleep with the monitor on. They should not remove the monitor to shower. PTS: 1 DIF: Cognitive Level: Application REF: Page 1539 OBJ: 6 TOP: Remote telemetry KEY: Nursing Process Step: Evaluation MSC: NCLEX: Physiological Integrity

7. The nurse is creating a plan of care for a patient with a new diagnosis of hypertension. Which is a potential nursing diagnosis for the patient taking antihypertensive medications? a. Diarrhea b. Sexual dysfunction c. Urge urinary incontinence d. Impaired memory

ANS: B Sexual dysfunction is a potential nursing diagnosis related to possible adverse effects of antihypertensive drug therapy. The other nursing diagnoses are not appropriate. DIF: COGNITIVE LEVEL: Applying (Application) REF: p. 361 TOP: NURSING PROCESS: Nursing Diagnosis MSC: NCLEX: Physiological Integrity: Pharmacological and Parenteral Therapies

32. Which assessment would lead the nurse to examine the leg closely for evidence of a stasis ulcer? a. Cool dry lower limb b. Edematous, red scaly skin on medial surface of the leg c. Lack of hair and shiny appearance of the lower leg d. Lack of a pedal pulse

ANS: B Suggestion of a stasis ulcer in the making is an edematous, dry scaly area on the medial surface of the lower leg that has a darker pigmentation (rubor). Cool hairless limbs with absent or weak pedal pulses are indicative of arterial insufficiency. PTS: 1 DIF: Cognitive Level: Application REF: Page 1582 OBJ: 21 TOP: Medications KEY: Nursing Process Step: Assessment MSC: NCLEX: Physiological Integrity

15. The nurse reminds the patient who is on Coumadin for the treatment of atrial fibrillation that the ideal is to maintain the international normalized ratio (INR) at between: a. 1 and 2. b. 2 and 3. c. 3 and 4. d. 4 and 5.

ANS: B The desired INR for the monitoring of anticoagulant therapy is between 2 and 3. PTS: 1 DIF: Cognitive Level: Knowledge REF: Page 1546 OBJ: 8 TOP: INR KEY: Nursing Process Step: Implementation MSC: NCLEX: Physiological Integrity

6. A patient has a digoxin level of 1.4 ng/mL. The nurse interprets that this level is a. below the therapeutic level. b. within the therapeutic range. c. above the therapeutic level. d. at a toxic level.

ANS: B The normal therapeutic drug level of digoxin is between 0.5 and 2 ng/mL. The other options are incorrect. DIF: COGNITIVE LEVEL: Analyzing (Analysis) REF: p. 388 TOP: NURSING PROCESS: Evaluation MSC: NCLEX: Physiological Integrity: Pharmacological and Parenteral Therapies

4. A patient who has been anticoagulated with warfarin (Coumadin) has been admitted for gastrointestinal bleeding. The history and physical examination indicates that the patient may have taken too much warfarin. The nurse anticipates that the patient will receive which antidote? a. vitamin E b. vitamin K c. protamine sulfate d. potassium chloride

ANS: B Vitamin K is given to reverse the anticoagulation effects of warfarin toxicity. Protamine sulfate is the antidote for heparin overdose. The other options are incorrect. DIF: COGNITIVE LEVEL: Understanding (Comprehension) REF: p. 424 TOP: NURSING PROCESS: Implementation MSC: NCLEX: Physiological Integrity: Pharmacological and Parenteral Therapies

42. Which information should be taught to patients starting on anticoagulant therapy for a valvular disorder? (Select all that apply.) a. Increase the dose of aspirin for better therapy. b. Take medication at the same time each day. c. Report to physician cuts that do not stop bleeding with direct pressure. d. No restrictions for food or drink. e. Report for prescribed blood tests (PTT, INR, CBC, blood sugar).

ANS: B, C Aspirin should not be used with anticoagulant therapy because it will increase bleeding. Gums, nosebleeds, excessive bruising, and cuts that do not stop bleeding with direct pressure should be reported to the physician. Alcohol and dark green and yellow vegetables should be avoided because they contain vitamin K. Normal blood tests for anticoagulant therapy are PTT, INR, and PT. PTS: 1 DIF: Cognitive Level: Analysis REF: Page 1591, Nursing Care Plan OBJ: 10 TOP: Anticoagulant therapy KEY: Nursing Process Step: Planning MSC: NCLEX: Physiological Integrity

41. The nurse would design teaching for a patient with Raynaud disease to include which of the following? (Select all that apply.) a. Warming hands and feet with a heating pad b. Using mittens in cold weather c. Practicing stress-reducing techniques d. Complete smoking cessation e. Using caution when cleaning the refrigerator or freezer

ANS: B, C, D, E Nursing interventions include patient teaching in techniques for stress reduction, avoiding exposure to cold, and techniques for smoking cessation. PTS: 1 DIF: Cognitive Level: Analysis REF: Page 1595, Nursing Care Plan OBJ: 20 TOP: Raynaud disease KEY: Nursing Process Step: Planning MSC: NCLEX: Physiological Integrity

47. When assessing a patient with a possible MI, what should the nurse assess for? (Select all that apply.) a. Pain radiating to left arm and jaw b. Hypertension c. Pallor d. Diaphoresis e. Erratic behavior f. Cardiac rhythm changes

ANS: B, C, D, E, F Hypertension, vomiting, diaphoresis, hypotension, pallor, and cardiac rhythm changes are objective data seen in patients with an MI. PTS: 1 DIF: Cognitive Level: Analysis REF: Page 1558, Table 47-2 OBJ: 10 TOP: Myocardial infarction KEY: Nursing Process Step: Assessment MSC: NCLEX: Health Promotion and Maintenance

48. Which of the following are signs of digoxin (Lanoxin) toxicity? (Select all that apply.) a. Ringing in the ears b. Bradycardia c. Headache d. Visual disturbance e. Hematuria f. Gastrointestinal complaints

ANS: B, C, D, F Major signs of digoxin toxicity are nausea, bradycardia (HR <60), headache, and visual disturbances, as well as fatigue and arrhythmias. PTS: 1 DIF: Cognitive Level: Application REF: Page 1548, Table 47-1 OBJ: 10 TOP: Digitoxin toxicity KEY: Nursing Process Step: Assessment MSC: NCLEX: Physiological Integrity

44. Which patient teaching would help to prevent venous stasis? (Select all that apply.) a. Dangle legs when sitting b. Avoid crossing legs at the knee c. Elevate legs when lying in bed or sitting d. Massage extremities to help maintain blood flow e. Wear elastic stockings when ambulating

ANS: B, C, E Avoid prolonged sitting or standing. Avoid crossing the legs at the knee. Elevate legs when sitting. Wear elastic stockings when ambulatory. Do not massage extremities because of danger of embolization of clots (thrombus breaking off and becoming an embolus). PTS: 1 DIF: Cognitive Level: Analysis REF: Page 1597 OBJ: 16 TOP: Thrombophlebitis KEY: Nursing Process Step: Planning MSC: NCLEX: Physiological Integrity

40. The nurse would assess closely for signs of right-sided heart failure which include (select all that apply): a. cough. b. increasing abdominal girth. c. shortness of breath. d. edema of feet and ankles. e. distended jugular veins. f. orthopnea.

ANS: B, D, E Indicators of right-sided heart failure are distended jugular veins, anorexia, abdominal distention from ascites, liver enlargement with right upper quadrant pain, and edema of feet and ankles. PTS: 1 DIF: Cognitive Level: Analysis REF: Page 1563, Box 47-4 OBJ: 9 TOP: Right-sided heart failure KEY: Nursing Process Step: Assessment MSC: NCLEX: Physiological Integrity

1. When a patient is experiencing digoxin toxicity, which clinical situation would necessitate the use of digoxin immune Fab (Digifab)? (Select all that apply.) a. The patient reports seeing colorful halos around lights. b. The patient's serum potassium level is above 5 mEq/L. c. The patient is experiencing nausea and anorexia. d. The patient is experiencing severe sinus bradycardia that does not respond to cardiac pacing. e. The patient has received an overdose of greater than 10 mg of digoxin. f. The patient reports fatigue and headaches.

ANS: B, D, E Clinical situations that would require the use of digoxin immune Fab in a patient with digoxin toxicity include serum potassium level above 5 mEq/L, severe sinus bradycardia that does not respond to cardiac pacing, or an overdose of more than 10 mg of digoxin. Seeing colorful halos around lights and experiencing nausea, anorexia, fatigue, and headaches are potential adverse effects of digoxin therapy but are not necessarily reasons for digoxin immune Fab treatment. DIF: COGNITIVE LEVEL: Analyzing (Analysis) REF: p. 389 TOP: NURSING PROCESS: Assessment MSC: NCLEX: Physiological Integrity: Pharmacological and Parenteral Therapies

2. A patient with a history of angina will be started on ranolazine (Ranexa). The nurse is reviewing the patient's history and will note potential contraindications to this drug therapy if which condition is present? (Select all that apply.) a. Type 2 diabetes mellitus b. Prolonged QT interval on the electrocardiogram c. Heart failure d. Closed-angle glaucoma e. Decreased liver function

ANS: B, E Ranolazine is contraindicated in patients with preexisting QT prolongation or hepatic impairment. The other options are not contraindications. DIF: COGNITIVE LEVEL: Applying (Application) REF: p. 376 TOP: NURSING PROCESS: Assessment MSC: NCLEX: Physiological Integrity: Pharmacological and Parenteral Therapies

3. A patient has been diagnosed with angina and will be given a prescription for sublingual nitroglycerin tablets. When teaching the patient how to use sublingual nitroglycerin, the nurse will include which instruction? a. Take up to five doses at 15-minute intervals for an angina attack. b. If the tablet does not dissolve quickly, chew the tablet for maximal effect. c. If the chest pain is not relieved after one tablet, call 911 immediately. d. Wait 1 minute between doses of sublingual tablets, up to three doses.

ANS: C According to current guidelines, if the chest pain or discomfort is not relieved in 5 minutes, after one dose, the patient (or family member) must call 911 immediately. The patient may take one more tablet while awaiting emergency care and may take a third tablet 5 minutes later, but no more than a total of three tablets. The sublingual dose is placed under the tongue, and the patient needs to avoid swallowing until the tablet has dissolved. DIF: COGNITIVE LEVEL: Applying (Application) REF: pp. 379-380 TOP: NURSING PROCESS: Implementation MSC: NCLEX: Safe and Effective Care Environment: Management of Care

6. A patient has been instructed to take one enteric-coated low-dose aspirin a day as part of therapy to prevent strokes. The nurse will provide which instruction when providing patient teaching about this medication? a. Aspirin needs to be taken on an empty stomach to ensure maximal absorption. b. Low-dose aspirin therapy rarely causes problems with bleeding. c. Take the medication with 6 to 8 ounces of water and food. d. Coated tablets may be crushed if necessary for easier swallowing.

ANS: C Enteric-coated aspirin is best taken with 6 to 8 ounces of water and with food to help decrease gastrointestinal upset. Enteric-coated tablets should not be crushed. Risk for bleeding increases with aspirin therapy, even at low doses. DIF: COGNITIVE LEVEL: Applying (Application) REF: p. 438 TOP: NURSING PROCESS: Implementation MSC: NCLEX: Physiological Integrity: Pharmacological and Parenteral Therapies

6. The nurse is reviewing drug therapy for hypertension. According to the JNC 7 guidelines, antihypertensive drug therapy for a newly diagnosed stage 1 hypertensive African-American patient would most likely include which drug or drug classes? a. Vasodilators alone b. ACE inhibitors alone c. Calcium channel blockers with thiazide diuretics d. Beta blockers with thiazide diuretics

ANS: C According to the JNC 7 guidelines, calcium channel blockers and diuretics have been shown to be more effective in African Americans than in white patients. Thiazide diuretics are also recommended for newly diagnosed stage 1 hypertension. DIF: COGNITIVE LEVEL: Applying (Application) REF: p. 361 TOP: NURSING PROCESS: Implementation MSC: NCLEX: Physiological Integrity: Pharmacological and Parenteral Therapies

11. A patient who is taking warfarin (Coumadin) therapy has a headache and calls the prescriber's office to ask about taking a pain reliever. The nurse expects to receive instructions for which type of medication? a. aspirin tablets b. ibuprofen (Advil) c. acetaminophen (Tylenol) d. An opioid

ANS: C Acetaminophen should be safe in regular doses; high doses, however, as well as other nonsteroidal antiinflammatory drugs and aspirin, may cause an increased anticoagulant effect. Taking an opioid for a headache may not be appropriate. DIF: COGNITIVE LEVEL: Analyzing (Analysis) REF: p. 425 TOP: NURSING PROCESS: Implementation MSC: NCLEX: Physiological Integrity: Pharmacological and Parenteral Therapies

10. While assessing a patient who is taking a beta blocker for angina, the nurse knows to monitor for which adverse effect? a. Nervousness b. Hypertension c. Bradycardia d. Dry cough

ANS: C Adverse effects of beta blockers include bradycardia, hypotension, dizziness, drowsiness, impotence, and several other effects, but not dry cough. DIF: COGNITIVE LEVEL: Understanding (Comprehension) REF: p. 373 TOP: NURSING PROCESS: Evaluation MSC: NCLEX: Physiological Integrity: Pharmacological and Parenteral Therapies

10. The nurse recognizes the echocardiogram report that shows an ejection factor of 42% as an indication of: a. normal heart action. b. mild heart failure. c. moderate heart failure. d. severe heart failure.

ANS: C An ejection factor (cardiac output) of 42% indicates moderate heart failure. PTS: 1 DIF: Cognitive Level: Comprehension REF: Page 1540 OBJ: 6 TOP: Heart failure KEY: Nursing Process Step: Assessment MSC: NCLEX: Physiological Integrity

9. The nurse notes in the patient's medication orders that the patient will be starting anticoagulant therapy. What is the primary goal of anticoagulant therapy? a. Stabilizing an existing thrombus b. Dissolving an existing thrombus c. Preventing thrombus formation d. Dilating the vessel around a clot

ANS: C Anticoagulants prevent thrombus formation but do not dissolve or stabilize an existing thrombus, nor do they dilate vessels around a clot. DIF: COGNITIVE LEVEL: Understanding (Comprehension) REF: p. 422 TOP: NURSING PROCESS: Implementation MSC: NCLEX: Physiological Integrity: Pharmacological and Parenteral Therapies

12. A patient is in the intensive care unit and receiving an infusion of milrinone (Primacor) for severe heart failure. The prescriber has written an order for an intravenous dose of furosemide (Lasix). How will the nurse give this drug? a. Infuse the drug into the same intravenous line as the milrinone. b. Stop the milrinone, flush the line, and then administer the furosemide. c. Administer the furosemide in a separate intravenous line. d. Notify the prescriber that the furosemide cannot be given at this time.

ANS: C Furosemide must not be injected into an intravenous line with milrinone because it will precipitate immediately. The infusion must not be stopped because of the patient's condition. A separate line will be needed. The other options are incorrect. DIF: COGNITIVE LEVEL: Applying (Application) REF: p. 387 TOP: NURSING PROCESS: Planning MSC: NCLEX: Safe and Effective Care Environment: Safety and Infection Control

29. The home health nurse warns the patient who is taking warfarin (Coumadin) for anticoagulant therapy for thrombophlebitis to stop taking the herbal remedy of ginkgo because ginkgo can: a. cause severe episodes of diarrhea. b. cause a severe skin eruption if taken with Coumadin. c. increase the action of the Coumadin. d. cause the Coumadin to be less effective.

ANS: C Herbal remedies such as ginkgo, garlic, angelica, and red clover can increase (potentiate) the action of the Coumadin. PTS: 1 DIF: Cognitive Level: Comprehension REF: Page 1589 OBJ: 21 TOP: Coumadin KEY: Nursing Process Step: Implementation MSC: NCLEX: Physiological Integrity

14. The nurse is reviewing the orders for a patient and notes a new order for an angiotensin-converting enzyme (ACE) inhibitor. The nurse checks the current medication orders, knowing that this drug class may have a serious interaction with what other drug class? a. Calcium channel blockers b. Diuretics c. Nonsteroidal antiinflammatory drugs d. Nitrates

ANS: C Nonsteroidal antiinflammatory drugs (NSAIDs) such as ibuprofen can reduce the antihypertensive effect of ACE inhibitors. In addition, the use of NSAIDs and ACE inhibitors may also predispose patients to the development of acute renal failure. DIF: COGNITIVE LEVEL: Applying (Application) REF: p. 355 TOP: NURSING PROCESS: Assessment MSC: NCLEX: Physiological Integrity: Pharmacological and Parenteral Therapies

14. The nurse notes a run of three ventricular contractions (PVC) that are not preceded by a P wave. This particular arrhythmia can progress into: a. atrial fibrillation and possible emboli. b. sinus tachycardia and syncope. c. ventricular tachycardia and death. d. sinus bradycardia and fatigue.

ANS: C PVCs are capable of progressing into ventricular tachycardia and death. PTS: 1 DIF: Cognitive Level: Application REF: Page 1547 OBJ: 10 TOP: PVCs KEY: Nursing Process Step: Assessment MSC: NCLEX: Physiological Integrity

9. The nurse administering the phosphodiesterase inhibitor milrinone (Primacor) recognizes that this drug will have a positive inotropic effect. Which result reflects this effect? a. Increased heart rate b. Increased blood vessel dilation c. Increased force of cardiac contractions d. Increased conduction of electrical impulses across the heart

ANS: C Positive inotropic drugs increase myocardial contractility, thus increasing the force of cardiac conduction. Positive chronotropic drugs increase the heart rate. Positive dromotropic drugs increase the conduction of electrical impulses across the heart. Blood vessel dilation is not affected. DIF: COGNITIVE LEVEL: Applying (Application) REF: p. 384 TOP: NURSING PROCESS: Assessment MSC: NCLEX: Physiological Integrity: Pharmacological and Parenteral Therapies

13. A patient has received an overdose of intravenous heparin, and is showing signs of excessive bleeding. Which substance is the antidote for heparin overdose? a. vitamin E b. vitamin K c. protamine sulfate d. potassium chloride

ANS: C Protamine sulfate is a specific heparin antidote and forms a complex with heparin, completely reversing its anticoagulant properties. Vitamin K is the antidote for warfarin (Coumadin) overdose. The other options are incorrect. DIF: COGNITIVE LEVEL: Applying (Application) REF: p. 437 TOP: NURSING PROCESS: Implementation MSC: NCLEX: Physiological Integrity: Pharmacological and Parenteral Therapies

24. The patient has become very dyspneic, respirations are 32, and the pulse is 100. The patient is coughing up frothy red sputum. What should be the initial nursing intervention? a. Lay the patient flat to reduce hypotension and the symptoms of cardiogenic shock. b. Place patient in side-lying position to reduce the symptoms of atrial fibrillation. c. Place patient upright with legs in dependent position to reduce the symptoms of pulmonary edema. d. Lay the patient flat and elevate the feet to increase venous return in cardiogenic shock.

ANS: C Signs and symptoms of pulmonary edema are restlessness; vague uneasiness; agitation; disorientation; diaphoresis; severe dyspnea; tachypnea; tachycardia; pallor or cyanosis; cough producing large quantities of blood-tinged, frothy sputum; audible wheezing and crackles; and cold extremities. The legs in a dependent position will decrease venous return and ease the pulmonary edema. PTS: 1 DIF: Cognitive Level: Analysis REF: Page 1572 OBJ: 12 TOP: Pulmonary edema KEY: Nursing Process Step: Implementation MSC: NCLEX: Physiological Integrity

12. The nurse assessing a cardiac monitor notes that the cardiac complexes each have a P wave followed by a QRS and a T. The rate is 120. The nurse recognizes this arrhythmia as: a. sinus bradycardia. b. atrial fibrillation. c. sinus tachycardia. d. ventricular tachycardia.

ANS: C Sinus tachycardia has a P wave followed by the QRS and the T. All the components of the complex are present and in the correct order, but the rate is over 100 beats a minute. PTS: 1 DIF: Cognitive Level: Application REF: Page 1545 OBJ: 8 TOP: Arrhythmias KEY: Nursing Process Step: Assessment MSC: NCLEX: Physiological Integrity

35. How should the nurse advise a patient with an international normalized ratio (INR) of 5.8? a. Make arrangements to go to the emergency room immediately b. Increase fluid intake to 2000 mL/day c. Stop taking the anticoagulant and notify health care provider d. Add more leafy green vegetables to patient diet

ANS: C The INR that is desired should be maintained between 2 and 3. A reading of 5.8 puts the patient at risk for hemorrhage. The patient should stop taking the anticoagulant and contact the physician for further instruction. PTS: 1 DIF: Cognitive Level: Application REF: Page 1546 OBJ: 6 TOP: Myocardial infarction KEY: Nursing Process Step: Assessment MSC: NCLEX: Physiological Integrity

2. The nurse clarifies that the master pacemaker of the heart is the: a. left ventricle. b. atrioventricular (AV) node. c. sinoatrial (SA) node. d. bundle of His.

ANS: C The SA node is the master pacemaker of the heart. PTS: 1 DIF: Cognitive Level: Application REF: Page 1533 OBJ: 10 TOP: Acute myocardial infarction KEY: Nursing Process Step: Planning MSC: NCLEX: Physiological Integrity

36. The nurse making a teaching plan for a patient with Buerger disease (thromboangiitis obliterans) will focus on the need for: a. reduction of alcohol intake. b. avoiding cold remedies. c. cessation of smoking. d. weight reduction.

ANS: C The hazards of cigarette smoking and its relationship to Buerger disease are the primary focus of patient teaching. None of the palliative treatments are effective if the patient does not stop smoking. Nowhere are the cause and effect of smoking so dramatically seen as with Buerger disease. PTS: 1 DIF: Cognitive Level: Knowledge REF: Page 1594 OBJ: 20 TOP: Buerger disease KEY: Nursing Process Step: Assessment MSC: NCLEX: Physiological Integrity

1. When the nurse is administering topical nitroglycerin ointment, which technique is correct? a. Apply the ointment on the skin on the forearm. b. Apply the ointment only in the case of a mild angina episode. c. Remove the old ointment before new ointment is applied. d. Massage the ointment gently into the skin, and then cover the area with plastic wrap.

ANS: C The old ointment should be removed before a new dose is applied. The ointment should be applied to clean, dry, hairless skin of the upper arms or body, not below the elbows or below the knees. The ointment is not massaged or spread on the skin, and it is not indicated for the treatment of acute angina. DIF: COGNITIVE LEVEL: Applying (Application) REF: p. 378 TOP: NURSING PROCESS: Implementation MSC: NCLEX: Safe and Effective Care Environment: Management of Care

7. A patient will be receiving a thrombolytic drug as part of the treatment for acute myocardial infarction. The nurse explains to the patient that this drug is used for which purpose? a. To relieve chest pain b. To prevent further clot formation c. To dissolve the clot in the coronary artery d. To control bleeding in the coronary vessels

ANS: C Thrombolytic drugs lyse, or dissolve, thrombi. They are not used to prevent further clot formation or to control bleeding. As a result of dissolving of the thrombi, chest pain may be relieved, but that is not the primary purpose of thrombolytic therapy. DIF: COGNITIVE LEVEL: Understanding (Comprehension) REF: p. 421 TOP: NURSING PROCESS: Implementation MSC: NCLEX: Physiological Integrity: Pharmacological and Parenteral Therapies

11. When teaching a patient who has a new prescription for transdermal nitroglycerin patches, the nurse tells the patient that these patches are most appropriately used for which situation? a. To prevent palpitations b. To relieve shortness of breath c. To prevent the occurrence of angina d. To keep the heart rate from rising too high during exercise

ANS: C Transdermal dosage formulations of nitroglycerin are used for the long-term prophylactic management (prevention) of angina pectoris. Transdermal nitroglycerin patches are not appropriate for the relief of shortness of breath, to prevent palpitations, or to control the heart rate during exercise. DIF: COGNITIVE LEVEL: Applying (Application) REF: p. 372 TOP: NURSING PROCESS: Implementation MSC: NCLEX: Physiological Integrity: Pharmacological and Parenteral Therapies

6. A calcium channel blocker (CCB) is prescribed for a patient, and the nurse provides instructions to the patient about the medication. Which instruction is correct? a. Chew the tablet for faster release of the medication. b. To increase the effect of the drug, take it with grapefruit juice. c. If the adverse effects of chest pain, fainting, or dyspnea occur, discontinue the medication immediately. d. A high-fiber diet with plenty of fluids will help prevent the constipation that may occur.

ANS: D Constipation is a common effect of CCBs, and a high-fiber diet and plenty of fluids will help to prevent it. Grapefruit juice decreases the metabolism of CCBs. Extended-release tablets must never be chewed or crushed. These medications should never be discontinued abruptly because of the risk for rebound hypertension. DIF: COGNITIVE LEVEL: Applying (Application) REF: p. 379 TOP: NURSING PROCESS: Implementation MSC: NCLEX: Physiological Integrity: Pharmacological and Parenteral Therapies

3. During a follow-up visit, the health care provider examines the fundus of the patient's eye. Afterward, the patient asks the nurse, "Why is he looking at my eyes when I have high blood pressure? It does not make sense to me!" What is the best response by the nurse? a. "We need to monitor for drug toxicity." b. "We must watch for increased intraocular pressure." c. "The provider is assessing for visual changes that may occur with drug therapy." d. "The provider is making sure the treatment is effective over the long-term."

ANS: D The physician would examine the fundus of a patient's eyes during antihypertensive therapy because it is a more reliable indicator than blood pressure readings of the long-term effectiveness of treatment. DIF: COGNITIVE LEVEL: Applying (Application) REF: p. 364 TOP: NURSING PROCESS: Evaluation MSC: NCLEX: Physiological Integrity: Physiological Adaptation

9. What action is often recommended to help reduce tolerance to transdermal nitroglycerin therapy? a. Omit a dose once a week. b. Leave the patch on for 2 days at a time. c. Cut the patch in half for 1 week until the tolerance subsides. d. Remove the patch at bedtime, and then apply a new one in the morning.

ANS: D To prevent tolerance, remove the transdermal patch at night for 8 hours, and apply a new patch in the morning. Transdermal patches must never be cut or left on for 2 days, and doses must not be omitted. DIF: COGNITIVE LEVEL: Applying (Application) REF: p. 371 TOP: NURSING PROCESS: Implementation MSC: NCLEX: Physiological Integrity: Pharmacological and Parenteral Therapies

5. When administering heparin subcutaneously, the nurse will follow which procedure? a. Aspirating the syringe before injecting the medication b. Massaging the site after injection c. Applying heat to the injection site d. Using a - to -inch 25- to 28-gauge needle

ANS: D A - to -inch 25- to 28-gauge needle is the correct needle to use for a subcutaneous heparin injection. The other options would encourage hematoma formation at the injection site. DIF: COGNITIVE LEVEL: Applying (Application) REF: p. 437 TOP: NURSING PROCESS: Implementation MSC: NCLEX: Physiological Integrity: Pharmacological and Parenteral Therapies

1. A patient has a new order for the adrenergic drug doxazosin (Cardura). When providing education about this drug, the nurse will include which instructions? a. "Weigh yourself daily, and report any weight loss to your prescriber." b. "Increase your potassium intake by eating more bananas and apricots." c. "The impaired taste associated with this medication usually goes away in 2 to 3 weeks." d. "Be sure to lie down after taking the first dose, because first-dose hypotension may make you dizzy."

ANS: D A patient who is starting doxazosin should take the first dose while lying down because there is a first-dose hypotensive effect with this medication. The other options are incorrect. DIF: COGNITIVE LEVEL: Analyzing (Analysis) REF: p. 362 TOP: NURSING PROCESS: Implementation MSC: NCLEX: Physiological Integrity: Reduction of Risk Potential

Hypertension is above 130/85. True or false?

False it is 140/90

2. A patient with severe liver disease is receiving the angiotensin converting enzyme (ACE) inhibitor, captopril (Capoten). The nurse is aware that the advantage of this drug for this patient is which characteristic? a. Captopril rarely causes first-dose hypotensive effects. b. Captopril has little effect on electrolyte levels. c. Captopril is a prodrug and is metabolized by the liver before becoming active. d. Captopril is not a prodrug and does not need to be metabolized by the liver before becoming active.

ANS: D A prodrug relies on a functioning liver to be converted to its active form. Captopril is not a prodrug, and therefore it would be safer for the patient with liver dysfunction. DIF: COGNITIVE LEVEL: Applying (Application) REF: p. 354 TOP: NURSING PROCESS: Assessment MSC: NCLEX: Physiological Integrity: Pharmacological and Parenteral Therapies

8. A patient has been taking a beta blocker for 4 weeks as part of his antianginal therapy. He also has type II diabetes and hyperthyroidism. When discussing possible adverse effects, the nurse will include which information? a. "Watch for unusual weight loss." b. "Monitor your pulse for increased heart rate." c. "Use the hot tub and sauna at the gym as long as time is limited to 15 minutes." d. "Monitor your blood glucose levels for possible hypoglycemia or hyperglycemia."

ANS: D Beta blockers can cause both hypoglycemia and hyperglycemia. They may also cause weight gain if heart failure is developing, and decreased pulse rate. The use of hot tubs and saunas is not recommended because of the possibility of hypotensive episodes. DIF: COGNITIVE LEVEL: Understanding (Comprehension) REF: p. 373 TOP: NURSING PROCESS: Implementation MSC: NCLEX: Safe and Effective Care Environment: Safety and Infection Control

3. A patient is receiving thrombolytic therapy, and the nurse monitors the patient for adverse effects. What is the most common undesirable effect of thrombolytic therapy? a. Dysrhythmias b. Nausea and vomiting c. Anaphylactic reactions d. Internal and superficial bleeding

ANS: D Bleeding, both internal and superficial, as well as intracranial, is the most common undesirable effect of thrombolytic therapy. The other options list possible adverse effects of thrombolytic drugs, but they are not the most common effects. DIF: COGNITIVE LEVEL: Remembering (Knowledge) REF: p. 431 TOP: NURSING PROCESS: Evaluation MSC: NCLEX: Physiological Integrity: Pharmacological and Parenteral Therapies

7. The nurse is reviewing discharge teaching for a patient who will be taking digoxin (Lanoxin) therapy. The nurse will teach the patient to avoid which foods when taking the digoxin? a. Leafy green vegetables b. Dairy products c. Grapefruit juice d. Bran muffins

ANS: D Bran, in large amounts, may decrease the absorption of oral digitalis drugs. The other foods do not affect digoxin levels. DIF: COGNITIVE LEVEL: Understanding (Comprehension) REF: p. 389 TOP: NURSING PROCESS: Implementation MSC: NCLEX: Physiological Integrity: Pharmacological and Parenteral Therapies

13. After an influenza-like illness, the patient complains of chills and small petechiae in his mouth and his legs. A heart murmur is detectable. These are characteristic signs of: a. congestive heart failure. b. heart block. c. aortic stenosis. d. infective endocarditis.

ANS: D Collection of subjective data includes noting patient complaints of influenza-like symptoms with recurrent fever, undue fatigue, chest pain, and chills. Objective data may reveal the significant signs of petechiae in the conjunctiva and mouth. Both subjective data and objective data are indicative of infective endocarditis. PTS: 1 DIF: Cognitive Level: Analysis REF: Page 1576 OBJ: 13 TOP: Endocarditis KEY: Nursing Process Step: Assessment MSC: NCLEX: Physiological Integrity

1. A patient about to receive a morning dose of digoxin has an apical pulse of 53 beats/minute. What will the nurse do next? a. Administer the dose. b. Administer the dose, and notify the prescriber. c. Check the radial pulse for 1 full minute. d. Withhold the dose, and notify the prescriber.

ANS: D Digoxin doses are held and the prescriber notified if the apical pulse is 60 beats/minute or lower or is higher than 100 beats/minute. The other options are incorrect. DIF: COGNITIVE LEVEL: Applying (Application) REF: p. 392 TOP: NURSING PROCESS: Implementation MSC: NCLEX: Physiological Integrity: Pharmacological and Parenteral Therapies

4. A patient has been taking digoxin at home but took an accidental overdose and has developed toxicity. The patient has been admitted to the telemetry unit, where the physician has ordered digoxin immune Fab (Digifab). The patient asks the nurse why the medication is ordered. What is the nurse's best response? a. "It will increase your heart rate." b. "This drug helps to lower your potassium levels." c. "It helps to convert the irregular heart rhythm to a more normal rhythm." d. "This drug is an antidote to digoxin and will help to lower the blood levels."

ANS: D Digoxin immune Fab (Digifab) is the antidote for a severe digoxin overdose. It is given intravenously. The other options are incorrect. DIF: COGNITIVE LEVEL: Applying (Application) REF: p. 388 TOP: NURSING PROCESS: Implementation MSC: NCLEX: Physiological Integrity: Pharmacological and Parenteral Therapies

2. The nurse is preparing to administer dipyridamole (Persantine). Which statement about this drug is true? a. It has antiinflammatory and antipyretic properties. b. It has analgesic properties as well as antithrombotic effects. c. It is useful in reducing the risk for fatal and nonfatal thrombotic stroke. d. It is used with warfarin to prevent postoperative thromboembolic complications.

ANS: D Dipyridamole (Persantine) is used as an adjunct to warfarin in the prevention of postoperative thromboembolic complications. The other options are incorrect. DIF: COGNITIVE LEVEL: Remembering (Knowledge) REF: p. 429 TOP: NURSING PROCESS: Planning MSC: NCLEX: Physiological Integrity: Pharmacological and Parenteral Therapies

39. The nurse making the schedule for the daily dose of furosemide (Lasix) would schedule the administration for which of the following times? a. Late in the afternoon b. At bedtime c. With any meal d. In the morning

ANS: D Diuretics should be scheduled for morning administration to avoid causing the patient nocturia. PTS: 1 DIF: Cognitive Level: Analysis REF: Page 1567, Table 47-6 OBJ: 12 TOP: Lasix KEY: Nursing Process Step: Planning MSC: NCLEX: Physiological Integrity

38. The nurse caring for a 92-year-old patient with pneumonia who is receiving IV carefully monitors the flow rate of the IV infusion because rapid infusion can cause: a. hypotension. b. thrombophlebitis. c. pulmonary emboli. d. heart failure.

ANS: D Heart failure can result from rapid infusion of intravenous fluids in older adults. PTS: 1 DIF: Cognitive Level: Knowledge REF: Page 1542, Lifespan OBJ: 9 TOP: Heart failure KEY: Nursing Process Step: Assessment MSC: NCLEX: Physiological Integrity

6.Cyclosporine should be given on an empty stomach: true or false?

False, it should be given with food

14. The nurse is reviewing new medication orders for a patient who has an epidural catheter for pain relief. One of the orders is for enoxaparin (Lovenox), a low-molecular-weight heparin (LMWH). What is the nurse's priority action? a. Give the LMWH as ordered. b. Double-check the LMWH order with another nurse, and then administer as ordered. c. Stop the epidural pain medication, and then administer the LMWH. d. Contact the prescriber because the LMWH cannot be given if the patient has an epidural catheter.

ANS: D LMWHs are contraindicated in patients with an indwelling epidural catheter; they can be given 2 hours after the epidural is removed. This is very important to remember, because giving an LMWH with an epidural has been associated with epidural hematoma. DIF: COGNITIVE LEVEL: Analyzing (Analysis) REF: p. 423 TOP: NURSING PROCESS: Planning MSC: NCLEX: Physiological Integrity: Pharmacological and Parenteral Therapies

11. A pregnant woman is experiencing hypertension. The nurse knows that which drug is commonly used for a pregnant patient who is experiencing hypertension? a. mannitol (Osmitrol) b. enalapril (Vasotec) c. hydrochlorothiazide (HydroDIURIL) d. methyldopa (Aldomet)

ANS: D Methyldopa is used in the treatment of hypertension during pregnancy. The other options are incorrect. DIF: COGNITIVE LEVEL: Understanding (Comprehension) REF: p. 352 TOP: NURSING PROCESS: Planning MSC: NCLEX: Physiological Integrity: Pharmacological Therapies

18. The patient has been hospitalized for hypertensive episodes three times in the last months. While preparing the discharge teaching plan, the nurse assesses that he does not comply with his medication regimen. The nurse's immediate course of action would be to: a. reteach him about his medications. b. have a serious talk with him and his family about compliance. c. arrange for home visits after discharge. d. collect more information to identify his reasons for noncompliance.

ANS: D Nursing interventions include measures to prevent disease progression and complications. Reteaching about medication will not identify the cause of noncompliance. PTS: 1 DIF: Cognitive Level: Application REF: Page 1556 OBJ: 18 TOP: Noncompliance KEY: Nursing Process Step: Planning MSC: NCLEX: Health Promotion and Maintenance

9. What do dark or "cold" spots on a thallium scan indicate? a. Tissue with adequate blood supply b. Dilated vessels c. Areas of neoplastic growth d. Tissue that has inadequate perfusion

ANS: D Thallium scans show adequate perfused areas by the collection of thallium. Dark spots or "cold spots" indicate tissues that have inadequate perfusion. PTS: 1 DIF: Cognitive Level: Application REF: Page 1539 OBJ: 6 TOP: Thallium scan KEY: Nursing Process Step: Planning MSC: NCLEX: Physiological Integrity

26. The nurse reminds the patient that the National Heart, Lung, and Blood Institute recommends a lipid study every years. a. 2 b. 3 c. 4 d. 5

ANS: D The National Heart, Lung, and Blood Institute recommend a lipid study every 5 years for all Americans, but especially for the older adult. PTS: 1 DIF: Cognitive Level: Comprehension REF: Page 1542 OBJ: 6 TOP: Lipid studies KEY: Nursing Process Step: Implementation MSC: NCLEX: Health Promotion and Maintenance

15. An elderly patient will be taking a vasodilator for hypertension. Which adverse effect is of most concern for the older adult patient taking this class of drug? a. Dry mouth b. Restlessness c. Constipation d. Hypotension

ANS: D The elderly are more sensitive to the blood pressure-lowering effects of vasodilators, and consequently experience more problems with hypotension, dizziness, and syncope. The other options are incorrect. DIF: COGNITIVE LEVEL: Applying (Application) REF: p. 361 TOP: NURSING PROCESS: Evaluation MSC: NCLEX: Physiological Integrity: Reduction of Risk Potential

1. The nurse is aware that the muscle layer of the heart, which is responsible for the heart's contraction, is the: a. endocardium. b. pericardium. c. mediastinum. d. myocardium.

ANS: D The myocardium is the specialized muscle layer that allows the heart to contract. PTS: 1 DIF: Cognitive Level: Comprehension REF: Page 1533 OBJ: 2 TOP: Myocardium KEY: Nursing Process Step: Implementation MSC: NCLEX: Physiological Integrity

2. The nurse is giving intravenous nitroglycerin to a patient who has just been admitted because of an acute myocardial infarction. Which statement is true regarding the administration of the intravenous form of this medication? a. The solution will be slightly colored green or blue. b. The intravenous form is given by bolus injection. c. It can be given in infusions with other medications. d. Non-polyvinylchloride (non-PVC) plastic intravenous bags and tubing must be used.

ANS: D The non-PVC infusion kits are used to avoid absorption and/or uptake of the nitrate by the intravenous tubing and bag and/or decomposition of the nitrate. The medication is given by infusion via an infusion pump and not with other medications. It is not given by bolus injection. If the parenteral solution is discolored blue or green, it should be discarded. DIF: COGNITIVE LEVEL: Applying (Application) REF: p. 379 TOP: NURSING PROCESS: Implementation MSC: NCLEX: Safe and Effective Care Environment: Management of Care

16. What should a person with unstable angina avoid? a. Walking outside b. Eating red meat c. Swimming in warm pool d. Shoveling snow

ANS: D The person with angina should avoid exposure to cold, heavy exercise, eating heavy meals, and emotional stress. PTS: 1 DIF: Cognitive Level: Application REF: Page 1552 OBJ: 9 TOP: Angina KEY: Nursing Process Step: Planning MSC: NCLEX: Physiological Integrity

5. A patient has been placed on a milrinone (Primacor) infusion as part of the therapy for end-stage heart failure. What adverse effect of this drug will the nurse watch for when assessing this patient during the infusion? a. Hypertension b. Hyperkalemia c. Nausea and vomiting d. Cardiac dysrhythmias

ANS: D The primary adverse effects seen with milrinone are cardiac dysrhythmias, mainly ventricular. It may also cause hypotension, hypokalemia, and other effects, but not nausea and vomiting. DIF: COGNITIVE LEVEL: Applying (Application) REF: p. 387 TOP: NURSING PROCESS: Assessment MSC: NCLEX: Physiological Integrity: Reduction of Risk Potential

9. A patient with primary hypertension is prescribed drug therapy for the first time. The patient asks how long drug therapy will be needed. Which answer by the nurse is the correct response? a. "This therapy will take about 3 months." b. "This therapy will take about a year." c. "This therapy will go on until your symptoms disappear." d. "Therapy for high blood pressure is usually lifelong."

ANS: D There is no cure for the disease, and treatment will be lifelong. The other answers are not appropriate. DIF: COGNITIVE LEVEL: Understanding (Comprehension) REF: p. 361 TOP: NURSING PROCESS: Implementation MSC: NCLEX: Physiological Integrity: Pharmacological and Parenteral Therapies

1. A patient has been prescribed warfarin (Coumadin) in addition to a heparin infusion. The patient asks the nurse why he has to be on two medications. The nurse's response is based on which rationale? a. The oral and injection forms work synergistically. b. The combination of heparin and an oral anticoagulant results in fewer adverse effects than heparin used alone. c. Oral anticoagulants are used to reach an adequate level of anticoagulation when heparin alone is unable to do so. d. Heparin is used to start anticoagulation so as to allow time for the blood levels of warfarin to reach adequate levels.

ANS: D This overlapping is done purposefully to allow time for the blood levels of warfarin to rise, so that when the heparin is eventually discontinued, therapeutic anticoagulation levels of warfarin will have been achieved. The full therapeutic effect of warfarin does not occur until 4 to 5 days after the first dose. This overlap of activity is required in patients who have been receiving heparin for anticoagulation and are to be switched to warfarin so that prevention of clotting is continuous. DIF: COGNITIVE LEVEL: Applying (Application) REF: p. 437 TOP: NURSING PROCESS: Implementation MSC: NCLEX: Physiological Integrity: Pharmacological and Parenteral Therapies

59) The nurse is preparing to administer adenosine to a patient with supraventricular tachycardia. Important considerations before administration include which of the following? (Select all that apply.) a. Constant ECG monitoring. b. Initial recommended dosage of 12 mg IV bolus. c. Rapid IV bolus administration. d. Saline flush following bolus. e. Long half-life of adenosine, which may prolong adverse medication effects.

ANSWER: a, c, d a. Continuous cardiac monitoring is required when administering any IV medications to treat dysrhythmias. c. Due to the short half-life of adenosine, the IV bolus should be administered rapidly. d. The initial dosage of adenosine is rapid IV bolus followed by a saline flush. b. The initial dosage of adenosine is rapid IV bolus of 6 mg. e. Adenosine has a short half-life.

60) The label for emergency IV lidocaine administration should read: a. "0.1% lidocaine with preservative." b. "2% lidocaine for topical use." c. "lidocaine with epinephrine." d. "lidocaine for cardiac dysrhythmias."

ANSWER: d; Lidocaine for IV use is different from lidocaine used as a local anesthetic. The label for lidocaine for IV use should read "lidocaine for dysrhythmias" or "lidocaine without preservatives." a, b, c: This is for topical anesthetic use.

Lorsartan (Cozaar) is a ___________

ARB

Check the cupboards of an elderly relative or patient (or your own cupboards) and read nutritional labels on packages. Determine if a typical day's use of the products on the shelf would meet the nutri- tional restrictions for someone on a cardioprotective diet. (Don't forget to check condiments, if they are likely to be used daily). Record your findings and the recommendations that you would make about the choice of food products. (1565)

Activity 5 72. Recall the patient teaching points when you are doing the food product calculations. • Recommended daily intake is 2 g sodium, 1500 calories, low cholesterol, and fluid restrictions. • Limit total fat intake to 25% to 35% of total calories each day. Limit intake of saturated fats to less than 7% of total fat intake. Teach the patient that saturated fats (e.g., shortening, lard, or butter) are solid at room temperature; better sources of fat include vegetable, olive, and fish oils. • Teach the patient to avoid foods high in sodium, saturated fats, and triglycerides. Review alternative ways of seasoning foods to avoid cooking with salt. Explain the need to limit intake of eggs, cream, butter, and foods high in animal fat. Teach the patient and family how to read labels on foods. • Teach the patient to eat 20-30 g of soluble fiber every day. Foods such as bran, beans, and peas help lower bad cholesterol (low-density lipoprotein). Recommendations will be based on what you found on the shelf. Typically, canned foods are higher in sodium than fresh foods and frozen premade meals are higher in fat. For elderly housebound people, canned or frozen food is likely to be more convenient, but some product lines are better than others. One suggestion for single elders (or busy nursing students) is to make a batch of healthy homemade soups, beans, casseroles, etc., and freeze in single-serving portions.

What is the first intervention for a client experiencing myocardial infarction? Administer morphine Administer oxygen Administer sublingual nitroglycerin Obtain an EKG

Administer oxygen

54. The patient with a history of heart failure tells the home health nurse, "Every night I sleep in this recliner chair. I feel better if I sleep with my head up" What will the nurse assess first? (1566) 1. Check for dependent edema in the lower extremities. 2. Look at accessibility to the bedroom and bathroom. 3. Assess ability to independently move and ambulate. 4. Ask about compliance with low-sodium, low-fat diet.

Answer 1: The patient is describing a corrective action that he uses to deal with orthopneaorthopnea. Worsening heart failure is accompanied by fluid retention and it is likely that sleeping in a chair is causing the fluid to collect in the lower extremities. As the edema worsens, the abdominal girth will increase and the breathing will become more labored as the fluid progresses upwards. The nurse is also likely to assess compliance with diet and medications. The home health nurse has an additional advantage of being able to look at the environment. Climbing stairs or navigating distances between rooms may be an issue as the patient becomes progressively more fatigued

Why must the INT be monitored for a patient with vavle replacement?

After vavle surgery will be place on anticogulants.

11.Pulus Paradoxus (R/t Pericarditis.. know that this is for that for this test)

Also paradoxic pulse or paradoxical pulse, is an abnormally large decrease in stroke volume, systolic blood pressure and pulse wave amplitude during inspiration. The normal fall in pressure is less than 10 mmHg. When the drop is more than 10 mmHg, it is referred to as pulsus paradoxus.

Dysrhythmia(1546)

An abnormal hearbeat

Cardioversion(1541)

An electric shock to your chest; restores your regular heartbeat

Which of the following complications is of greatest concern when caring for a preoperative AAA client? HTN Aneurysm rupture Cardiac arrhythmias Diminished pedal pulses

Aneurysm rupture

5. Beta blockers and calcium channel blockers decrease the frequency of what kind of attack?

Angina

Angioedema is a side effect of ACE inhibitors!

Angioedema is a side effect of ACE inhibitors!

Irbesartan (Avapro)

Angiotensin II Receptor Blocker

Valsartan (Diovan)

Angiotensin II Receptor Blocker

0) Sartan-

Angiotensin II Receptor blockers

these drugs end in -sartan

Angiotension II Receptor Blockers

65. A patient receives a prescription for antico- agulant medication for treatment of arterial emboli. What dietary information should the nurse give? (1592) 1. Do not increase intake of dark-green veg- etables because of vitamin K. 2. Take extra dairy products to ensure cal- cium intake and vitamin D. 3. Eat fruits such as citrus and bananas that provide potassium. 4. Avoid eating saturated fats by limiting use of butter, oils, and red meats.

Answer 1: Dark-green vegetables contain vitamin K which counteracts the effect of the anticoagulant drug

35. The nurse is planning care for several patients who are scheduled to have diagnostic test- ing for cardiac disorders. Which patient will require postprocedural checks for peripheral pulses, color, and sensation of the extremity ev- ery 15 minutes for 1 hour? (1538) 1. Needs cardiac catheterization to diagnose extent of atherosclerotic heart disease 2. Is scheduled for electrocardiogram to identify specific cardiac dysrthimas 3. Requires chemically induced stress electrocardiogram for poor exercise tolerance 4. Must have positron emission tomography because of coronary artery disease

Answer 1: During cardiac catheterization, the catheter is inserted into a peripheral vessel (usually the arm or the groin). There is a potential for bleeding or injury to nerves, so pulses and sensation distal to the site of insertion must be checked. Electrocardiograms and positron emission tomography are considered noninvasive.

42. The patient who had a myocardial infarction 2 weeks ago is now having frequent episodes of ventricular tachycardia. For this patient, what is the clinical significance of this dysrhythmia? 1.Waring sign for ventricular fibrillation 2.Expected finding at this stage 3.Reaction to a beta-adrenergic blocker 4.Treatment is given only for symptoms

Answer 1: For this patient, there is an increased risk for ventricular fibrillation. The patient may or may not have symptoms during the episodes, but aggressive treatment is likely in order to prevent ventricular fibrillation, which is a lethal dysrhythmia. Betaadrenergic blockers are used in the ongoing suppression of ventricular tachycardia.

58. Which disorder of the cardiovascular system places the patient at highest risk for the poten- tially life-threatening condition of cardiac tamponade? (1576) 1. Pericarditis 2. Valvular heart disease 3. Buerger's disease 4. Endocarditis

Answer 1: In pericarditis, the membranous sac that surrounds the heart becomes inflamed. Fluid collects in the sac and the heart becomes compressed by the pressure of the fluid. The effusion restricts the movement of the heart (cardiac tamponade)

33. The nurse is caring for a patient who is on anti- coagulant therapy. Which laboratory values are the most important to monitor? (1541) 1. Prothrombin time, International Normalized Ratio, and partial thrombo- plastin time 2. Blood glucose, potassium, sodium, calcium, and magnesium 3. Enzyme creatine kinase, creatine phosphokinase, and myoglobin 4. B-type natriuretic peptide and troponins 1 and 2

Answer 1: Prothrombin time, International Normalized Ratio, and partial thromboplastin time reflect blood clotting, so these laboratory values are the most important to follow up for patients who are on anticoagulant therapy. The electrolytes are important for heart muscle contraction. Enzyme creatine kinase, creatine phosphokinase, and myoglobin can be used to assist with the diagnosis of myocardial infarction, but troponin levels are now more commonly used. B-type natriuretic peptide is used in the diagnosis of heart failure.

62. The patient had a recent cardiac trans- plant. Which intervention is required for posttransplant care? (1580) 1. Immunosuppressivetherapy 2. Pericardiocentesis 3. Percutaneous transluminal angioplasty 4. Contact isolation

Answer 1: Transplant patients need immunosuppressive therapy and protective isolation. Pericardiocentesis is performed for cardiac tamponade. Percutaneous transluminal angioplasty is diagnostic and reparative for coronary artery disease or embolism.

46.Which instruction would the nurse give to the patient for self-administration of nitrate medi- cations? (1557) 1. Refrigerate the oral tablets and nitroglycerin patches until use. 2. Apply patches in the morning and remove them at bedtime. 3. A burning sensation on the tongue indicates an allergic reaction. 4. Pain relief should occur after a minimum of two doses.

Answer 2: Applying patches in the morning and removing them at bedtime prevents the development of tolerance. Nitroglycerin tablets should always be carried in a pocket or purse for immediate availability. A burning sensation under the tongue is expected during activation of the tablet. Up to three tablets should be taken to determine if pain relief is adequate.

37. During a discharge teaching session, the pa- tient voices concern about why her risk of heart disease is elevated simply because she has a history of diabetes mellitus. What is the best explanation to give to the patient? (1545) 1. Fluctuating insulin levels cause vasocon- striction. 2. Elevated blood glucose levels contribute to arterial damage. 3. Diabetics are obese and thus at higher risk. 4. Risk for heart disease is not higher for people with diabetes.

Answer 2: Elevation of blood glucose is thought to contribute to damage to the arterial intima and contribute to atherosclerosis.

56. The patient arrives in the emergency depart- ment with severe dyspnea, agitation, cyanosis, audible wheezes, and a cough with blood- tinged sputum. What is the priority nursing action? (1572, 1573) 1. Obtain a blood sample for arterial blood gases 2. Administer oxygen 3. Auscultate lung sounds 4. Establish a peripheral IV

Answer 2: Remember the priorities of airway and breathing and give the patient oxygen. Next establish a peripheral IV for morphine and diuretics. Arterial blood gases and auscultating lung sounds will assist in the diagnosis, but the patient is in severe distress and the symptoms are attended to first.

Modifiable risk factors for cardiovascular disease with a 23-year-old patient who is currently asymptomatic. What does the nurse recommend? (1544) 1. find out if any first-degree relatives had cardiovascular problems before 50 years of age. 2. Stop smoking or consider greatly reducing the number of cigarettes smoked per day. 3. Ask your health care provider for a choles- terol-lowering drug, such as simvastatin (Zocor). 4. Monitor weight and calorie intake to main- tain a body mass index of 30

Answer 2: Smoking cessation or at least reducing the number of cigarettes is a modifiable factor. Heredity plays a role, but is considered nonmodifiable. Prophylactic drugs would not be the first line of therapy for this healthy patient. Discussions of diet and exercise would be more appropriate. Body mass index of 30 is too high because this indicates obesity.

60. The nurse sees an elderly woman sitting in the waiting room and she is crying, "My grand- daughter was just diagnosed with infective endocarditis. Those patients usually die within a year" what would you say to comfort the grandmother? (1579) 1. "Surgical procedures can repair the diseased valves" 2. "These days, intensive antibiotic therapy cures 90% of Pts" 3. "If she is able to rest her heart, she will be okay" 4. "We will do everything we can to take care of her"

Answer 2: The grandmother is historically correct in thinking that patients die within a year, so she may be thinking about something that happened in the past. Giving her accurate and up-to-date information can help her reevaluate her granddaughter's chances for recovery. Talking about surgical procedures is premature at this point. Telling her about heart rest and staff taking care of the child are okay, but these are generalized statements that do little to explain the therapeutic advantages of current treatment.

66. The nurse is monitoring a patient who is wait- ing for diagnostic testing to determine if he has an aortic aneurysm. The patient suddenly reports severe chest pain. He becomes pale, weak, and confused. His pulse is 130 beats/ min and blood pressure is 85/50 mm Hg. What should the nurse do first ?(1594) 1. Call the health care provider 2. Put the patient in a supine position 3. Assess pain and give opioid medication 4. Establish a patent peripheral IV

Answer 2: The patient is showing signs and symptoms of a ruptured aneurysm and hypovolemic shock. The nurse would place the patient in a shock position and immediately call for help. (Note to student: See Chapter 46 nursing interventions for hypovolemic shock for additional information. Rapid response team, code team, or hospitalist may be available in different facilities.) The patient does need a patent IV. Giving pain medication is not a priority, although oxygen should be started.

50. The health care provider is considering tissue plasminogen activator (TPA) for a patient who is having an acute myocardial infarction. The wife suddenly rushes to the nurse and says, "We forgot to tell you something" Which disclosure is a contraindication for TPA? (1562) 1. "My husband is a Jehovah's Witness" 2. "My husband recently had a head injury" 3 "he forgot to take his insulin this morning" 4. "he had a small heart attack last year"

Answer 2: Thrombolytics are not used for patients with active internal bleeding, suspected aortic dissecting aneurysm, recent head trauma, history of hemorrhagic stroke within the past year, or surgery within the past 10 days

43. The patient is on the cardiac monitor under- going a diagnostic procedure. Suddenly, the health care provider says, "The patient is having ventricular fibrillation." What piece of equipment is the most vital? (1549) 1. Temporary pacemaker 2. Defibrillator 3. Bag-valve-mask 4. Crash cart

Answer 2: Ventricular fibrillation can be reversed if an electrical countershock is applied using the defibrillator. If defibrillation fails to convert the dysrhythmia, a bag-valve-mask with supplemental oxygen and a crash cart will be needed. A temporary pacemaker i

61. Which patient should be counseled about the risk of cardiomyopathy related to lifestyle choices? (1579) 1. High-risk sexual behavior 2. poor intake of dietary fiber 3. use of crack 4.Social consumption of alcohol

Answer 3: Cardiomyopathy caused by cocaine abuse is seen more frequently now than ever before. Cocaine causes intense vasoconstriction of the coronary arteries and peripheral vasoconstriction, resulting in hypertension. Cocaine also causes high circulating levels of catecholamines, which may further damage myocardial cells, leading to ischemic or dilated cardiomyopathy. The prognosis is poor. Excessive alcohol intake over a prolonged period of time also increases the risk.

55. The nurse is supervising a nursing student who must administer digoxin to a patient. The nurse would intervene if the student performs which action? (1570) 1. Stops to check the potassium level before administering the drug 2. Asks the patient if he has any questions or concerns about the drug 3. Tells the patient that his pulse is 55 beats/min and prepares to administer the drug 4. Checks to see if the drug causes any interactions with other prescribed drugs

Answer 3: Digoxin should be held for a pulse under 60/min. The other actions are correct.

67. The nurse is caring for a postsurgical patient. Which intervention is the most important in preventing deep vein thrombosis in the legs? (1600) 1. Applying elastic compression stockings 2. Elevating the lower extremities 3. Ensuring early ambulation and mobility 4. Measuring the calf circumference daily

Answer 3: Early ambulation and encouraging mobility, which includes change of position and range-of-motion exercises are the most important preventive measures. Compression stockings and calf measurements are part of prevention and detection. Elevating the legs may be ordered as a comfort measure if DVT occurs.

59. Which sign/symptom indicates to the nurse that a patient with endocarditis is experiencing a serious and common complication of the dis- ease? (1578) 1. Fever and chills 2. Joint pains and aches 3. Sudden shortness of breath 4. Petechiae on neck and chest

Answer 3: Endocarditis puts the patient at risk for emboli that can travel to any organ. Sudden shortness of breath suggest that a large embolus or numerous small emboli have lodged in the lungs. The other signs/symptoms are p

49. A neighbor tells the nurse that he has indiges- tion that has lasted 60 minutes. He tried "taking nitroglycerin, but that didn't help" What should the nurse do first? (1555) 1. Tell the neighbor to take an aspirin and then drive to the emergency department. 2. Stay with the neighbor, assist him to re- main calm, and call 911. 3. Assess the neighbor's use of nitroglycerin and assess for other symptoms. 4. Phone the neighbor's health care provider and ask for recommendations.

Answer 3: First, the nurse would determine if the correct dose and form of the nitroglycerin were taken. If the nitroglycerin was taken correctly, than the nurse may opt to quickly assess for other symptoms that suggest cardiac or digestive problems. Based on the assessment, the nurse may decide to call 911 or the health care provider. The neighbor should not drive himself to the hospital.

64. The nurse is caring for a patient who has pe- ripheral arterial disease with burning pain that occurs at rest in the right leg. For the nursing diagnosis of Ineffective Tissue Perfusion related to decreased artieal blood flow. what intervention will the nurse use? (1590) 1. Elevate the leg on a pillow 2. Use a covered ice compress 3. Place the leg in a dependent position 4. Encourage aerobic exercise for circulation

Answer 3: For arterial insufficiency, the leg should be dependent, because this will increase the blood flow to the tissues and help decrease the pain. The other options are likely to increase pain. Elevation and ice will decrease the bloo

41. For which dysrhythmia would a pacemaker mostly likely be necessary? (1552) 1. Sinus tachycardia 2.Premature ventricular contractions 3.Third-degree heart block 4. atrial fibrillation

Answer 3: In third-degree heart block, the impulses to stimulate heart muscle contraction are not being transmitted through the AV junction. The rate is very slow and symptoms of hypotension and angina are likely.

47.For a patient with myocardial infarction, what symptom is the most important? (1559) 1. Diaphoresis 2. Palpitations 3. Pain 4.Shortness of breath

Answer 3: Pain is the foremost symptom and is the target of immediate therapy, because pain is a signal of ischemia. Diaphoresis is secondary to pain or possibly hypotension. Palpitations could occur, but are not a typical complaint. Shortness of breath is related to the body's attempt to increase oxygen to the tissues.

57. The nurse is caring for a patient with valvular heart disease. Which task could be assigned to the UAP? (1574) 1. Identifying ADLs that cause fatigue 2. Check meal trays for high-sodium foods 3. Weigh the patient at the same time everyday 4. Explain the plan for rest periods

Answer 3: The UAP can weigh the patient. The other tasks are nursing responsibilities. (Note to student: Knowledge of correct nursing action and principles of delegation are combined to decide which action can be assigned or delegated to a UAP. Remember that UAP need specific instructions.)

51. The nurse is caring for a patient who is 40 hours post-myocardial infarction. Which instruction should be given to the UAP? (1563) 1. Assist the patient to ambulate in the hall three times. 2. Check to see if the patient is too tired to get up. 3. Encourage the patient to independently get out of bed. 4. Help the patient get to the commode chair.

Answer 4: For 24-48 hours, the patient is usually limited to getting up to the bedside commode; thereafter, the activity is gradually increased, but the nurse should carefully assess the patient before and after exertion and then give the UAP additional instructions about how to assist the patient.

48. A 63-year-old patient presents with fever, in- creased pulse, epistaxis, and joint involvement. Heart murmurs are auscultated. The patient has a history of inadequately treated childhood group A β-hemolytic streptococci pharyngitis. These findings are consistent with which medical diagnosis? (1575) 1. Cardiomyopathy 2. Angina 3. Left-sided heart block 4. Rheumatic heart disease

Answer 4: Fortunately, rheumatic fever now occurs less frequently in the United States, because treatment for group A β-hemolytic streptococci infections has improved. For older patients or for patients who have emigrated from undeveloped countries, the possibility for rheumatic heart disease still exists.

34. Laboratory results show a low hemoglobin for a patient diagnosed with myocardial infarction. What is the first therapy that the nurse would ensure to address this laboratory result? (1563) 1. Obtain an order for an intramuscular iron supplement. 2. Help the patient to order an iron-rich meal tray. 3. Obtain an order for type and cross for blood transfusion. 4. Check to see that oxygen is delivered as ordered.

Answer 4: Low hemoglobin indicates decreased ability to carry oxygen to the body cells and anemia, so the first action is to make sure that the patient is getting supplemental oxygen. (Oxygen is likely to have been previously ordered for a diagnosis of MI; if not, the nurse should start oxygen and then obtain an order.) The other options could also be included to correct low hemoglobin.

40. The patient experiences dizziness and light- headedness while trying to pass a bowel movement. An immediate pulse check shows 45 beats/min that rapidly recovers to a regular rate of 70. What is the most probable cause of this episode of sinus bradycardia? (1546) 1. Digitalis toxicity 2. Endocrine disturbance 3. Intracranial tumor 4. Vagal stimulation

Answer 4: Recall that bearing down is one way to cause vagal stimulation. The other options can also cause sinus bradycardia, but are less likely to have such a rapid recovery to a regular rate.

45. The patient had a percutaneous transluminal coronary angioplasty with stent placement. What type of medication is the patient most likely to be prescribed for at least 3 months? (1556) 1. Digitalis preparation 2. Diuretic 3.Opioid pain medication 4. Anticoagulant

Answer 4: Stents are thrombogenic; thus, the patient is likely to be prescribed an anticoagulant.

52. What is the best method to help a patient comply with dietary restrictions associated with atherosclerotic heart disease? (1565) 1. Tell him to avoid all foods that are high in fats. 2. Remind him that total fat intake is 35-40% of total caloric intake. 3.Tell him to eat 10-15 grams of soluble fiber every day. 4. Teach him how to read the nutritional labels on food products.

Answer 4: Teaching him how to read the labels gives him a practical skill that he can use at the grocery store. The other options are incorrect. Healthy fats that do not exceed 30% of the total calories are part of good nutrition. Fiber intake should be 20-30 grams

44. A patient is being discharged after receiving a permanent pacemaker. What is the best rationale to give to the patient about refraining from sports such as tennis, swimming, golf, and weight-lifting for 6-8 weeks?(1552) 1. "First, you have to be able to climb at least two flights of stairs" 2. "Active sports will interfere with the pacemaker's fixed mode" 3. "These sports are too strenuous and rapidly increase the heart rate" 4. "The arm on the pacemaker side should not be lifted over the head"

Answer 4: The arm on the pacemaker side should be immobilized for the first several hours; then for 6-8 weeks, the patient must refrain from lifting the arm over the head. Climbing stairs and participation in active sports are more related to recovery during cardiac rehabilitation. Electrical sources may interfere with the pacemaker's fixed mode.

63. What treatments and/or advice are given to patients who are prehypertensive? (1585) 1. Diuretics and low-sodium diet 2. Beta-adrenergic blockers and weight loss 3. Angiotensin II receptor blockers and lowfat diet 4. Lifestyle change and routine health appointments

Answer 4: The prehypertensive category was created to help people recognize that small increases in blood pressure can have large consequences on health. Patients would be advised about controlling modifiable risk factors and encouraged to participate in routine health appointments

53. The nurse is caring for a patient who has right ventricular heart failure. After therapy, the nurse sees that the patient has lost 5 pounds of weight. Assuming that all the weight represents fluid loss how much fluid has the Pt lost?______________ (1565)

Answer: 2.27 rounded to 2.3 liters. One liter of fluid equals 1 kg (2.2 pounds); a weight gain of 2.2 pounds signifies a gain of 1 liter of body fluid. 2.2 pounds : 5 pounds = 2.272 1 liter x

1.Precordial

Anterior chest over heart

simvastatin (Zocor)

Anti-lipid

Adenosine (Adenocard)

Antidysrhythmic agent. -Slows conduction time through the AV node, interrupts AV node pathways to restore NSR. Converts supraventricular tachycardia (SVT) to sinus rhythm. Rapid IV (1-2 seconds) push, flush immediately with normal saline. *Used for stress test

Amiodarone (Cordarone)

Antidysrhythmic agent. Prolongs repolarization, relaxes smooth muscles, decreases vascular resistance. For ventricular fibrillation and unstable ventricular tachycardia. Incompatible with heparin, may be given in PO maintenance dose, monitor for respiratory complications.

Procainamide (Pronestyl)

Antidysrhythmics may cause pericaridits

lidocaine (Xylocaine) IV

Antidysrhythmics; blocks vagus stimulation of myocytes

Atorvastatin (Lipitor)

Antihyperlipidemic

Lovastatin (Mevacor)

Antihyperlipidemic

Pravastatin (Pravachol)

Antihyperlipidemic

1. Angioedema is a side effect of which drug?

Antihypertensive

Hydralazine (Apresoline)

Antihypertensive may cause pericaridits

Classes of medications for hypertension?

Antihypertensives, beta blockers, ace inhibtiors, diuretics

Clopidogrel (Plavix)

Antiplatelet *for pts who cannot use ASA

Ticlopidine (Ticlid)

Antiplatelet *for pts who cannot use ASA

Cilostazol (Pletal)

Antiplatelet Used for intermittent claudication

Pentoxifylline (Trental)

Antiplatelet Used to prevent MI, stroke Used for intermittent claudication Can cause N/V Avoid anticoagulants May increase levels of theophylline

Dipyridamole (Persantine)

Antiplatelet Agent *Used for stress test

Nesiritide may cause _________

Apena

Which of the following parameters should be checked before administering digoxin? Apical pulse O2 saturation Radial pulse Respiratory rate

Apical pulse

What is the most common complication of a myocardial infarction? Cardiogenic shock Heart failure Arrhythmias Pericarditis

Arrhythmias

What is the definitive test used to diagnose a AAA? Abdominal x-ray Arteriogram CT Scan Ultrasound

Arteriogram

Why does hypertension go untreated?

Asymptomatic

Which of the following conditions most commonly results in CAD? Atherosclerosis DM MI Renal failure

Atherosclerosis

Analysis of rhythm strip: Atrial Rate: unmeasurable (rate of 350-600 bpm) Ventricular Rate: variable up 180 bpm. Rhythm: classically described as "irregularly irregular." P wave: not recognizable. PR interval: unmeasurable. QRS complex: normal. T wave: normal.

Atrial Fibrillation

Analysis of rhythm strip: Atrial Rate: between 240-320 bpm. Ventricular Rate: variable. Rhythm: regular or irregular depending on the ratio of block P wave: hallmark saw-toothed pattern. PR interval: not measurable. QRS complex: normal. T wave: not measureable.

Atrial Flutter

13) Enhance SA node automaticity and Av node conduction, reverse cholerogenic mediated decrease in HR, SVR, BP-

Atropine, antidote Physostigmine

QUESTION : A patient , age 65 , has chronic angina pectoris . Her proper use of nitroglycerin for pain take nitroglycerin fo r an episode of angina. management. She was unsure about how many times she should The best reply the nurse could make is: a. 'Continue to take nitroglycerin sublingually at 5-minute intervals until the pain is relieved. ) B."If the pain is not relieved after three physician and come to the hospital." doses of nitroglycerin at 5-minute intervals, call your c. "When nitroglycerin is not relieving the pain, lie down and rest." d. "Use oxygen at home to relieve pain when nitroglycerin is not successful."

B

2)Which of the following cardiac markers is specific to the heart, not influenced by skeletal muscle trauma or renal failure, and rises 3 hours following a myocardial infarction? A) CK-MB B) troponin I C) homocysteine D) myoglobin

B Troponin I is a myocardial muscle protein released into circulation after myocardial injury. It can identify very small amounts of myocardial injury. It rises 3 hours following a myocardial infarction, peaks at 14 to 18 hours, and returns to normal in 5 to 7 days. Troponin I is specific to the heart, not influenced by skeletal muscle trauma or renal failure, and is very useful in diagnosing a myocardial infarction. CK-MB is a cardiac enzyme that also is elevated following a myocardial infarction; however, it is also elevated by other factors (surgery, muscle trauma). It rises in 2 to 3 hours following a myocardial infarction, peaks at 24 hours, and returns to normal in 24 to 40 hours. Homocysteine is not a cardiac marker. It is an amino acid produced during protein digestion. Elevated levels of homocysteine may act as an independent risk factor for ischemic heart disease, cerebrovascular disease, peripheral arterial disease, and venous thrombosis. It appears to promote the development of atherosclerosis by causing endothelial damage, promoting LDL deposits, and promoting vascular smooth muscle growth. Myoglobin is a cardiac marker that is released into circulation a few hours after a myocardial infarction. Because it is also present in skeletal muscle, it lacks cardiac specificity.REF: Page 1541

9)Of the following, which is the most serious type of arrhythmia? a)Atrial fibrillation b)Ventricular fibrillation c)Supraventricular tachycardia d)Sinus bradycardia

B Ventricular fibrillation is a medical emergency that will result in death if left untreated. It is a state whereby the ventricles are quivering with disorganized electrical and mechanical activity. Prompt treatment, including CPR and defibrillation, are essential and must be performed promptly (ideally within 20 seconds) to give the patient the best chance of recovery. Atrial fibrillation, although serious, is not as imminently life threatening as ventricular fibrillation. In atrial fibrillation, electrical activity is disorganized, and the atria quiver at a rate of 350 to 600/min rather than contract in an organized manner. Ventricular response may be 100 to 180 beats per minute, and the patient experiences decreased cardiac output along with symptoms of palpitations, dyspnea, syncope, light-headedness, decreased level of consciousness, and pulmonary edema. Treatment involves slowing the ventricular rate, treating the atrial irritability, and treating the cause of the arrhythmia. Supraventricular tachycardia is the sudden onset of a rapid heartbeat, originating in the atria. It is characterized by a pulse rate of 150 to 250 beats per minute. The patient with supraventricular tachycardia may experience palpitations, light-headedness, dyspnea, and angina. Sinus bradycardia is a slow rhythm that originates in the SA node and is characterized by a rate of less than 60 beats per minute. Causes can be sleep, vomiting, intracranial tumors, myocardial infarction, vagal stimulation, endocrine disorders, and hypothermia. It may be completely normal in athletes. Treatment of sinus bradycardia depends on the cause.REF: Page 1548

Intermittent claudication (1582)

Cramps and weakness in your legs caused by decreased blood flow to your muscles

Which valves are most affect due to higher workload (Pressure)? (select all that apply) A) Tricuspid B) Mitral C) Pulmonary D) Aorta

B and D

32) A nurse knowing an anticoagulant act on clots by: A) Dissolving the clot B) Preventing further clot formation, allowing the body's natural response to lysis it C) Goes through a process called thromboxane D) Modifies the thromboxane, similar to NSAIDS and Aspirin

B) Preventing further clot formation, allowing the body's natural response to lysis it

Treatm ent of angina pectoris includes which of the following? (Select all that apply.) a. ECG b. Coronary artery bypass C. Coronary angioplasty D. Avoidance of caffeine and emotional stress. e Use of nitrates.

B,c,d,e

28. Some researchers believe that elevated levels of homocysteine can be treated by administration of vita- mins ___________, ___________, and ________________________. (1543)

B6; B12; folate

61) What is the a vasovagal maneuver?

Bearing Down: Medically referred to as the Valsalva Maneuver, this technique is one of the most common ways to stimulate the vagus nerve. The patient is instructed to bear down as if they were having a bowel movement. In effect, the patient is expiring against a closed glottis. An alternative way to perform a Valsalva Maneuver is to tell the patient to blow through an occluded straw or barrel of a 10 ml syringe for 15-20 seconds. These maneuvers increase intrathoracic pressure and stimulate the vagus nerve. Due to its ease of use, the physiology behind the Valsalva Maneuver will be discussed below.

Temazepam (Restoril)

Benzodiazepine Used to promote rest

Propranolol (Inderal)

Beta adrenergic blocker

Which of the following classes of drugs is most widely used in the treatment of cardiomyopathy? Antihypertensives Beta adrenergic blockers Calcium channel blockers Nitrates

Beta adrenergic blockers

Which type of medication reduces the oxygen requiremerfts of the myocardium by slowing the heart rate?

Beta blocking agents

Which of the following classes of medications protects the ischemic myocardium by blocking catecholamine and sympathetic nerve stimulation? Beta-adrenergic blockers Calcium channel blockers Narcotics Nitrates

Beta-adrenergic blockers

Beta-adrenergic blockers

Block beta-adrenergic stimulation and decrease myocardial oxygen demands, thus decreasing myocardial damage; decrease mortality rate

1) Beta blocker-

Blocks receptors in the heart, DRUGS end in olol

Nitroprusside monitor metheoglobin levels, whats a sign of it?

Blood is choclate brown without change on exposure to air

What types of fluids are administered to he patient with hypovolemic shock?

Bloof, colloid, crystaloid (salt in water aka saline)

vegatation

Build up On the valves, typically from an infection

A patient is i. Low dose dobutamine drip for heart failure. She had been feeling "better" but now has a feeling of tightness in her chest, palpitations, as well as a bit of anxiety. Her heart rate is up to 110 per minute, and her blood preasure is 150/98 mm Hg (previous HR was 86 per minute and BP 120/80mm Hg). What is nurse's immediate concern for this patient? A. She is experenicing normal adverse effects of dobutamine B. She may be experiencing an allergic reaction C. The medication may be causing a worsing of preexisiting cardiac disorder D. The dosage of the dobutamine should be increased to better control these symptoms

C

Nitroprusside may cause __________ toxicity

Cyanide

3)Which of the following is the correct impulse pattern of the cardiac conduction system? A)Pacemaker→Bundle of His→SA node→Bundle branches B)Purkinje fibers→SA node→Right and left bundle branches→AV node C)SA node→AV node→Bundle of His→Right and left bundle branches→Purkinje fibers D)Pacemaker→SA node→Bundle of His→AV node→Purkinje fibers

C The impulse pattern of the cardiac conduction system is as follows: SA node→AV node→Bundle of His→Right and left bundle branchesàPurkinje

65) What happens in cardioversion, if you do not synchronous with the patient's rhythm and hit them on the QRS? A) Nothing this is fine B) A little pain, but we saved his life C) V Fib D) Pulmonary edema

C) V Fib

14) What is the "widowmaker"? A) Digoxin toxicity B) Super Tachycardia C) Left anterior descending and left main artery D) Right anterior descending and right main artery

C) Left anterior descending and left artery (is a branch of the left coronary artery)

Propranolol (inderal), a beta blocker controls hypertension by: A. Blocking alpha receptors theoughout the body B. Increasing the diuretic response in the renal tubules C. Blocking the beta receptor stimulation in cardiac muscle and vessels D. Inhibit the RAAS system

C. Blocking the beta receptor stimulation in myocytes and blood vessels

Antihypertensive drug. Therapy for a newly dx'd, stage one african american patient woukd most likely indicate which drug? A. Vasodilators alone B. Ace inhibitors alone C. Calcium channel blockers with thiazide diuretic D. Beta-blockers

C. Calcium channel blockers with thiazide diuretics

A Pt is receving a theombolytic for treatment of an acute myocardial infarction. The nurse explains this is to: A. Relieve chest pain B. Prevent furher clot formation C. Dissolve the clot in his cornary artery D. Control bleeding in the cornary microcirculation

C. Dissolve the clot in his cornaty artery

A patient is guven a order fo ticlopidine (Ticilid) to a client. The nurse checks the clients's medical record to be sure the client does not have a history of which fillowing? A. Pulmonary enbolism B. Deep vein theombosis C. Liver disease D. Diabetes mellitus

C. Liver diseae

As an initial step in treating a client with angina, the physician prescribes nitroglycerin tablets, 0.3mg given sublingually. This drug's principle effects are produced by: a. Antispasmodic effect on the pericardium b. Causing an increased myocardial oxygen demand c. Vasodilation of peripheral vasculature d. Improved conductivity in the myocardium

C. Vasodilation of peripheral vasculature Given for chest pain , one every 5 minutes times 3 Encouraged to decrease activity To ED, if no relief Common side effect include throbbing headache, flushing, dizziness, burning under tongue

These drugs end in "zem and ipine"

Calcium channel blockers

Angiotensin-converting enzyme (ACE) inhibitors

Can help prevent ventricular remodeling and prevent or slow the progression of HF Prevent conversion of angiotensin I to angiotensin II Decrease endothelial dysfunction

Digoxin (Lanoxin)

Cardiac glycoside Positive inotrope, negative chronotrope and dromotrope Used for HF, a-fib Can cause dysrhythmias, bradycardia, GI effects (EARLY), vision changes (EARLY) Hypokalemia = toxicity; hyperkalemia = decreased effects Avoid verapamil Hold if pulse < 60 Therapeutic level = 0.5-2.0ng/mL Antidote: cholestyramine or Digibind Interacts w/ many meds

What is the term used to describe an enlargement of the heart muscle? Cardiomegaly Cardiomyopathy Myocarditis Pericarditis

Cardiomegaly

50% of heart transplants are performed due to what?

Cardiomyopathy

Which of the following conditions is most closely associated with weight gain, nausea, and a decrease in cardiac output? Angina pectoris Cardiomyopathy Left sided heart failure Right sided heart failure

Caridomyopathy

19) _________ are substances that produce a sympathomimetic response (stimulate the SNS)

Catecholamines

68) Identify three symptoms of a patient with a dysfunctioning pacemaker.

Change in LOC, Bradycardia, Hypotension

Which of the following is the most common symptom of myocardial infarction? Chest pain Dyspnea Edema Palpitations

Chest pain

Anginapectoris(1553)

Chest pain and choking sensations that are relieved by nitroglycerin

10) Paroxysmal:

Comes and goes

Caduet

Contains Norvasc and Lipitor; Lotensin HCT, which contains benazepril and hydrochlorothiazide; and others, in an attempt to lower the number of pills taken so patients will be more compliant in taking blood pressure-lowering medications

vs Inotropic=

Contraction force

Systole

Contraction of venticulars, S1 (Do not confuse with atrial systole)

Which of the following illnesses is the leading cause of death in the US? Cancer Coronary artery disease Liver failure Renal failure

Coronary artery disease

Which of the following conditions is most commonly responsible for myocardial infarction? Aneurysm Heart failure Coronary artery thrombosis Renal failure

Coronary artery thrombosis

Six Second Method

Count the number of QRS complexes in a 6 second strip and multiply by 10

Which of the following symptoms is most commonly associated with left sided heart failure? Crackles Arrhythmias Hepatic engorgement Hypotension

Crackles

What is intermittent claudication?

Cramp like pain in the calves due is ischemia

5)The nurse is caring for an older woman with cardiac disease. How does the older cardiac patient differ from the younger cardiac patient? A) The cardiac output tends to be increased in the older adult. B) The younger adult tends to have a more extensive network of collateral circulation than the older adult. C) Dyspnea is a typical symptom of a myocardial infarction in the younger adult, whereas angina is the more common symptom in the older adult. D) Even with lower doses of medications, the older adult should be observed for signs and symptoms of toxicity.

D Even with lower doses of medications, the older adult should be observed for signs and symptoms of toxicity, because the rate of drug metabolism and excretion decreases with age. The older adult should be encouraged to maintain regular contact with his or her physician and to seek care at the first sign of problems. The cardiac output tends to be decreased in the older adult due to changes in cardiac musculature and reduced efficiency and strength. Progressive coronary artery changes can lead to the development of collateral coronary circulation. The older adult tends to have a more extensive network of collateral circulation than the younger adult. Angina is a typical symptom of a myocardial infarction in the younger adult, whereas dyspnea may be a more common symptom in the older adult, owing to the severity of signs and symptoms being modified by the development of collateral circulation.REF: Page 1542

7)In evaluating risk factors for cardiovascular disease, which of the following does the nurse identify as a modifiable risk factor? A)Family history of cardiovascular disease B)Age C)Active lifestyle D)Hyperlipidemia

D Hyperlipidemia is a risk factor for cardiovascular disease. The ratio of high-density lipoproteins (HDLs) to low-density lipoproteins (LDLs) is the best predictor for cardiovascular disease. A diet high in saturated fat, calories, and cholesterol contributes to hyperlipidemia. Dietary control is an important factor in modifying the risk factor. Family history of cardiovascular disease is a risk factor for cardiovascular disease; however, it is a nonmodifiable risk factor. Advanced age is a risk factor for cardiovascular disease; however, it is a nonmodifiable risk factor. Inactive lifestyle is the modifiable risk factor for cardiovascular disease. Regular exercise can improve the heart's efficiency, lower blood glucose levels, lower blood pressure, help the patient to reduce weight, reduce stress, improve the ratio of HDLs to LDLs, and improve overall feelings of well-being.REF: Pages 1543-1545

67) Where is the current discharged during cardioversion? A) P Wave B) T Wave C) U wave D) QRS complex

D) QRS complex

5.Allopurinol inhibits the metabolism most of which drug? A) NSAIDS B) Tylenol C) Aspirin D) Azathioprine

D)Azathioprine

1 A patient has a new order for glatiramer acetate. The patient has not had an organ transplant. The nurse knows that the patient is receiving this drug for which condition? a Psoriasis b Rheumatoid arthritis c Irritable bowel syndrome d Relapse-remitting multiple sclerosis

D- Relapse-remitting multiple sclerosis

6)A patient was admitted yesterday for a myocardial infarction. Which of the following statements is true regarding treatment for a patient with myocardial infarction? A) Thrombolytic agents, such as aspirin, are used to minimize infarct size and maximize heart function. B)Thrombolytic agents must be initiated within 24 hours of the onset of symptoms in order to be effective. C)Morphine sulfate is contraindicated in a patient with a myocardial infarction due to its effects on the central nervous system. D)The patient with an acute myocardial infarction will be on bed rest with commode privileges for 24 to 48 hours.

D The patient with an acute myocardial infarction will be on bed rest with commode privileges for 24 to 48 hours. After this period, activities are resumed gradually, depending on the size of the infarct and patient characteristics. A program of cardiac rehabilitation will be designed for the patient and implemented. Thrombolytic agents are used to minimize infarct size and maximize heart function. Aspirin is an antiplatelet medication, not a thrombolytic agent. Thrombolytic agents must be initiated within 3 to 5 hours of the onset of symptoms to be effective, although it is most effective if administered within 30 minutes to 1 hour. Morphine sulfate is useful for the patient with a myocardial infarction, because it helps with vasodilation of coronary arteries, relief of pain, and reduction of apprehension. It also decreases myocardial oxygen demands, reduces contractility, and slows the heart rate.REF: Page 1562

A client ks taking amlodipine (NorVasc for hypertensijn. The nurse would tell the client to: A. Take with breakfest and grapefruit juice B. Take the dose with a source if protein C. The the does every other day D. Take on a empty stomach

D. Empty stomach

The label for emergency IV lidocaine adminsteration should read: A. "0.1 lidocaine with preservative" B. "2% lidocaine for topical use" C. "Lidocaine with epinephrine" D. "Lidocaine for cardiac dysrhythmia"

D. Lidocaine for cardiac dysrhythmia

Which nursing assessment finding confirms that the angiotension 2 receptor anataginist is effective? A. Weight loss less than 2 pounds B. LDL cholesterol levels have decreased C. Urinary output is increased D. Blood pressure has decreased

D. Lower BP

What physiological response is responible for the damage to organs with maligant hypertension?

Damage to the small arterioles

62) Which nursing diagnosis focuses on the physiologic aspects of dysrhythmias?

Decreased cardiac output

How is the degree of pitting edema meausred?

Depth of tissue

Calcium Channel Blockers increase serum ________.

Digoxin

Atrioventricular Block

Failure of conduction of impulses from the AV node to the atrioventricular bundle and ventricles of the heart.

A positive Homans sign is a reliable indicator of DVT? true of false

False

57) You should crush an anti-dysrhythmia: True or False

False (A lot are sustained released, crushing would defeat purpose)

3) A 12 Lead ECG is one single view of the heart at a point in time: True or false

False (An 12 Lead ECG is multiple portions of the heart electrical activity)

Undertreated or untreated Rheumatic heart disease is the third leading cause of valvar disease in adults: True or false

False (It is the leading case)

66) Dentures are required to stay in cardioversion, to maintain Pt homeostasis: True or false?

False (Remove them, they're a hazard)

Lubb (S1) occurs when the Mitral and Aorta valves close: True or False?

False (S1, first heart sound (lubb), produced by atrioventricular valve closure this is ALSO the SYSTOLE used to measure BP)

Dubb (S2) occurs when the tricuspid and pulmonary close: True or False?

False (S2, second heart sound (dubb), produced by semilunar valve closure, this is ALSO DIASTOLE we see when we measure BP)

31) True/False. Atrial fibrillation produces a hallmark saw-toothed pattern that is produced by the rapid firing of the atrial focus.

False (Saw-tooth pattern is Atrial flutter)

9.Tissue cross is DONE to confirm a match for heart transplant? True or false?

False (Tissue cross matching does not correlate with a prognosis outcome enough to justify the risk of a heart tissue biopsy , blood type and other matchings are used)

12.One reading of hypertension on one occasion is enough to Dx hypertension: True or false?

False (Two readings, two separate occasions)

64) You do not need to obtain consent for cardioversion. True or false

False (Typically informed consent is required, it isn't necessarily a emergency, Pt's are stable)

Cardiac Catheterization goes through which blood vessel to view the right side of the heart?

Femoral Vein (OR a VEIN in general, the take away is you need to use a vein to see the right side of heart)

47) List the abnormalities in each dysrhythmia: 1st Degree Heart Block

First degree AV block: prolonged PR interval

Grapefruit juice will increase serum levels of just about anything...

Grapefruit juice will increase serum levels of just about anything...

Which of the following conditions is linked to more than 50% of clients with a AAA? DM HTN PVD Syphillis

HTN

36) If the R is far from the P, then you have a first degree Longer, longer, longer, drop! Then you have a Wenkebach If some the P's just don't get through, then you have a Mobitz II If P's and Q's just don't agree, then you have a Thrid Degree

Heart Block Poem

Which of the following recurring conditions most commonly occurs in clients with cardiomyopathy? Heart failure DM MI Pericardial effusion

Heart failure

PTT is use for __________

Heparin

What is HIT?

Heparin Induced Thrombocytopenia

Which of the following risk factors for coronary artery disease cannot be corrected? Cigarette smoking DM Heredity HTN

Heredity

Identify the four types of shock:

Hypovolemic, distruputive, obstructive, cardiogenic

Azathioprine (Imuran)

Immunosuppressant

Cyclosporine

Immunosuppressant

40) Amiodarone

Known as an anti-arrhythmic drug. It works by blocking certain electrical signals in the heart that can cause an irregular heartbeat.

18) Potent rapid acting diuretic-

Lasix

Which of the following landmarks is the correct one for obtaining an apical pulse? Left fifth intercostal space, mid-axillary Left fifth intercostal space, mid-clavicular Left second intercostal space, mid-clavicular Left seventh intercostal space, mid-clavicular

Left fifth intercostal space, mid-clavicular

How does infection lead to heart failure?

Increase myocardial demand

55) Lidocaine

Increases K conductance, decreases APD (Action Potential duration) and ERP (effective refractory period).

25) Inflammation of heart-

Indomethacin, NSAIDS, corticosteroids, antibiotics, pericardial tap

Which of the following classes of medications maximizes cardiac performance in clients with heart failure by increasing ventricular contractility? Beta-adrenergic blockers Calcium channel blockers Diuretics Inotropic agents

Inotropic agents

31. ________________________ is pain (usually in the calves) brought on by exercise and relieved by rest. (1582)

Intermittent claudication

Dysrhythmias is a rsult of what?

Irritability of myocardial cells

2. What is angioedema?

Is the rapid edema, or swelling, of the area beneath the skin or mucosa. It is normally an allergic reaction, but it can also be hereditary. The swelling happens because fluid accumulates.

What are subjective symptoms of rheumatic heart disease?

Joint pain, lethargy, fatigue, abdominal pain

After an anterior wall myocardial infarction, which of the following problems is indicated by auscultation of crackles in the lungs? Left sided heart failure Pulmonic valve malfunction Right sided heart failure Tricuspid valve malfunction

Left sided heart failure

These drugs end in statin

Lipid lowering agents

Beta blockers usually end in

Lol

What are theee symptoms of a dysfunctionung pacemaker?

Loss of cosciousness, Hypotension, bradycardia

Amrinone (Inocor)

Mechanism of action: Phosphodiesterase-3 inhibitor -> Increased cAMP -> increased PKA activity. Cardiac: results in increased L-type Ca2+ opening and increased cardiac output. Vascular: increased K+ channel opening results in vasodilation Use: ACUTE decompensated heart failure Side effects: arrhythmia Notes: IV for acute decompensated heart failure. Short duration (minutes)

19) Selective B1 receptor blocker, reduces HR, contractile, and CO. prolongs AV conduction supre4ss renin secretion-

Metoprolol, Bronchospasms

33) Cather ablation

Minimally invasive procedure in which the doctor advances a flexible thin tube (catheter) through the blood vessels to your heart to ablate (stop) abnormal electrical pathways (signals) in the heart tissue.

Digitalis preparations, such as digoxin (Lanoxin)

Monitor apical pulse to ensure rate greater than 60 bpm blurred or colored vision anorexia, dysrhythmia, bradycardia, "tachycardia" (Odd)

nitroglycerin (Cardabid)

Monitor blood pressure for hypotension Monitor for headache and flushing

What is a fluoroscopy?

Moving X-ray

Analysis of rhythm strip: Rate: 60-100 bpm. Rhythm: regular (Consistent R-R and P-P intervals). P wave: present for each QRS complex, normal configuration, and each P wave is identical. P-R interval: normal (0.12-0.20 sec). QRS interval: normal (0.06-0.12 sec). T wave: upright.

NSR (Normal Sinus Rhythm)

________ should not be used for patients with cardiogenic shock or with a systolic blood pressure <90 mm Hg

Natrecor

S wave

Negative deflection after the R wave

Identify the 3 compensatory mechanisms in the first stage shock:

Neuro, endocrine, chemical

This is often used as a preliminary diagnostic tool to quickly differentiate angina from an MI.

Nitroglycerin (Nitrostat)

How can angina be differenttiated from myocardial infarction quickly and non invasively?

Nitroglyercin will relieve angina but nit myocardial infarction

71) Can a Pt with a Pacemaker use an electric blanket?

No, it may infer with the pacemaker.

Medical treatment of coronary artery disease includes all of the following procedures except? Stent placement Coronary artery bypass graft Oral medication administration Percutaneous transluminal coronary angioplasty

Oral medication administration

Identify the waves of a normal complex, the electrical activity they represent and the normal time intervals.

P wave: atrial depolarization. PR interval: time it takes impulse to travel from SA node through AV node (0.12-0.20 secs). QRS complex: ventricular depolarization (0.06-0.12 secs). T wave: ventricular repolarization.

What mnemonic device is used to assist in the assessment and documentation of perioheral vascular disease?

PATCHES

9)0.12-0.20 seconds is the normal ___ Interval

PR

_____________ are early beats that interrupt the underlying rhythm; they can arise from a single ectopic focus or from multiple foci within the ventricles.

PVC

51) List the abnormalities in each dysrhythmia: premature ventricular contraction

PVC: prolonged QRS Can delevop into ventricular tachycardia

37) Define PVC

PVCs are early beats that interrupt the underlying rhythm; they can arise from a single ectopic focus or from multiple foci within the ventricles.

Occlusion(1559)

Part of the blood vessel is blocked

Atherosclerosis impedes coronary blood flow by which of the following mechanisms? Plaque obstructs the vein Plaque obstructs the artery Blood clots form outside the vessel wall Hardened vessels dilate to allow the blood to flow through

Plaque obstructs the artery

What supplemental medication is most frequently ordered in conjunction with furosemide (Lasix)? Chloride Digoxin Potassium Sodium

Potassium

Analysis of rhythm strip: Rate: underlying rate may be slow, normal or fast Rhythm: underlying rhythm is usually regular with PVC coming earlier than the next expected complex P wave: not visible PR interval: not measurable QRS complex: wide and bizarre (greater than 0.12 seconds) T wave: may be oriented opposite to the direction of the QRS complex of the PVC

Premature Ventricular Contractions

Why are antibiotics used in the treatment VHD (Valvlar heart disease)?

Prevention of infective endocarditis Prophylaxis

2) ACE Inhibitors-

Prevents the RAAS system, Drugs end in PRIL, Angioedema, Dry cough, hyper kalmia

ACE inhbitors usually end in

Pril

Which of the following systems is the most likely origin of pain the client describes as knifelike chest pain that increases in intensity with inspiration? Cardiac Gastrointestinal Musculoskeletal Pulmonary

Pulmonary

A murmur is heard at the left second intercostal space along the sternal border. Which valve is this? Aortic Mitral Pulmonic Tricuspid

Pulmonic

What is the current discharged during cardioversion?

QRS complex

diasystole

RELAXATION PHASE OF THE HEART BEAT, WHEN VENTRICLES OPEN UP TO RECEIVE BLOOD FROM ATRIA, S2

Anticoagulant Bridge Therapy is what?

Refers to the fact that enoxaparin acts as a bridge to provide anticoagulation while the patient must be off of his or her warfarin therapy.

Endarterectomy(1591)

Removing the plaques from the inner part of arteries

In which of the following disorders would the nurse expect to assess sacral edema in a bedridden client? DM Pulmonary emboli Renal failure Right sided heart failure

Right sided heart failure

With which of the following disorders is jugular vein distention most prominent? Abdominal aortic aneurysm Right sided heart failure Myocardial infarction Pneumothorax

Right sided heart failure

Which of the following group of symptoms indicated a ruptured abdominal aneurysm? Lower back pain, increased BP, decreased RBC Severe lower back pain, decreased BP, decreased RBC Severe lower back pain, decreased BP, increased RBC Intermittent lower back pain, decreased BP, decreased RBC

Severe lower back pain, decreased BP, decreased RBC

Analysis of rhythm strip: Rate: less than 60 bpm. Rhythm: R-R and P-P intervals are regular. P wave: present for each QRS complex, normal configuration, and each P wave is identical. PR interval: normal. QRS complex: normal. T wave: normal.

Sinus Bradycardia

Analysis of rhythm: Rate: greater than 100. Rhythm: regular. P wave: present for each QRS complex, normal configuration, and each P wave is identical. P-R interval: normal. QRS complex: normal. T wave: normal.

Sinus Tachycardia

10) Adenosine-

Slows/stops conduction through AV node used for Tx of stable narrow complex SVT

34) Transesophageal Echocardiogram

Special type of echocardiogram. It is usually done when your doctor wants to look more closely at your heart to see if it could be producing blood clots. Done prior the cardioversion

Blood dyscrasia

Specifically, it is defined in current medicine as a morbid general state resulting from the presence of abnormal material in the blood, usually applied to diseases affecting blood cells or platelets. Evidence of dyscrasia can be present with a WBC (White Blood Cell) count of over 1,000,000.

7.Cardiac output equals?

Stroke volume X Heart rate (SVxHR)

28) Carotid sinus pressure is seen in ___________________?

Supraventricular Tachycardia

Analysis of rhythm strip: Rate: usually 150-250 bpm. Rhythm: regular. P wave: present for each QRS complex, normal configuration, and each P wave is identical. PR interval: normal. QRS complex: normal. T wave: normal.

Supraventricular Tachycardia

21) Ventricular Tachycardia-

Sync'd Cardioversion (low energy)

Stimulation of the sympathetic nervous system produces which of the following responses? Bradycardia Tachycardia Hypotension Decreased myocardial contractility

Tachycardia

58) What is the BEERS list?

The Beers Criteria for Potentially Inappropriate Medication Use in Older Adults, commonly called the Beers List, are guidelines for healthcare professionals to help improve the safety of prescribing medications for older adults.

ST segment

The ST segment encompasses the region between the end of ventricular depolarization and beginning of ventricular repolarization on the ECG. ... In clinical terms, the ST segment represents the period in which the myocardium maintains contraction to expel blood from the ventricles.

3.TEE reasoning

The advantage of TEE over TTE is usually clearer images, especially of structures that are difficult to view transthoracically (through the chest wall). The explanation for this is that the heart rests directly upon the esophagus leaving only millimeters that the ultrasound beam has to travel. This reduces the attenuation (weakening) of the ultrasound signal, generating a stronger return signal, ultimately enhancing image and Doppler quality. Comparatively, transthoracic ultrasound must first traverse skin, fat, ribs and lungs before reflecting off the heart and back to the probe before an image can be created. All these structures, along with the increased distance the beam must travel, weaken the ultrasound signal thus degrading the image and Doppler quality. In adults, several structures can be evaluated and imaged better with the TEE, including the aorta, pulmonary artery, valves of the heart, both atria, atrial septum, left atrial appendage, and coronary arteries. TEE has a very high sensitivity for locating a blood clot inside the left atrium.[3]

27. For many years, creatine phosphokinase (CK-MB) was the gold standard for the diagnosis of myocardial infarction, but now ________________________ are preferred. (1542)

Troponins 1 and 2 (I & M)

Why are arteries preferred over veins for bypass surgery?

The arteries are more elastic and stonger

Arteriosclerosis(1588)

The arteries are thicker and not as stretchy

10.A ST elevation may also me indicated in Pericarditis: True or false?

True

R wave

The first positive deflection produced by ventricular depolarization.

Myocardial infarction (1559)

The heart is being damaged by the lack of blood

Pulmonary edema(1570)

The heart is not pumping correctly so fluid is settling in your lungs

Grade 1 pitting edema

The pressure applied by the doctor leaves an indentation of 0-2 millimeters (mm) that rebounds immediately. This is the least severe type of pitting edema.

Grade 3 pitting edema

The pressure leaves an indentation of 5-6 mm that takes up to 30 seconds to rebound.

Grade 4 pitting edema

The pressure leaves an indentation of 8 mm or deeper. It takes more than 20 seconds to rebound.

3. Combs test is a test that detects:

The test detects antibodies against foreign red blood cells

Why you give a lower dose to old people?

They metabolize it slower, and are prone to toxicity

50) List the abnormalities in each dysrhythmia: 3rd Degree heart block

Third degree AV block: no relationship between P and QRS More Ps than QRSs Prolonged QRS

What is the primary reason for administering morphine to a client with a myocardial infarction? To sedate the client To decrease the client's pain To decrease the client's anxiety To decrease oxygen demand on the client's heart

To decrease oxygen demand on the client's heart

Toilet Paper My Ass

Tricuspid, Pulmonary, Mitral (Bicuspid) and Aorta

The gold standard of cardiac enzymes? And what does it establish?

Troponin I

Which of the following blood tests is most indicative of cardiac damage? Lactate dehydrogenase Complete blood count Troponin I Creatnine kinase

Troponin I

53) List the abnormalities in each dysrhythmia: ventricular fibrillation

V Fib: no discernible waveforms

52) List the abnormalities in each dysrhythmia: ventricular tachycardia

V Tach: rate (too high) No visible P waves QRS prolonged

Dopamine (Intropin)

Vasoactive-Sympathomimetics Raises systemic arterial pressure and cardiac output

Nitroglycerin (Nitrostat)

Vasodialtor take up to 3 tabs during single episode place under tongue store in original container discard on expiration date

11) Synthetic analog of ADH increase the reabsorption of water, used for diabetes insipidus, refractory V fib and vasodilaoty shock-

Vassopressin

Analysis of rhythm strip: Rate: not discernible Regularity: not discernible Interval Measurements: not discernible Shape and sequence: The baseline is wavy and chaotic, with no PQRST complexes

Ventricular Fibrillation

Analysis of rhythm strip: Rate: usually 140 to 240 bpm. Interval measurements: no PR interval. QRS complex is > 0.12 seconds. Shape and Sequence: QRS waves are consistent in shape but appear wide and bizarre.

Ventricular Tachycardia

three or more succesive PVC, rapid life-threatening dysrhythmia originating from a single ectopic focus in ventricles

Ventricular Tachycardia

Tachycardia(1546)

Very fast heart rate

What is VLDL, LDL and HDL

Very low, low and high density lipoprotein cholstrols

PT is used for _____________

Warfarin

What are the classic sides of right sided heart failure?

Weight gain greater than 10lbs Junglar vein distention

6 When assessing a patient who is to begin therapy with an immunosuppressant drug, the nurse recognizes that such drugs should be used cautiously in patients with which condition(s)? (Select all that apply.) a Pregnancy b Glaucoma c Anemia d Myalgia e Renal dysfunction f Hepatic dysfunction

a Pregnancy d Myalgia f Hepatic dysfunction

5 During drug therapy with basiliximab, the nurse monitors for signs of cytokine release syndrome, which results in a fever, dyspnea, and general malaise. b neurotoxicity and peripheral neuropathy. c hepatotoxicity with jaundice. d thrombocytopenia with increased bleeding tendencies.

a fever, dyspnea, and general malaise.

Ventricular Tachycardia

a very rapid heartbeat that begins within the ventricles

13) The nurse caring for a patient who is receiving beta1 agonist drug therapy needs to be aware that these drugs cause which effect? a) Increased cardiac contractility b) Decreased heart rate c) Bronchoconstriction d) Increased GI tract motility

a) Increased cardiac contractility

6. Do Adrenergics improve strength of heart contraction?

Yes

______________________________ and _________________________________ have been identified as good forms of exercise to decrease the risk of developing cardiovascular disease. (1545)

Yoga; walking

1. Adrenergic blockade at the alpha-adrenergic recep- tors leads to which of the following effects? (Select all that apply.) a. Vasodilation b. Decreased blood pressure c. Increased blood pressure d. Constriction of the pupil e. Tachycardia

a,b,d

Hypoxemia (1541)

You do not have enough oxygen in your blood

3. The nurse is aware that adrenergic drugs may be used to treat which conditions? (Select all that apply.) a. Asthma b. Open-angle glaucoma c. Hypertension d. Nasal congestion e. Seizures f. Nausea and vomiting

a,b,d

1 The nurse caring for a patient who is receiving beta1 agonist drug therapy needs to be aware that these drugs cause which effect? a Increased cardiac contractility b Decreased heart rate c Bronchoconstriction d Increased GI tract motility

a

1 Which action by the nurse is most appropriate for the patient receiving an infusion of packed red blood cells? a Flush the IV line with normal saline before the blood is added to the infusion. b Flush the IV line with dextrose before the blood is added to the infusion. c Check the patient's vital signs once the infusion is completed. d Anticipate that flushed skin and fever are expected reactions to a blood transfusion.

a

2 When administering niacin, the nurse needs to monitor for which adverse effect? a Cutaneous flushing b Muscle pain c Headache d Constipation

a

2 When giving antihypertensive drugs, the nurse will consider giving the first dose at bedtime for which class of drugs? a Alpha blockers such as doxazosin (Cardura) b Diuretics such as furosemide (Lasix) c ACE inhibitors such as captopril (Capoten) d Vasodilators such as hydralazine (Apresoline)

a

2 When monitoring laboratory test results for patients receiving loop and thiazide diuretics, the nurse knows to look for a decreased serum levels of potassium. b increased serum levels of calcium. c decreased serum levels of glucose. d increased serum levels of sodium.

a

2. The nurse discovers that the intravenous infusion of a patient who has been receiving an intravenous va- sopressor has infiltrated. The nurse will expect which drug to be used to reverse the effects of the vasopres- sor in the infiltrated area? a. phentolamine (Regitine) b. prazosin (Minipress) c. epinephrine (Adrenalin) d. metoprolol (Lopressor)

a

3 When monitoring a patient who is receiving an intravenous infusion of nesiritide (Natrecor), the nurse will look for which adverse effect? a Dysrhythmia b Proteinuria c Hyperglycemia d Hypertension

a

3 Which statement by the patient reflects the need for additional patient education about the calcium channel blocker diltiazem (Cardizem)? a "I can take this drug to stop an attack of angina." b "I understand that food and antacids alter the absorption of this oral drug." c "When the long-acting forms are taken, the drug cannot be crushed." d "This drug may cause my blood pressure to drop, so I need to be careful when getting up."

a

3. Which statement regarding digoxin therapy and po- tassium levels is correct? a. Low potassium levels increase the chance of di- goxin toxicity. b. High potassium levels increase the chance of digoxin toxicity. c. Digoxin reduces the excretion of potassium in the kidneys. d. Digoxin promotes the excretion of potassium in the kidneys.

a

4. A patient is being treated for a hypertensive emer- gency. The nurse expects which drug to be used? a. sodium nitroprusside (Nitropress) b. losartan (Cozaar) c. captopril (Capoten) d. prazosin (Minipress)

a

5. Patients who are taking beta blockers for angina need to be taught which information? a. These drugs are for long-term prevention of an- gina episodes. b. These drugs must be taken as soon as angina pain occurs. c. These drugs will be discontinued if dizziness is experienced. d. These drugs need to be carried with the patient at all times in case angina occurs

a

6. A beta1 blocker is prescribed for a patient with heart failure and hypertension. Which adverse effects, if present, may indicate a serious problem is develop- ing while the patient is on this medication? (Select all that apply.) a. Edema b. Nightmares c. Shortness of breath d. Nervousness e. Constipation

a

7. During his morning walk, a man begins to experi- ence chest pain. He sits down and takes one nitro- glycerin sublingual tablet. After 5 minutes, the chest pain is worsening. What action would be the priority in this situation? a. Call 911 (emergency medical services). b. Take another nitroglycerin tablet. c. Take two more nitroglycerin tablets at the same time. d. Sit quietly to wait for the pain to subside.

a

2 When giving antihypertensive drugs, the nurse will consider giving the first dose at bedtime for which class of drugs? a Alpha blockers such as doxazosin (Cardura) b Diuretics such as furosemide (Lasix) c ACE inhibitors such as captopril (Capoten) d Vasodilators such as hydralazine (Apresoline)

a Alpha blockers such as doxazosin (Cardura)

6 The nurse is administering an intravenous infusion of a phosphodiesterase inhibitor to a patient who has heart failure. The nurse will evaluate the patient for which therapeutic effects? (Select all that apply.) a Positive inotropic effects b Vasodilation c Decreased heart rate d Increased blood pressure e Positive chronotropic effect

a,b,e

6.The nurse is reviewing the medication orders of a newly admitted patient who has an infusion of the adrenergic drug dopamine. Which of these drugs or drug classes, if also given to the patient, may cause an interaction? (Select all that apply.) a. Tricyclic antidepressants b. Monoamine oxidase inhibitors (MAOIs) c. Anticoagulants d. Corticosteroids e. Antihistamines

a,b,e

6 A patient is taking an alpha blocker as treatment for benign prostatic hyperplasia. The nurse will monitor for which potential drug effects? (Select all that apply.) a Orthostatic hypotension b Increased blood pressure c Increased urine flow d Headaches e Bradycardia

a,c,d

5. When caring for a patient who is taking digoxin, the nurse will monitor for which signs and symptoms of toxicity? (Select all that apply.) a. Anorexia b. Diarrhea c. Visual changes d. Nausea and vomiting e. Headache f. Bradycardia

a,c,d,e,f

6 The nurse is monitoring a patient who is receiving an infusion of a beta-adrenergic agonist. Which adverse effects may occur with this infusion? (Select all that apply.) a Mild tremors b Bradycardia c Tachycardia d Palpitations e Drowsiness f Nervousness

a,c,d,f

3. The nurse is reviewing dosage forms of nitroglycerin (Nitrostat). This drug can be given by which routes? (Select all that apply.) a. Continuous intravenous drip b. Intravenous bolus c. Sublingual spray d. Oral dosage forms e. Topical ointment f. Rectal suppository

a,c,d.e

6 The nurse is reviewing drug interactions with a male patient who has a prescription for isosorbide dinitrate (Isordil) as treatment for angina symptoms. Which substances listed below could potentially result in a drug interaction? (Select all that apply.) a A glass of wine b Thyroid replacement hormone c tadalafil (Cialis), an erectile dysfunction drug d metformin (Glucophage), an antidiabetic drug e carvedilol (Coreg), a beta blocker

a,c,e

6 An elderly patient has been discharged following treatment for a mild case of heart failure. He will be taking a loop diuretic. Which instruction(s) from the nurse are appropriate? (Select all that apply.) a "Take the diuretic at the same time each morning." b "Take the diuretic only if you notice swelling in your feet." c "Be sure to stand up slowly because the medicine may make you feel dizzy if you stand up quickly." d "Drink at least 8 glasses of water each day." e "Here is a list of foods that are high in potassium—you need to avoid these." f "Please call your doctor immediately if you notice muscle weakness or increased dizziness."

a,c,f

13. The nurse is reviewing the use of anticoagulants. Anticoagulant therapy is appropriate for which con- ditions? (Select all that apply.) a. Atrial fibrillation b. Thrombocytopenia c. Myocardial infarction d. Presence of mechanical heart valve e. Aneurysm f. Leukemia

a. Atrial fibrillation c. Myocardial infarction d. Presence of mechanical heart valve

6. A beta1 blocker is prescribed for a patient with heart failure and hypertension. Which adverse effects, if present, may indicate a serious problem is develop- ing while the patient is on this medication? (Select all that apply.) a. Edema b. Nightmares c. Shortness of breath d. Nervousness e. Constipation

a. Edema c. Shortness of breath

15. A patient is at risk for a stroke .Which drug is recommended to prevent platelet aggregation for stroke prevention by the American Stroke Society? a. aspirin b.warfarin sodium (Coumadin) c. heparin d. alteplase (Activase)

a. aspirin

4. A patient is being treated for a hypertensive emergency. The nurse expects which drug to be used? a. sodium nitroprusside (Nitropress) b. losartan (Cozaar) c. captopril (Capoten) d. prazosin (Minipress)

a. sodium nitroprusside (Nitropress)

Clonidine (Catapres)

alpha 2 agonist

Supraventricular Tachycardia (SVT)

an abnormal heart rhythm arising from aberrant electrical activity in the heart; originates at or above the AV node

isosorbide dinitrate (Isordil)

antianginal, vasodilator

isosorbide mononitrate (Imdur)

antianginal, vasodilator

Diazepam (Valium)

anxiety *reduce stress, hence workload

PQ (PR) interval

atria contract and begin to relax, ventricles begin to contract

P wave

atrial depolarization (contraction)

1 A nurse administering niacin would implement which action to help to reduce adverse effects? a Give the medication with grapefruit juice. b Administer a small dose of aspirin or an NSAID 30 minutes before the niacin dose. c Administer the medication on an empty stomach. d Have the patient increase dietary fiber intake.

b

1 The nurse is administering antihypertensive drugs to older adult patients. The nurse knows that which adverse effect is of most concern for these patients? a Dry mouth b Hypotension c Restlessness d Constipation

b

1 When teaching the patient about the signs and symptoms of cardiac glycoside toxicity, the nurse should alert the patient to watch for a visual changes such as photophobia. b flickering lights or halos around lights. c dizziness when standing up. d increased urine output.

b

2. The nurse is administering an adrenergic drug and will monitor for which possible effect? a. Hypotension b. Increased heart rate c. Decreased respiratory rate d. Urinary retention

b

3 A 56-year-old man started antihypertensive drug therapy 3 months earlier and is in the office for a follow-up visit. While the nurse is taking his blood pressure, he informs the nurse that he has had some problems with sexual intercourse. Which would be the most appropriate response by the nurse? a "Not to worry. Eventually, tolerance will develop." b "The physician can work with you on changing the dose and/or drugs." c "Sexual dysfunction happens with this therapy, and you will learn to accept it." d "This is an unusual occurrence, but it is important to stay on your medications."

b

3. A patient with type 2 diabetes mellitus has developed hypertension. What is the blood pressure goal for this patient? a. Less than 110/80 mm Hg b. Less than 130/80 mm Hg c. Less than 130/84 mm Hg d. Less than 140/90 mm Hg

b. Less than 130/80 mm Hg

3 The nurse is teaching a patient about self-administration of enoxaparin (Lovenox). Which statement should be included in this teaching session? a "We will need to teach a family member how to give this drug in your arm." b "This drug is given in the folds of your abdomen, but at least 2 inches away from your navel." c "This drug needs to be taken at the same time every day with a full glass of water." d "Be sure to massage the injection site thoroughly after giving the drug."

b

3 When assessing a patient who is about to receive an albumin infusion, the nurse knows that a contraindication for albumin would be a acute liver failure. b heart failure. c severe burns. d fluid-volume deficit.

b

3. A patient with type 2 diabetes mellitus has devel- oped hypertension. What is the blood pressure goal for this patient? a. Less than 110/80 mm Hg b. Less than 130/80 mmHg c. Less than 130/84 mmHg d. Less than 140/90 mmHg

b

4 A patient is receiving dabigatran (Pradaxa), 150 mg twice daily, as part of treatment for atrial fibrillation. Which condition, if present, would be a concern if the patient were to receive this dose? a Asthma b Renal impairment c History of myocardial infarction d Elevated liver enzyme

b

4 A patient is taking a beta blocker as part of the treatment plan for heart failure. The nurse knows that the purpose of the beta blocker for this patient is to a increase urine output. b prevent stimulation of the heart by catecholamines. c increase the contractility of the heart muscle. d cause peripheral vasodilation.

b

4 Which statement needs to be included when the nurse provides patient education for a patient with heart failure who is taking daily doses of spironolactone (Aldactone)? a "Be sure to eat foods that are high in potassium." b "Avoid foods that are high in potassium." c "Avoid grapefruit juice while taking this medication." d "A low-fiber diet will help prevent adverse effects of this medication."

b

4. A patient has been given an alpha-blocker as treat- ment for benign prostatic hyperplasia. Which in- struction is important to include when the nurse is teaching him about the effects of this medication? a. Avoid foods and drinks that contain caffeine. b. Change to sitting or standing positions slowly to avoid a sudden drop in blood pressure. c. Watch for weight loss of 2 pounds within a week. d. Take extra supplements of calcium.

b

4. When infusing milrinone (Primacor), the nurse will keep which consideration in mind? a. The patient must be monitored for hyperkalemia. b. The patient's cardiac status must be monitored closely. c. The drug may cause reddish discoloration of the extremities. d. Hypertension is the primary effect seen with ex- cessive doses.

b

5 A 68-year-old man has been taking the nitrate isosorbide dinitrate (Isordil) for 2 years for angina. He recently has been experiencing erectile dysfunction and wants a prescription for sildenafil (Viagra). Which response would the nurse most likely hear from the prescriber? a "He will have to be switched to isosorbide mononitrate if he wants to take sildenafil." b "Taking sildenafil with the nitrate may result in severe hypotension, so a contraindication exists." c "I'll write a prescription, but if he uses it, he needs to stop taking the isosorbide for one dose." d "These drugs are compatible with each other, and so I'll write a prescription."

b

5 A patient has received a double dose of heparin during surgery and is bleeding through the incision site. While the surgeons are working to stop the bleeding at the incision site, the nurse will prepare to take what action at this time? a Give IV vitamin K as an antidote b Give IV protamine sulfate as an antidote c Call the blood bank for an immediate platelet transfusion d Obtain an order for packed red blood cells

b

5 A patient is currently taking a statin. The nurse considers that the patient may have a higher risk of developing rhabdomyolysis when also taking which product? a NSAIDs b A fibric acid derivative c Orange juice d Fat soluble vitamins

b

5 When a drug is characterized as having a negative chronotropic effect, the nurse knows to expect which effect? a Reduced blood pressure b Decreased heart rate c Decreased ectopic beats d Increased force of cardiac contractions

b

6. A patient with coronary artery spasms will be most effectively treated with which type of antianginal medication? a. Beta blockers b. Calcium channel blockers c. Nitrates d. Nitrites

b

3 A 56-year-old man started antihypertensive drug therapy 3 months earlier and is in the office for a follow-up visit. While the nurse is taking his blood pressure, he informs the nurse that he has had some problems with sexual intercourse. Which would be the most appropriate response by the nurse? a "Not to worry. Eventually, tolerance will develop." b "The physician can work with you on changing the dose and/or drugs." c "Sexual dysfunction happens with this therapy, and you will learn to accept it." d "This is an unusual occurrence, but it is important to stay on your medications."

b "The physician can work with you on changing the dose and/or drugs."

3 The nurse is teaching a patient about self-administration of enoxaparin (Lovenox). Which statement should be included in this teaching session? a "We will need to teach a family member how to give this drug in your arm." b "This drug is given in the folds of your abdomen, but at least 2 inches away from your navel." c "This drug needs to be taken at the same time every day with a full glass of water." d "Be sure to massage the injection site thoroughly after giving the drug."

b "This drug is given in the folds of your abdomen, but at least 2 inches away from your navel."

5 A patient has a new prescription for an ACE inhibitor. During a review of the patient's list of current medications, which would cause concern for a possible interaction with this new prescription? (Select all that apply.) a A benzodiazepine taken as needed for allergies b A potassium supplement taken daily c An oral anticoagulant taken daily d An opioid used for occasional severe pain e An NSAID taken as needed for headaches

b A potassium supplement taken daily e An NSAID taken as needed for headaches

6 A patient is starting warfarin (Coumadin) therapy as part of treatment for atrial fibrillation. The nurse will follow which principles of warfarin therapy? (Select all that apply.) a Teach proper subcutaneous administration b Administer the oral dose at the same time every day c Assess carefully for excessive bruising or unusual bleeding d Monitor laboratory results for a target INR of 2 to 3 e Monitor laboratory results for a therapeutic aPTT value of 1.5 to 2.5 times the control value

b Administer the oral dose at the same time every day c Assess carefully for excessive bruising or unusual bleeding d Monitor laboratory results for a target INR of 2 to 3

2 While assessing a patient who is to receive muromonab-CD3, the nurse knows that which condition would be a contraindication for this drug? a Acute myalgia b Fluid overload c Polycythemia d Diabetes mellitus

b Fluid overload

5 A patient has received a double dose of heparin during surgery and is bleeding through the incision site. While the surgeons are working to stop the bleeding at the incision site, the nurse will prepare to take what action at this time? a Give IV vitamin K as an antidote b Give IV protamine sulfate as an antidote c Call the blood bank for an immediate platelet transfusion d Obtain an order for packed red blood cells

b Give IV protamine sulfate as an antidote

1 The nurse is administering antihypertensive drugs to older adult patients. The nurse knows that which adverse effect is of most concern for these patients? a Dry mouth b Hypotension c Restlessness d Constipation

b Hypotension

4 A patient is receiving dabigatran (Pradaxa), 150 mg twice daily, as part of treatment for atrial fibrillation. Which condition, if present, would be a concern if the patient were to receive this dose? a Asthma b Renal impairment c History of myocardial infarction d Elevated liver enzymes

b Renal impairment

20) When a patient has experienced extravasation of a peripheral infusion of dopamine, the nurse will inject the alpha blocker phentolamine (Regitine) into the area of extravasation and expect which effect? a Vasoconstriction b Vasodilation c Analgesia d Hypotension

b Vasodilation

16)F0or a patient receiving a vasoactive drug such as intravenous dopamine, which action by the nurse is most appropriate? a) Monitor the gravity drip infusion closely, and adjust as needed. b) Assess the patient's cardiac function by checking the radial pulse. c) Assess the intravenous site hourly for possible infiltration. d) Administer the drug by intravenous boluses according to the patient's blood pressure.

b) Assess the patient's cardiac function by checking the radial pulse.

18) When a drug is characterized as having a negative chronotropic effect, the nurse knows to expect which effect? a) Reduced blood pressure b) Decreased heart rate c) Decreased ectopic beats d) Increased force of cardiac contractions

b) Decreased heart rate

6 A patient is starting warfarin (Coumadin) therapy as part of treatment for atrial fibrillation. The nurse will follow which principles of warfarin therapy? (Select all that apply.) a Teach proper subcutaneous administration b Administer the oral dose at the same time every day c Assess carefully for excessive bruising or unusual bleeding d Monitor laboratory results for a target INR of 2 to 3 e Monitor laboratory results for a therapeutic aPTT value of 1.5 to 2.5 times the control value

b,c,d

6 The nurse is administering an IV solution that contains potassium chloride to a patient in the critical care unit who has a severely decreased serum potassium level. Which action(s) by the nurse are appropriate? (Select all that apply.) a Administer the potassium by slow IV bolus. b Administer the potassium at a rate no faster than 20 mEq/hr. c Monitor the patient's cardiac rhythm with a heart monitor. d Use an infusion pump for the administration of IV potassium chloride. e Administer the potassium IV push

b,c,d

2. When administering angiotensin-converting enzyme (ACE) inhibitors, the nurse keeps in mind that which are possible adverse effects? (Select all that apply.) a. Diarrhea b. Fatigue c. Restlessness d. Headaches e. A dry cough f. Tremors

b,d,e

6 The nurse is administering cholestyramine (Questran), a bile acid sequestrant. Which nursing intervention(s) is appropriate? (Select all that apply.) a Administering the drug on an empty stomach b Administering the drug with meals c Instructing the patient to follow a low-fiber diet while taking this drug d Instructing the patient to take a fiber supplement while taking this drug e Increasing fluid intake f Not administering this drug at the same time as other drugs

b,d,e,f

5 A patient has a new prescription for an ACE inhibitor. During a review of the patient's list of current medications, which would cause concern for a possible interaction with this new prescription? (Select all that apply.) a A benzodiazepine taken as needed for allergies b A potassium supplement taken daily c An oral anticoagulant taken daily d An opioid used for occasional severe pain e An NSAID taken as needed for headaches

b,e

14. During teaching of a patient who will be taking warfarin sodium (Coumadin) at home, which statement by the nurse is correct regarding over-the-counter drug use? a. "Choose nonsteroidal anti-inflammatory drugs as needed for pain relief." b. "Aspirin products may result in increased bleeding." c. "Vitamin E therapy is recommended to improve the effect of the warfarin." d. "Mineral oil is the laxative of choice while taking anticoagulants."

b. "Aspirin products may result in increased bleeding."

2. When administering angiotensin-converting enzyme (ACE) inhibitors, the nurse keeps in mind that which are possible adverse effects? (Select all that apply.) a. Diarrhea b. Fatigue c. Restlessness d. Headaches e. A dry cough f. Tremors

b. Fatigue d. Headaches e. A dry cough

19. The nurse is preparing a patient's morning medications and, upon reviewing the list of drugs, notes that the patient is to receive heparin 5,000 units and enoxaparin (Lovenox), both subcutaneously. What is the nurse's priority action at this time? a. Administer the drugs in separate sites. b. Hold the drugs and clarify the order with the prescriber. c. Administer the enoxaparin and hold the heparin. d. Check the patient's aPTT.

b. Hold the drugs and clarify the order with the prescriber.

17. During thrombolytic therapy the nurse monitors for bleeding. Which symptoms may indicate a serious bleeding problem? (Select all that apply.) a. Hypertension b. Hypotension c. Decreased level of consciousness d. Increased pulse rate e. Restlessness

b. Hypotension c. Decreased level of consciousness d. Increased pulse rate e. Restlessness

Carvedilol (Coreg)

beta blocker

Nadolol (Corgard)

beta blocker

atenolol (Tenormin)

beta blocker

metoprolol (Lopressor)

beta blocker

timolol (Blocadren)

beta blocker

These drugs end in olol

beta blockers

1 A patient has a new prescription for transdermal nitroglycerin patches. The nurse teaches the patient that these patches are most appropriately used for which reason? a To relieve exertional angina b To prevent palpitations c To prevent the occurrence of angina d To stop an episode of angina

c

1. A 46-year-old man has been taking clonidine for 5 months. For the last 2 months, his blood pressure has been normal. During this office visit, he tells the nurse that he would like to stop taking the drug. What is the nurse's best response? a. "I'm sure the doctor will stop it—your blood pressure is normal now." b. "Your doctor will probably have you stop taking the drug for a month, and then we'll see how you do." c. "This drug should not be stopped suddenly; let's talk to your doctor." d. "It's likely that you can stop the drug if you ex- ercise and avoid salty foods."

c

1. As part of treatment for early heart failure, a patient is started on an angiotensin-converting enzyme (ACE) inhibitor. The nurse will monitor the patient's laboratory work for which potential effect? a. Agranulocytosis b. Proteinuria c. Hyperkalemia d. Hypoglycemia

c

1. What is the purpose of antianginal drug therapy? a. To increase myocardial oxygen demand b. To increase blood flow to peripheral arteries c. To increase blood flow to ischemic cardiac muscle d. To decrease blood flow to ischemic cardiac muscle

c

2 A nurse with adequate knowledge about the administration of intravenous nitroglycerin will recognize that which statement is correct? a The intravenous form is given by IV push injection. b Because the intravenous forms are short-lived, the dosing must be every 2 hours. c Intravenous nitroglycerin must be protected from exposure to light through use of special tubing. d Intravenous nitroglycerin can be given via gravity drip infusions.

c

2 A patient is receiving instructions regarding warfarin therapy and asks the nurse about what medications she can take for headaches. The nurse will tell her to avoid which type of medication? a Opioids b acetaminophen (Tylenol) c NSAIDs d There are no restrictions while taking warfarin.

c

2 During assessment of a patient who is receiving digoxin, the nurse monitors for findings that would indicate an increased possibility of toxicity, such as: a apical pulse rate of 62 beats/min. b digoxin level of 1.5 ng/mL. c serum potassium level of 2.0 mEq/L. d serum calcium level of 9.9 mEq/L.

c

2 When administering beta blockers, the nurse will follow which guideline for administration and monitoring? a The drug may be discontinued at any time. b Postural hypotension rarely occurs with this drug. c Tapering off the medication is necessary to prevent rebound hypertension. d The patient needs to stop taking the medication at once if he or she gains 3 to 4 pounds in a week.

c

2 When preparing an IV solution that contains potassium, the nurse knows that a contraindication to the potassium infusion would be a diarrhea. b serum sodium level of 145 mEq/L. c serum potassium level of 5.6 mEq/L. d dehydration.

c

2. The nurse will teach a patient who will be taking ni- troglycerin (Nitrostat) about which common adverse effect of this drug? a. Blurred vision b. Dizziness c. Headache d. Weakness

c

3 For a patient receiving a vasoactive drug such as intravenous dopamine, which action by the nurse is most appropriate? a Monitor the gravity drip infusion closely, and adjust as needed. b Assess the patient's cardiac function by checking the radial pulse. c Assess the intravenous site hourly for possible infiltration. d Administer the drug by intravenous boluses according to the patient's blood pressure.

c

3 The nurse providing teaching for a patient who has a new prescription for beta1 blockers will keep in mind that these drugs may result in which effect? a Tachycardia b Tachypnea c Bradycardia d Bradypnea

c

3 When the nurse is checking the laboratory data for a patient taking spironolactone (Aldactone), which result would be a potential concern? a Serum sodium level of 140 mEq/L b Serum calcium level of 10.2 mg/dL c Serum potassium level of 5.8 mEq/L d Serum magnesium level of 2.0 mg/dL

c

3 Which point will the nurse emphasize to a patient who is taking an antilipemic medication in the "statin" class? a The drug needs to be taken on an empty stomach before meals. b A low-fat diet is not necessary while taking these medications. c It is important to report muscle pain immediately. d Improved cholesterol levels will be evident within 2 weeks.

c

3. A patient has a new prescription for a beta blocker as part of treatment for hypertension. The nurse is re- viewing the patient's current medications and notes that there may be a concern regarding interactions with which medication? a. Thyroid hormone supplement b. Antibiotic for a sinus infection c. Oral hypoglycemic for type 2 diabetes mellitus d. Oral contraceptive

c

4 A patient is being assessed before a newly ordered antilipemic medication is given. Which condition would be a potential contraindication? a Diabetes insipidus b Pulmonary fibrosis c Liver cirrhosis d Myocardial infarction

c

4 When a patient is being taught about the potential adverse effects of an ACE inhibitor, which of these effects should the nurse mention as possibly occurring when this drug is taken to treat hypertension? a Diarrhea b Nausea c Dry, nonproductive cough d Sedation

c

6. A patient has been taking tamsulosin (Flomax) for about a year. During today's office visit, he asks the physician about taking a drug for erectile dysfunc- tion. How does the nurse expect the physician to respond? a. "These drugs are safe to take together." b. "You can take them together, but the dosage of the Flomax will need to be reduced." c. "Taking these two drugs together may lead to dangerously low blood pressure." d. "You will be able to try taking these two drugs together, but watch for side effects."

c

4 During a teaching session for a patient receiving an immunosuppressant drug, the nurse will include which statement? a "It is better to use oral forms of these drugs to prevent the occurrence of thrush." b "You will remain on antibiotics to prevent infections." c "It is important to use some form of contraception during treatment and for up to 12 weeks after the end of therapy." d "Be sure to take your medications with grapefruit juice to increase absorption."

c "It is important to use some form of contraception during treatment and for up to 12 weeks after the end of therapy."

22) The nurse providing teaching for a patient who has a new prescription for beta1 blockers will keep in mind that these drugs may result in which effect? a Tachycardia b Tachypnea c Bradycardia d Bradypnea

c Bradycardia

4 When a patient is being taught about the potential adverse effects of an ACE inhibitor, which of these effects should the nurse mention as possibly occurring when this drug is taken to treat hypertension? a Diarrhea b Nausea c Dry, nonproductive cough d Sedation

c Dry, nonproductive cough

2 A patient is receiving instructions regarding warfarin therapy and asks the nurse about what medications she can take for headaches. The nurse will tell her to avoid which type of medication? a Opioids b acetaminophen (Tylenol) c NSAIDs d There are no restrictions while taking warfarin.

c NSAIDs

21) When administering beta blockers, the nurse will follow which guideline for administration and monitoring? a The drug may be discontinued at any time. b Postural hypotension rarely occurs with this drug. c Tapering off the medication is necessary to prevent rebound hypertension. d The patient needs to stop taking the medication at once if he or she gains 3 to 4 pounds in a week.

c Tapering off the medication is necessary to prevent rebound hypertension.

6. A patient who has heart failure will be started on an oral ACE inhibitor. While monitoring the patient's response to this drug therapy, which laboratory tests would be a priority? (Select all that apply.) a. White blood cell count b. Platelet count c. Serum potassium level d. Serum magnesium level e. Creatinine level f. Blood urea nitrogen (BUN)

c,e,f

1. A 46-year-old man has been taking clonidine for 5 months. For the last 2 months, his blood pressure has been normal. During this office visit, he tells the nurse that he would like to stop taking the drug. What is the nurse's best response? a. "I'm sure the doctor will stop it—your blood pressure is normal now." b. "Your doctor will probably have you stop taking the drug for a month, and then we'll see how you do." c. "This drug should not be stopped suddenly; let's talk to your doctor." d. "It's likely that you can stop the drug if you ex- ercise and avoid salty foods."

c. "This drug should not be stopped suddenly; let's talk to your doctor."

16. When administering subcutaneous heparin, the nurse will remember to perform which action? a. Use the same sites for injection to reduce trauma. b. Use a 1-inch needle for subcutaneous injections. c. Inject the medication without aspirating for blood return. d. Massage the site after the injection to increase absorption.

c. Inject the medication without aspirating for blood return.

Amlodipine (Norvasc)

calcium channel blocker

Nicardipine (Cardene)

calcium channel blocker

Nifedipine (Procardia)

calcium channel blocker

Verapamil (Calan)

calcium channel blocker

Diltiazem (Cardizem)

calcium channel blocker ****this was STRESSED****

Start at a low dose, increasing the dose slowly every 2 wk as tolerated by the patient If <50 bpm, withhold drug and call health care provider

carvedilol (Coreg)

Two beta blockers approved by the FDA to treat HF

carvedilol (an alpha blocker and nonselective beta blocker) and metoprolol

1 The nurse is reviewing the medications that have been ordered for a patient for whom a loop diuretic has just been prescribed. The loop diuretic may have a possible interaction with which of the following? a Vitamin D b warfarin c Penicillins d NSAIDs

d

1. What is another name for an adrenergic drug? a. Anticholinergic drug b. Parasympathetic drug c. Central nervous system drug d. Sympathomimetic drug

d

2. Before giving oral digoxin (Lanoxin), the nurse dis- covers that the patient's radial pulse is 52 beats/min. What will be the nurse's next action? a. Give the dose. b. Delay the dose until later. c. Hold the dose and notify the physician. d. Check the apical pulse for 1 minute.

d

4 A patient is receiving dobutamine for shock and is complaining of feeling more "skipping beats" than yesterday. What will the nurse do next? a Monitor for other signs of a therapeutic response to the drug. b Titrate the drug to a higher dose to reduce the palpitations. c Discontinue the dobutamine immediately. d Assess the patient's vital signs and cardiac rhythm.

d

4 A patient who has recently had a myocardial infarction (MI) has started therapy with a beta blocker. The nurse explains that the main purpose of the beta blocker for this patient is to a cause vasodilation of the coronary arteries. b prevent hypertension. c increase conduction through the SA node. d protect the heart from circulating catecholamines

d

4 The nurse is preparing an infusion for a patient who has a deficiency in clotting factors. Which type of infusion is most appropriate? a Albumin 5% b Packed RBCs c Whole blood d Fresh frozen plasma

d

4 While assessing a patient with angina who is to start beta blocker therapy, the nurse is aware that the presence of which condition may be a problem if these drugs are used? a Hypertension b Essential tremors c Exertional angina d Asthma

d

4. For a patient using transdermal nitroglycerin patches, the nurse knows that the prescriber will or- der which procedure for preventing tolerance? a. Leave the old patch on for 2 hours when apply- ing a new patch. b. Apply a new patch every other day. c. Leave the patch off for 24 hours once a week. d. Remove the patch at night for 8 hours and then apply a new patch in the morning.

d

5 A patient with diabetes has a new prescription for a thiazide diuretic. Which statement will the nurse include when teaching the patient about the thiazide drug? a "There is nothing for you to be concerned about when you are taking the thiazide diuretic." b "Be sure to avoid foods that are high in potassium." c "You need to take the thiazide at night to avoid interactions with the diabetes medicine." d "Monitor your blood glucose level closely, because the thiazide diuretic may cause the levels to increase."

d

5 Before initiating therapy with a nonselective beta blocker, the nurse will assess the patient for a history of which condition? a Hypertension b Liver disease c Pancreatitis d Asthma

d

5 The nurse is assessing a patient who is receiving a milrinone infusion and checks the patient's cardiac rhythm on the heart monitor. What adverse cardiac effect is most likely to occur in a patient who is receiving intravenous milrinone? a Tachycardia b Bradycardia c Atrial fibrillation d Ventricular dysrhythmia

d

5 While monitoring a patient who is receiving an infusion of a crystalloid solution, the nurse will monitor for which potential problem? a Bradycardia b Hypotension c Decreased skin turgor d Fluid overload

d

5. A patient in her eighth month of pregnancy has pre- eclampsia. Her blood pressure is 210/100 mm Hg this morning. This type of hypertension is classified as which of the following? a. Primary b. Idiopathic c. Essential d. Secondary

d

5. A patient who has been taking a beta blocker for 6 months tells the nurse during a follow-up visit that she wants to stop taking this medication. She is wondering if there is any problem with stopping the medication today. What is the nurse's best response? a. "No, there are no ill effects if this medication is stopped." b. "There should be only minimal effects if you stop this medication." c. "You may experience orthostatic hypotension if you stop this medication abruptly." d. "If you stop this medication suddenly, there is a possibility you may experience chest pain or rebound hypertension."

d

24)Before initiating therapy with a nonselective beta blocker, the nurse will assess the patient for a history of which condition? a Hypertension b Liver disease c Pancreatitis d Asthma

d Asthma

1 The nurse is monitoring a patient who is receiving antithrombolytic therapy in the emergency department because of a possible MI. Which adverse effect would be of the greatest concern at this time? a Dizziness b Blood pressure of 130/98 mm Hg c Slight bloody oozing from the IV insertion site d Irregular heart rhythm

d Irregular heart rhythm

3 During therapy with azathioprine (Imuran), the nurse must monitor for which common adverse effect? a Bradycardia b Diarrhea c Vomiting d Thrombocytopenia

d Thrombocytopenia

23)A patient who has recently had a myocardial infarction (MI) has started therapy with a beta blocker. The nurse explains that the main purpose of the beta blocker for this patient is to a cause vasodilation of the coronary arteries. b prevent hypertension. c increase conduction through the SA node. d protect the heart from circulating catecholamines.

d protect the heart from circulating catecholamines.

17) A patient is receiving dobutamine for shock and is complaining of feeling more "skipping beats" than yesterday. What will the nurse do next? a) Monitor for other signs of a therapeutic response to the drug. b) Titrate the drug to a higher dose to reduce the palpitations. c) Discontinue the dobutamine immediately. d) Assess the patient's vital signs and cardiac rhythm.

d) Assess the patient's vital signs and cardiac rhythm.

18. Which drug is most often used for deep vein thrombosis (DVT) prevention after major orthopedic surgery, even after the patient has gone home? a. Antiplatelet drugs, such as aspirin b. Adenosine diphosphate (ADP) inhibitors, such as clopidogrel (Plavix) c. Anticoagulants, such as warfarin sodium (Coumadin) d. Low-molecular-weight heparins, such as enoxaparin (Lovenox)

d. Low-molecular-weight heparins, such as enoxaparin (Lovenox)

5. A patient in her eighth month of pregnancy has pre- eclampsia. Her blood pressure is 210/100 mm Hg this morning. This type of hypertension is classified as which of the following? a. Primary b. Idiopathic c. Essential d. Secondary

d. Secondary

Store nitroglycerin in a ________ bottle and keep it in a dry place.

dark

First degree AV block

delayed conduction through the AV node causes prolonged PR interval

14) CCB negative inotropic and increase chronotropic used to treat A fib and A flutter, slows AV conduction-

diltizem (Cardizem)

24) Valve disorders-

diuretics, digoxin, antidysrhtmic, anti-coagulants (after valve replacement)

Make certain patient is not taking monoamine oxidase (MAO) inhibitors, tricyclic antidepressants, phenytoin (Dilantin), or haloperidol (Haldol). Watch for azotemia.

dobutamine (Dobutrex IV) dopamine (Intropin IV)

Premature Ventricular Contractions

extra, abnormal heartbeats that disrupt the regular ventricular rhythm of the heart

Second degree AV block

failure of some atrial action potentials to reach ventricles

7) Protamine sulfate is the antidote

for Heparin

20) Adenosine and metoprolol-

for Supraventricular tachycardia

6) Vit K is antidote

for warfarin (Coumadin)

14.Pregnancy-induced hypertension causes:

generalized vasospasm may be a factor, otherwise unknown

Grade 2 pitting edema

he pressure leaves an indentation of 3-4 mm that rebounds in fewer than 15 seconds.

MUGA scan

imaging the motion of heart wall muscles and assessing the function of the heart via a multiple-gated acquisition scan, which uses radioactive chemicals BEST for ejection fraction, most accurate

Orthopnea(1572)

in order to breath you need to sit or stand up

Pulse Scale +4

indicating a bounding pulse. Not obliterated with pressure

Pulse Scale +1

indicating a faint, but detectable pulse, easily obliterated with pressure

Pulse Scale +0

indicating no palpable pulse

Q wave

initial negative deflection produced by ventricular depolarization

Pulse Scale +3

is a NORMAL pulse; not easily obliterated with pressure

13.Coarctation of the aorta causes:

marked elevated blood pressure in upper extremities with decreased perfusion in lower extremities

Elevated ST segment

myocardial infarction

Depressed ST segment

myocardial ischemia

22) Angina-

nitro, beta blocker and calcium blocker

15) decrease afterload and preload-

nitroglycerin

tamponade

pathologic compression of an organ, such as the heart

Two drugs approved for use in the United States to specifically treat intermittent claudication are

pentoxifylline (Trental) and cilostazol (Pletal).

12) PEA

pulseless electrical activity

Chronotropic

rate of contraction

Dromotropic

rate of electrical conduction

3) Calcium-channels blockers-

relaxes vessels and heart contractions, ZEM, MIL and IPINE

Horizontal axis

represents time

Vertical axis

represents voltage


Kaugnay na mga set ng pag-aaral

Course 2: Law of Contracts Final Exam

View Set

Chapter 45 Hormones and Endocrine System Key Concepts

View Set

Algebra 2 Semester 1 Final Exam: Chapter 6.5

View Set

Civil Liberties & Civil Rights Quiz

View Set

LearningCurve 3b. Infancy and Childhood

View Set